+ All Categories
Home > Documents > AL Physics 1981-2004 MC

AL Physics 1981-2004 MC

Date post: 28-Nov-2014
Category:
Upload: olleh123
View: 199 times
Download: 3 times
Share this document with a friend
Popular Tags:
273
81 AL Physics/M.C./P.1 1981 Hong Kong Advanced Level Examination AL Physics Multiple Choice Question 1. If u and v represent speeds, x and y distances, t a time and a an acceleration, which of the following equations is dimensionally incorrect ? A. x² + y² = uvt² B. v² - u² = ax C. v = at + y/t D. ux = (x + y)/t E. av = (x - y)²/t³ 2. P Q R S X Y PQ = QR = RS = l m 4 3 2 m m m PQRS is a light, rigid rod with masses attached to it as shown in the diagram. The moment of inertia of the system about XY is A. 6 ml² B. 10 ml² C. 14 ml² D. 50 ml² E. 60 ml² 3. Which of the graphs below best represents the variation of the excess pressure P inside a soap bubble with its radius r? A. r P 0 B. r P 0 C. r P 0 D. r P 0 E. r P 0 4. The moon orbits the earth once every 27.3 days, with a mean orbital radius of R. What is the period of an earth satellite with an orbital radius of R/30? A. 4 hr B. 22 hr C. 68 hr D. 260 days E. 4500 days 5. A sphere of radius r and density ρ acquires a terminal speed v when it is dropped through air from a great height. Neglecting the buoyancy of the air, what is the terminal speed acquired by a sphere of radius 2 r and density 3 ρ under the same conditions? A. v/2 B. 3 v C. 6 v D. 12 v E. 24 v
Transcript
Page 1: AL Physics 1981-2004 MC

81 AL Physics/M.C./P.1

1981 Hong Kong Advanced Level Examination AL Physics

Multiple Choice Question

1. If u and v represent speeds, x and y distances, t a time and a an acceleration, which of the following equations is dimensionally incorrect?

A. x² + y² = uvt² B. v² - u² = ax C. v = at + y/t D. ux = (x + y)/t E. av = (x - y)²/t³ 2.

P Q R S

X

YPQ = QR = RS = l

m4 3 2m m m

PQRS is a light, rigid rod with masses attached

to it as shown in the diagram. The moment of inertia of the system about XY is

A. 6 ml² B. 10 ml² C. 14 ml² D. 50 ml² E. 60 ml² 3. Which of the graphs below best represents the

variation of the excess pressure P inside a soap bubble with its radius r?

A.

r

P

0 B.

r

P

0 C.

r

P

0 D.

r

P

0 E.

r

P

0 4. The moon orbits the earth once every 27.3

days, with a mean orbital radius of R. What is the period of an earth satellite with an orbital radius of R/30?

A. 4 hr B. 22 hr C. 68 hr D. 260 days E. 4500 days 5. A sphere of radius r and density ρ acquires a

terminal speed v when it is dropped through air from a great height. Neglecting the buoyancy of the air, what is the terminal speed acquired by a sphere of radius 2 r and density 3 ρ under the same conditions?

A. v/2 B. 3 v C. 6 v D. 12 v E. 24 v

Page 2: AL Physics 1981-2004 MC

81 AL Physics/M.C./P.2

6. One mole of an ideal gas expands isothermally at absolute temperature T from volume V1 to volume V2. The work done by the gas is

A. RT V V

V

( )2 1

1

B. RTV V

( )1 1

1 2−

C. RTV

V2

1

D. 2 2 1

1 2

RT V V

V V

( )

( )

+

E. RT V V(ln ln )2 1− 7.

i θ

A ray of light passes through a prism as shown.

As the angle i is increased from zero to 90º, the angle θ

A. remains constant B. passes through a minimum value C. passes through a maximum value D. decreases continuously E. increases continuously 8.

S L M

The diagram shows a converging lens L, of

focal length of magnitude F, a convex mirror M of focal length of magnitude f, and the position S at which an object is coincident with its own image. If SL = x and LM = y, then

A. 1 1 1

x y F+ =

B. 1 1 1

x y f F+

+=

C. 1 1

2

1

x y f F+

+=

D. y + F = 2f

E. xy F

f+ + =−( )1 1 1

9. The image of an object formed on a screen by

a thin converging lens has height a. By moving the lens towards the screen, a second lens position is found at which the height of the image formed on the screen is b. What is the height of the object?

A. 1

2( )a b+

B. 1

2ab

C. ab

D. a

b

3

E. b

a

3

10.

15 cm

object

A converging lens of focal length 15 cm is

used as a magnifying glass with the final image at infinity. If the least distance of distinct vision is 25 cm, the angular magnification achieved is

A. 0 B. 1 C. 5/3 D. 15 E. infinite 11. When two notes of nearly equal frequencies f1

and f2, with f2 > f1, are sounded together, beats are heard. Beats are periodic variations in

A. pitch, with beat frequency (f2 - f1). B. pitch, with beat frequency ½(f1 + f2). C. loudness, with beat frequency (f2 - f1). D. intensity, with beat frequency ½(f2 - f1).

Page 3: AL Physics 1981-2004 MC

81 AL Physics/M.C./P.3

E. intensity, with beat frequency ½(f1 + f2). 12. The diagram shows a mirror M which rotates

about an axis through X with constant angular velocity ω, and a spherical mirror Y whose centre of curvature is at X, where XY = r.

r

M

X

Y

ω A narrow parallel beam of light strikes M at X,

with angle of incidence θ , such that it is reflected along XY, then along YX and is again reflected by M, to form the emergent beam. What is the angle between the initial incident and final emergent beams? (c is the speed of light.)

A. ωr

c4

B. ωr

c2

C. ωr

c

D. 2ωr

c

E. 4ωr

c

13. A vertical wire 0.4 m long carries a current of

5 A in a magnetic field of flux density 10-3 T, which dips at an angle of 30º to the horizontal. The force on the wire is

A. 5.0 × 10-4 N B. 8.7 × 10-4 N C. 1.0 × 10-3 N D. 1.5 × 10-3 N E. 1.7 × 10-3 N 14. A thermionic diode is operated at 100 V and

the current through it is 30 mA. How many electrons reach the anode in one second?

Electronic charge = -1.6 × 10-19 C.

A. 1.875 × 1017 B. 3.000 × 1017 C. 9.375 × 1018 D. 1.875 × 1020 E. 3.000 × 1024 15. Two long vertical wires X and Y are at a short

distance apart, and the vertical plane containing them is at right angles to the earth’s magnetic field. When each wire carries the same current I, the force on X due to the earth’s field is Fe and the force on X due to the current in Y is Fc . If the current is now doubled in both wires, the resultant force on X is

A. F Fe c+ B. F Fe c+ 2 C. 2 2F Fe c+ D. 2 4F Fe c+ E. 4 4F Fe c+ 16. A galvanometer has a scale divided into 120

equal divisions. It has a current sensitivity of 10 divisions per milliampere, and a resistance of 4.0 ohms. What is its voltage sensitivity, in divisions per millivolt?

A. 0.25 B. 2.5 C. 4.0 D. 12 E. 30 17. The specific charge (charge per unit mass) of a

Zn2+ ion is 2.9 × 106 C/kg. A dry cell with a zinc anode has an e.m.f. of 1.53 V. When the cell drives a current round a circuit, zinc is removed from the anode. Assuming that the system is a perfectly efficient converter of energy, what is the electrical energy produced per kg of zinc?

A. 0.23 J B. 0.53 J C. 4.4 kJ D. 1.9 MJ E. 4.4 MJ 18. A demonstration Van de Graaff generator has a

hollow sphere of radius a supported by a pillar of resistance R. The belt conveys charge to the

Page 4: AL Physics 1981-2004 MC

81 AL Physics/M.C./P.4

sphere at a rate I. The maximum surface density of charge on the sphere is

A. 4 0πε aIR

B. ε0IR

a

C. ε0

2IR

a

D. IR

a4 0πε

E. IR

a4 02πε

19. A secondary coil of N turns and resistance 40

ohms is wound round the middle of a long solenoid of cross-sectional area A m² with n turns per metre carrying a current I amperes. A ballistic galvanometer of resistance 10 ohms and sensitivity k divisions per coulomb is connected in series with the secondary coil. Neglecting damping, how many divisions is the deflection of the galvanometer when the current in the solenoid is switched off?

(µ0 = the permeability of free space in henry

per metre.)

A. µ0

10

nNAIk

B. µ0

40

nNAIk

C. µ0

50

nNAIk

D. µ0

10

nNAI

k

E. µ0

50

nNAI

k

20.

120 V500 kΩ

Ω3 M

4 µF SP

Q

In the circuit shown, S is connected to P until

the capacitor is fully charged. Then S is switched to Q. What is the initial value of the current through the 3 MΩ resistor?

A. 2.50 × 10-6 A B. 3.33 × 10-5 A C. 3.75 × 10-5 A D. 4.00 × 10-5 A E. 4.80 × 10-4 A 21. A long solenoid uniformly wound with N turns

is of length l, cross-sectional area A and carries a current I. The total energy stored in the magnetic field inside the solenoid is given approximately by

A. µ0

2 2

2AI N

l

B. µ0

2 2

22AI N

l

C. µ0

2

2AIN

l

D. µ0

2

22

AIN

l

E. µ0

2 2

2

AI N

l

22.

G

X P

QR

The diagram shows a Wheatstone bridge

circuit. P, Q and R are resistance boxes whose resistance may be set at zero or any integral number of ohms, up to 1,000. X is an unknown resistance to be measured by adjusting R. With P and Q each set at 300 ohms, it is not possible to balance the bridge, and the current through the galvanometer is always in the same direction. Which one of the following explanations could account for this failure?

A. X is greater than 300 ohms. B. X is greater than 600 ohms. C. X is greater than 1000 ohms. D. X is not an integral number of ohms. E. X is less than 1 ohm. 23.

Page 5: AL Physics 1981-2004 MC

81 AL Physics/M.C./P.5

θ

Coil

magneticneedle

magneticnorth

A tangent galvanometer has a narrow circular

coil of radius r, with N turns. When a current I passes through the coil, the magnetic needle, constrained to move in a horizontal plane, sets at an angle θ to magnetic north, as shown. If the flux density of the earth’s magnetic field has magnitude B, and the horizontal component is Bh, then tan θ is equal to

A. 2

0

rB

NIh

µ

B. 2

0

rB

NIµ

C. 2 2

0

r B

NIµ

D. µ0

2

NI

rBh

E. µ0

22

NI

r Bh

24. A potential difference V is applied between

two large parallel plates, distance s apart. An electron, mass m, charge -e, starts from rest at the negative plate and travels across the gap to the positive plate. The time taken is

A. 2s

V

B. s

V

2

C. 2 2s

eV

D. 2 2ms

eV

E. m

eV2

25. The armature of a d.c. electric motor has

resistance 2 Ω . When the applied potential

difference is 150 V, a current of 5 A flows. The back e.m.f. is

A. 10 V B. 93 V C. 140 V D. 150 V E. 160 V 26. C1 and C2 are plane parallel plate capacitors of

the same area. The plates of C1 and C2 are separated by slabs of the same dielectric material of thickness d and 2d respectively. The capacitance of C1 is 0.12 µF. The capacitance of C1 and C2 connected together in series is

A. 0.04 µF B. 0.08 µF C. 0.18 µF D. 0.24 µF E. 0.36 µF 27. Given: the mass of a proton is 1.6733 × 10-27 kg the mass of a neutron is 1.6744 × 10-27 kg the mass of an alpha particle is 6.6443 × 10-27 kg the electronic charge e is -1.6 × 10-19 C the speed of light in vacuo is 3.00 × 108 m/s The binding energy per nucleon in a helium

nucleus is A. 1.15 × 10-12 J B. 2.30 × 10-12 J C. 4.60 × 10-12 J D. 1.44 × 107 eV E. 2.87 × 107 eV 28. In a Millikan experiment, a droplet of oil of

radius r with two electronic charges, 2e, remains stationary when a potential difference V is applied to the plates. When the potential difference is increased to 4V, a droplet of oil of radius 2r remains stationary. What is the charge on the second droplet?

A. e/2 B. e C. 2 e

Page 6: AL Physics 1981-2004 MC

81 AL Physics/M.C./P.6

D. 4 e E. 8 e 29. For the photoelectic effect, which of the

following is the correct relationship between the energy E of a photon, the work function w of the surface which it strikes, and the maximum kinetic energy K of the emitted photoelectron?

A. E = w + K B. E = w - K C. E = K - w D. K = 2(w + E) E. w = ½(K + E) 30. An electron of mass m and charge e, is

accelerated by a potential V, then strikes an atom, exciting it from its ground state to a higher energy state. The electron is scattered with speed u, and the excited atom subsequently decays back to the ground state with the emission of a photon of frequency f. If h is the Planck constant, the value of u is

A. eV - hf B. 2(eV + hf) C. 2(eV - hf)/m

D. eV hf

m

+

E. 2( )eV hf

m

31. A cathode ray tube has X and Y sensitivities of

5 V/cm and 10 V/cm respectively. A sinusoidal p.d. of frequency 100 Hz and amplitude 7.5 V is applied to the X plates, and a sinusoidal p.d. of frequency 50 Hz and amplitude 15 V is applied to the Y plates. Which one of the following sketches could represent the pattern seen on the screen?

A.

1.5

1.5

cm

cm

B.

3.0

1.5

cm

cm

C.

cm

cm

1.5

3 √ 2

√ 2

D.

1.5

1.5

cm

cm

E.

1.5

3.0cm

cm

32.

f0

α

β

γ

δ

A series circuit consisting of a pure inductor L,

a pure capacitor C and a pure resistor R is connected across an a.c. supply. The variations with applied frequency f of the resistance R, the reactance XC of the capacitor,

Page 7: AL Physics 1981-2004 MC

81 AL Physics/M.C./P.7

the reactance XL of the inductor and the impedance Z of the circuit are represented in the diagram. Which of the following sequences places the curves in an order so that they represent the magnitudes of R, XC , XL and Z?

A. αδβγ B. βδαγ C. γαβδ D. γβδα E. γδβα 33. A stationary radioactive nucleus of mass N

units emits an alpha particle of mass 4 units, leaving a residual nucleus of mass (N - 4) units. The ratio of the kinetic energy of the alpha particle to the kinetic energy of the residual nucleus is

A. N − 4

4

B. N

N

2

24( )−

C. ( )N

N

− 4 2

2

D. ( )N − 4

4

2

2

E. ( )N − 4

4

3

3

34. Projectiles X and Y are launched

simultaneously from the top of a cliff. X is launched horizontally with speed 20 m/s, and Y is launched at an angle of 60º above the horizontal with speed 40 m/s. (The motion of both X and Y is in the same plane.) Which of the following statements is correct?

(1) X and Y travel equal vertical distances in

equal times. (2) X and Y travel equal horizontal distances

in equal times. (3) X and Y never meet. A. (1), (2) and (3) B. (1) and (2) only C. (2) and (3) only D. (1) only E. (3) only

35. Before the start of a race, the momentum of each competitor is less than his momentum during the race. Which of the following statements is correct?

(1) This situation violates Newton’s law of

conservation of momentum. (2) The law of conservation of momentum

applies only to collisions between two objects.

(3) A force acts on each competitor to increase his momentum as he starts to race.

A. (1), (2) and (3) B. (1) and (2) only C. (2) and (3) only D. (1) only E. (3) only 36. An astronaut in an orbiting satellite is

sometimes said to be ‘weightless’. This condition occurs when

(1) the gravitational attraction between the

astronaut and the earth is just sufficient to provide the centripetal force which keeps him in orbit.

(2) there is no reaction of the floor of the satellite on the astronaut’s feet.

(3) the gravitational pull of the earth is exactly cancelled by the gravitational pull of the moon.

A. (1), (2) and (3) B. (1) and (2) only C. (2) and (3) only D. (1) only E. (3) only 37. An ideal gas is enclosed in a container at

absolute temperature T. The absolute temperature is now raised to 2T, whilst the volume is kept constant. Which of the following statements is correct?

(1) The average separation of the molecules is

reduced by a factor of √2. (2) The average speed of the molecules is

doubled. (3) The average kinetic energy of the

molecules is doubled. A. (1), (2) and (3) B. (1) and (2) only C. (2) and (3) only D. (1) only

Page 8: AL Physics 1981-2004 MC

81 AL Physics/M.C./P.8

E. (3) only 38. The equation of state for mass m of an ideal

gas may be written pV = mrT. With reference to this equation,

(1) the value of r depends upon the particular

gas used. (2) r is independent of m. (3) if R is the molar gas constant, then the

mass of each mole of this gas is R/r. A. (1), (2) and (3) B. (1) and (2) only C. (2) and (3) only D. (1) only E. (3) only 39. The specific heat capacity of a liquid may be

determined by heating it in an electrical calorimeter or by using a continuous flow method. The advantage of the continuous flow method is that

(1) the required temperatures are recorded

when a steady state has been reached. (2) it is not necessary to measure the specific

heat capacity of the apparatus. (3) no heat is lost to the surroundings. A. (1), (2) and (3) B. (1) and (2) only C. (2) and (3) only D. (1) only E. (3) only 40.

20 cm 40 cm

X Y

80 oC θ

oC 20

oC

The diagram shows an unlagged bar XY whose

ends are maintained at different temperatures, with three thermometers, registering 80 ºC, θ ºC and 20 ºC in the steady state. (The room temperature is below 20 ºC.) Which of the following statements is correct?

(1) The rate at which heat energy crosses any section of the bar decreases from X to Y.

(2) (80 - θ) is greater than ½(θ - 20). (3) θ must be less than 40. A. (1), (2) and (3) B. (1) and (2) only C. (2) and (3) only D. (1) only E. (3) only 41. A mixture of saturated water vapour and water

is heated in a closed container. As the temperature rises,

(1) the proportion of vapour to liquid

decreases. (2) the density of the saturated vapour

increases. (3) the saturated vapour pressure of the water

increases. A. (1), (2) and (3) B. (1) and (2) only C. (2) and (3) only D. (1) only E. (3) only 42. An enclosure contains a black body X which is

in thermal equilibrium with it. A second black body Y, at a higher temperature than X, is introduced into the enclosure. The temperature of the enclosure is maintained at a constant value, and all heat exchange is by radiation. Which of the following statements is correct?

(1) The temperature of Y falls until

equilibrium is re-established. (2) The temperature of X rises initially then

falls until equilibrium is re-established. (3) Equilibrium is re-established with both X

and Y at a higher temperature than the enclosure.

A. (1), (2) and (3) B. (1) and (2) only C. (2) and (3) only D. (1) only E. (3) only 43. In a Young's slits experiment to produce

interference fringes on a screen, the separation of the interference fringes is increased by increasing

Page 9: AL Physics 1981-2004 MC

81 AL Physics/M.C./P.9

(1) the distance between the slits and the screen.

(2) the wavelength of the light. (3) the distance between the source and the

slits. A. (1), (2) and (3) B. (1) and (2) only C. (2) and (3) only D. (1) only E. (3) only 44. A free electron travelling horizontally with

speed v enters a uniform vertical magnetic field B. Which of the following statements is correct?

(1) The path of the electron is a vertical circle. (2) The speed of the electron remains

constant. (3) The radius of curvature of the path is

inversely proportional to B. A. (1), (2) and (3) B. (1) and (2) only C. (2) and (3) only D. (1) only E. (3) only 45. S is a source of alternating voltage. In which

of the following circuits will the current flowing through component X be rectified, i.e. flowing in one direction only?

(1)

SX

(2)

XS

(3)

XS

A. (1), (2) and (3) B. (1) and (2) only C. (2) and (3) only D. (1) only E. (3) only 46.

E

B

O

A

eyepiecefocal length

objectivefocal length

f e

f o

The diagram represents a microscope with the

object at A. The image of A in the objective is formed at B. Which of the following statements is correct?

(1) AO is less than fo. (2) BE is less than fe. (3) The image at B is real. A. (1), (2) and (3) B. (1) and (2) only C. (2) and (3) only D. (1) only E. (3) only

- End of Paper -

Page 10: AL Physics 1981-2004 MC

81 AL Physics/M.C./P.10

Question No. Key 1. D 2. D 3. B 4. A 5. D 6. E 7. D 8. C 9. C 10. C 11. C 12. E 13. E 14. A 15. D 16. B 17. E 18. B 19. C 20. D 21. E 22. C 23. D 24. D 25. C

Question No. Key 26. A 27. A 28. D 29. A 30. E 31. D 32. E 33. A 34. C 35. E 36. B 37. E 38. A 39. B 40. B 41. C 42. B 43. B 44. C 45. A 46. C

Page 11: AL Physics 1981-2004 MC

82 AL Physics/M.C./P.1

1982 Hong Kong Advanced Level Examination AL Physics

Multiple Choice Question

1. A solid of mass m starts from rest and travels for a given time under the action of a given force. The speed which it acquires is

A. proportional to m. B. proportional to 1/m. C. proportional to √m. D. proportional to 1/√m. E. independent of m. 2. A communication satellite in a circular orbit of

radius R has a period of 24 hours. The period of a satellite in a circular orbit of radius R/4 is

A. 3 hours B. 6 hours C. 12 hours D. 24 hours E. 96 hours 3. Two objects of masses m and 4 m move

towards each other along a straight line with kinetic energies E and 4 E respectively. The total linear momentum of both masses taken together is

A. 3 2mE B. 4 2mE C. 5 2mE D. 15 2mE E. 17 2mE 4. A builder using a pulley system to lift a bucket

of cement of weight 150 N exerts a steady force F and pulls 30 m of rope through the system in order to raise the bucket 10 m. The friction in the system is small but NOT negligible. The value of the force F is most probably

A. less than 50 N. B. exactly equal to 50 N. C. between 50 N and 150 N. D. between 150 N and 450 N. E. more than 450 N.

5. A stone of weight w tied to a piece of string is swung in a vertical circle. At the topmost point of its path, the tension in the string is T and the centripetal force is F. Which of the following statements is true?

A. F = w + T. B. F = w - T. C. The net force acting downwards on the

stone is F + T + w. D. The net force acting downwards on the

stone is F - T + w. E. The net force acting downwards on the

stone is F - T - w. 6.

5 d5 d

dd

mm

m m3 3

P

Q

X

Y

X and Y are dumbell shaped objects comprising

two equal small masses rigidly joined together by a light rod. I1 and I2 are the moments of inertia of X and Y respectively about the axis PQ. I2/I1 is equal to

A. 9 B. 15 C. 25 D. 45 E. 75 7.

6 m

4 m/s

3 m/sYX

Two bodies X and Y are moving with constant

velocities in the directions indicated by the arrows. At time t = 0, they are at the positions shown. At time t = 2 s, the magnitude of the velocity of Y relative to X is

A. 4 m/s.

Page 12: AL Physics 1981-2004 MC

82 AL Physics/M.C./P.2

B. 5 m/s. C. 6 m/s. D. 8 m/s. E. 10 m/s. 8. In an experiment to measure the specific latent

heat of vaporisation of water, a potential difference V is applied across a heating coil immersed in boiling water, and the mass of water vaporised per second, m, is measured. Heat loss to the surroundings is negligible. If the potential difference is increased to 4 V, the mass vaporised per second is

A. 2 m. B. 4 m. C. 8 m. D. 16 m. E. 64 m. 9. When the temperature of a solid is increased,

what will happen to the average kinetic energy and the average potential energy of the molecules?

average kinetic

energy average potential

energy A. increases increases B. increases no change C. increases decreases D. no change increases E. no change decreases

10.

X P Q Y

(diagram not to scale) PQ is a metal bar with thin layers of plastic XP

and QY attached to each end. The whole apparatus is well lagged. X is maintained at 100 ºC and Y at 0 ºC. Which of the graphs below best represents the variation of temperature T along XY?

A.

YX

T

B.

YX

T

C.

YX

T

D.

YX

T

E.

YX

T

11. A mass m of an ideal gas initially has volume

V0 and temperature T0. When it is kept at constant volume V0, heat Q is required to increase its temperature to T0 + ∆T. If the volume is not kept constant, but the gas expands from V0 to 3V0 when the temperature increases from T0 to T0 + 2∆T, then the heat supplied must be

A. less than 2 Q. B. equal to 2 Q. C. more than 2 Q. D. more than 3 Q. E. more than 6 Q. 12. Mass M of an ideal gas at pressure p occupies

volume V. The r.m.s. speed of its molecules is A. pV M/ 3 . B. pV M/ . C. 3pV M/ . D. pVM .

Page 13: AL Physics 1981-2004 MC

82 AL Physics/M.C./P.3

E. 3pVM . 13.

100 cm55 cm

L 1 L 2

O

L1 and L2 are converging lenses, each of focal

length 50 cm. The final image of object O, formed by refraction through both lenses, is at I. The distance OI is

A. 54.5 cm. B. 110 cm. C. 200 cm. D. 550 cm. E. 605 cm. 14. When a diver looks up towards the surface

from underwater, the surface behaves like A. a converging lens. B. a diverging lens. C. a plane mirror covering the entire surface. D. a plane mirror with a circular hole in it,

centred above the diver’s head. E. a circular plane mirror of limited extent,

centred above the diver’s head. 15. A near-sighted person’s greatest distance of

distinct vision is 0.9 m. His sight is improved by wearing spectacles which increase his greatest distance of distinct vision to 18 m. What is the magnitude of the focal length of the spectacle lenses?

A. 0.86 m. B. 0.90 m. C. 0.95 m. D. 17.1 m. E. 18.9 m. 16. A plane mirror is rotating with angular speed

ω. When pulses of light, of pulse frequency f, strike the mirror, they are reflected such that the angle between successive reflected pulses is θ. If the direction of the incident pulses remains constant, the value of θ is

A. ω

π f

B. ω

π2 f

C. ω

π4 f

D. ω

f

E. 2ω

f

17. A diffraction grating ruled with 5 000 lines per

cm is illuminated normally by white light. If the wavelengths for yellow light and violet light are 600 nm and 400 nm respectively, which of the following statements is false?

A. The central image is white. B. The violet end of the first-order spectrum

is closer to the central image than is the red end of the first-order spectrum.

C. The second-order image of yellow light coincides with the third-order image of violet light.

D. The angular displacement of the second-order image of yellow light from the central image is sin-1 0.6.

E. There is no fourth-order image for violet light.

18.

charged particles

Bcentre ofscreen

1.5 cm3 cm

A beam of charged particles passes through a

vacuum chamber and strikes the centre of the fluorescent screen at the end of the chamber. When a magnetic field B is applied in the direction shown, two spots appear on the screen, one 1.5 cm above the centre, and one 3 cm above the centre.

Which of the following can be possible

constituents of the beam? A. electrons and protons, travelling at the

same speed B. electrons and alpha particles, travelling at

the same speed C. electrons of speed v and protons of speed

2v D. protons and alpha particles, travelling at

the same speed

Page 14: AL Physics 1981-2004 MC

82 AL Physics/M.C./P.4

E. protons travelling with speed v and alpha particles travelling with speed 2v

19.

P Q

R S

X Y

X and Y are identical flexible conducting

ribbons, suspended from points P and Q. The bottom parts of the ribbons lie in dishes of a conducting liquid at R and S. When a current 2I is passed from R to P through X, and a current I is passed from Q to S through Y, which of the diagrams best represents the shapes assumed by the ribbons (if the ribbons remain in contact with the conducting liquid at R and S)?

A.

P Q

B.

P Q

C.

P Q

D.

P Q

E.

P Q

20.

YX= 1 ΩR1

= 3 ΩR2

In the circuit above, an electron travelling from

Y to X through R1 loses energy E1, and an electron travelling from Y to X through R2 loses energy E2. E1 is equal to

A. E2/4. B. E2/3. C. E2. D. 3E2. E. 4E2. 21. An inductor L, a resistor R and a switch S are

connected in series across a source of alternating e.m.f. E. Initially the switch is closed, as shown in the figure below:

E

SRL

The switch S is now opened, and at the instant

it is opened, a spark passes across it. This is because

A. the breakdown potential of the air is less

than E. B. at the instant when the switch is opened

the whole of the e.m.f. E is across L. C. the rate of change of the current through

the circuit is very large when the switch is opened.

D. the self-inductance of the circuit generates an e.m.f. equal to E.

E. the self-inductance of the circuit generates an e.m.f. which is proportional to the current flowing in the circuit.

Page 15: AL Physics 1981-2004 MC

82 AL Physics/M.C./P.5

22.

C

A sinusoidal potential difference of fixed

frequency and variable amplitude V is applied across a capacitor of capacitance C. Which of the graphs below best represents the variation of the amplitude I of the current in the circuit with V?

A.

V

I

0 B.

V

I

0 C.

V

I

0 D.

V

I

0 E.

V

I

0 23. A ballistic galvanometer of resistance R1 is

connected in series with a search coil of resistance R2 (the resistance of the connecting

wires is negligible). When the search coil is removed from a magnetic field, the charge which passes through the ballistic galvanometer is proportional to

A. R2. B. (R1 + R2). C. 1/R1. D. 1/R2. E. 1/(R1 + R2). 24. In a circuit with a vibrating reed switch, a 2 µF

capacitor is charged from a battery of e.m.f. 12 V and completely discharged through a resistor with a frequency of 25 Hz. The average power dissipated as heat in the resistor is

A. 24 µW. B. 57.6 µW. C. 144 µW. D. 0.6 mW. E. 3.6 mW. 25.

S

L 2

L1

S

L1

L2

S is a long, current-carrying solenoid. L1 is a

wire loop just inside the solenoid, and L2 is a wire loop just outside the solenoid. The current in the solenoid is increased at a steady rate, such that the e.m.f. induced in L1 is 1.2 V. The e.m.f. induced in L2 is approximately

A. 0 V. B. 0.6 V. C. 1.2 V. D. 2.4 V. E. larger than 2.4 V. 26.

1 2 3 4 5 6 7 8 9 10 110

8

V / V

t / ms

Page 16: AL Physics 1981-2004 MC

82 AL Physics/M.C./P.6

The diagram shows the waveform of an alternating p.d. V applied across a resistor R. What is the value of the steady p.d. which should be applied across R to give the same heating effect?

A. 0 V B. 16/3 V C. 4√2 V D. 8 2 3/ V E. 8 V 27.

EE 2 3

4

1

5v

The diagram shows an electron in a uniform,

vertical electric field E. At the instant shown, the electron is travelling in the direction of the arrow v. Which of the arrows 1 to 5 gives the direction of the acceleration of the electron at this instant?

A. 1 B. 2 C. 3 D. 4 E. 5 28.

s

xP

A B

+ _

The figure above shows a charged parallel

plate capacitor. P is a point between the plates and its distance from plate A is x. If E is the electric field at point P, which of the following graphs best represents the variation of E with x? (The separation of the plates is s.)

A.

x

E

0s

B.

x

E

0s

C.

x

E

0s

D.

x

E

0s

E.

x

E

0s

29.

V1

cathode anode

e - e -BV2

In Thomson’s method of measuring e/m,

electrons are accelerated by a potential difference V1, then passed undeviated through a region with a magnetic flux density B and an electric field E. E is provided by a potential difference V2 between the two plates. If V1 is increased to 16 V1, in order for the electron

Page 17: AL Physics 1981-2004 MC

82 AL Physics/M.C./P.7

beam to remain undeviated, V2 must be changed to

A. 4 V2. B. 8 V2. C. 16 V2. D. 32 V2. E. 256 V2. 30. With the time base switched off, a cathode ray

oscilloscope can be used to display graphically A. the sawtooth waveform of the timebase. B. the sinusoidal waveform of the mains

supply. C. the variation of the potential difference

between the X plates with time. D. the variation of the potential difference

between the Y plates with time. E. the variation of the electric field between

the Y plates with the electric field between the X plates.

31. In an X-ray tube, an accelerating potential V

produces X-rays of minimum wavelength 1.2 × 10-10 m. In order to produce X-rays of minimum wavelength 0.6 × 10-10 m, the accelerating potential should be

A. V/4. B. V/2. C. V √2. D. 2 V. E. 4 V. 32. Which of the following graphs best represents

the variation of the maximum velocity v of photoelectrons emitted from a target with the frequency f of the incident light?

A.

f

v

0 B.

f

v

0

C.

f

v

0 D.

f

v

0 E.

f

v

0 33. If h is the Planck constant and c is the speed of

light in a vacuum, photons of wavelength λ have momentum p. Which of the following expressions is dimensionally correct for p?

A. hλ B. h/λ C. hc/λ D. hλ/c E. hcλ 34. In a Bainbridge mass spectrometer, an ion of

mass m and charge Q travels in a circular path of radius r. The path of an ion of mass 3m and charge 2Q has radius

A. r/6. B. r/3. C. 2 r/3. D. 3 r/2. E. 6 r. 35. A radioactive source with a half-life t½ initially

contains N atoms of the radioactive element. The energy released in each disintegration is E. What is the total energy released in time 2 t½?

A. ¼ NE B. ½ NE

Page 18: AL Physics 1981-2004 MC

82 AL Physics/M.C./P.8

C. ¾ NE D. ½ NEt½ E. NEt½ 36. Two copper wires, X and Y, are suspended

vertically, and the same downward vertical force F is applied to the lower end of each wire. The extension of X is twice the extension of Y. Which of the following may account for this difference?

(1) The diameter of X is half the diameter of

Y, but their lengths are equal. (2) X is twice as long as Y, but their diameters

are equal. (3) Y is twice as long as X, and its diameter is

twice that of X. A. (1), (2) and (3) B. (1) and (2) only C. (2) and (3) only D. (1) only E. (3) only 37. A bob of mass m supported by an elastic string

performs simple harmonic oscillations in a vertical plane. The tension in the string

(1) is independent of m. (2) is proportional to the amplitude of the

oscillations. (3) has its maximum value when the bob is at

its lowest point. A. (1), (2) and (3) B. (1) and (2) only C. (2) and (3) only D. (1) only E. (3) only 38. ‘The volume of a fixed mass of an ideal gas at

constant pressure is proportional to its temperature.’ The statement above is true only when

(1) an absolute temperature scale is used. (2) the gas is at a very low pressure. (3) the gas is at a very high pressure. A. (1), (2) and (3) B. (1) and (2) only C. (2) and (3) only D. (1) only E. (3) only

39. Which of the following statements about vapours is/are correct?

(1) Saturated vapour pressure is independent

of temperature. (2) If the volume of a closed vessel containing

saturated vapour is halved, the pressure will be doubled.

(3) The pressure inside a closed vessel containing air, water and water vapour increases with temperature.

A. (1), (2) and (3) B. (1) and (2) only C. (2) and (3) only D. (1) only E. (3) only 40. The critical temperature of a substance is the

temperature (1) below which the substance in its gaseous

phase can be liquified by the application of sufficient pressure.

(2) at which the densities of its liquid and saturated vapour are equal.

(3) at which solid, liquid and gaseous phases of the substance can exist together in equilibrium.

A. (1), (2) and (3) B. (1) and (2) only C. (2) and (3) only D. (1) only E. (3) only 41.

S1

S2

The figure above shows each of the stationary

traces S1 and S2 observed on the screen of a cathode ray oscilloscope connected to a microphone, when two tuning forks F1 and F2 respectively are sounded in turn. (The time base remains the same in each case.) Which of the following statements is/are correct?

(1) The period of F1 is greater than the period

of F2. (2) The pitch of F1 is greater than the pitch of

F2.

Page 19: AL Physics 1981-2004 MC

82 AL Physics/M.C./P.9

(3) The speed of sound from F1 is greater than the speed of sound from F2.

A. (1), (2) and (3) B. (1) and (2) only C. (2) and (3) only D. (1) only E. (3) only 42. Which of the following statements about

electromagnetic radiation is/are correct? (σ = Stefan constant)

(1) A black body at a given temperature T

radiates equal amounts of power for all wavelength bands of width δλ.

(2) If T increases to (T + δT), the emitted power of a black body increases from P to (P + δP), where δP = σ(δT)4.

(3) The wavelength λm for maximum emissive power from a black body is inversely proportional to temperature T.

A. (1), (2) and (3) B. (1) and (2) only C. (2) and (3) only D. (1) only E. (3) only 43. A very long solenoid with a metallic core has a

radius r and n turns per unit length. It carries a current I. The magnetic flux density B on its axis is

(1) independent of r. (2) proportional to n. (3) independent of the material of the core. A. (1), (2) and (3) B. (1) and (2) only C. (2) and (3) only D. (1) only E. (3) only 44.

P YX

E R

R E G2 2

1 1

The diagram shows a potentiometer circuit. Which of the following statements is/are correct?

(1) If the e.m.f. E1 drops, the balance point P

will be closer to Y. (2) If the e.m.f. E2 drops, the balance point P

will be closer to X. (3) If R2 is increased, the balance point P will

be closer to Y. A. (1), (2) and (3) B. (1) and (2) only C. (2) and (3) only D. (1) only E. (3) only 45. In the electrolysis of water acidulated with

dilute sulphuric acid, using platinum as electrodes,

(1) hydrogen is evolved at the cathode. (2) the concentration of sulphuric acid

decreases with time. (3) the voltameter obeys Ohm’s Law. A. (1), (2) and (3) B. (1) and (2) only C. (2) and (3) only D. (1) only E. (3) only 46.

G

P S

RQ

The figure above shows 4 resistors, of

resistance P, Q, R and S in a Wheatstone Bridge circuit. Which of the following statements is/are correct when the bridge is balanced?

(1) The potential difference across P is equal

to the potential difference across Q. (2) P/Q = S/R (3) P/(Q + R) = Q/(P + S) A. (1), (2) and (3) B. (1) and (2) only C. (2) and (3) only

Page 20: AL Physics 1981-2004 MC

82 AL Physics/M.C./P.10

D. (1) only E. (3) only 47.

V

X

R

In the circuit above, V is an a.c. source, X is an

air-cored solenoid and R is a heating coil used to boil some liquid in a vessel. Which of the following adjustments would decrease the time required to boil the liquid?

(1) The frequency of the a.c. supply is

increased. (2) A soft iron cylinder is inserted into the

solenoid. (3) A suitable capacitor is added in series with

the circuit. A. (1), (2) and (3) B. (1) and (2) only C. (2) and (3) only D. (1) only E. (3) only 48. Which of the following statements about a

thermionic diode is/are correct? (1) A thermionic diode can be used to convert

a.c. into d.c. (2) Increasing the filament current will

increase the saturation current. (3) The current flow to the anode increases

linearly with the anode voltage within a certain range.

A. (1), (2) and (3) B. (1) and (2) only C. (2) and (3) only D. (1) only E. (3) only 49. An electron travelling with velocity v enters a

region with a uniform electric field E and a

uniform magnetic flux density B. E, B and v are mutually perpendicular, as shown in the diagram.

B into the paper

v

E

Which of the following statements is/are

correct? (1) When B = 0, the path of the electron is

parabolic. (2) When E = 0, the path of the electron is

circular. (3) The curvature of the electron’s path is

independent of v. A. (1), (2) and (3) B. (1) and (2) only C. (2) and (3) only D. (1) only E. (3) only 50. In the Rutherford experiment on alpha particle

scattering, an alpha particle strikes a thin metal foil X and is deflected through an angle θ.

θ

X

α

α

Which of the following statements is/are true? (1) θ cannot be greater than 90º. (2) The closer the alpha particle approaches to

a nucleus, the greater is the value of θ. (3) For the same approach path, θ is larger for

slower alpha particles than for faster ones. A. (1), (2) and (3) B. (1) and (2) only C. (2) and (3) only D. (1) only E. (3) only

- End of Paper -

Page 21: AL Physics 1981-2004 MC

82 AL Physics/M.C./P.11

Question No. Key 1. B 2. A 3. A 4. C 5. A 6. E 7. B 8. D 9. A 10. B 11. C 12. C 13. C 14. D 15. C 16. E 17. E 18. D 19. A 20. C 21. C 22. A 23. E 24. E 25. C

Question No. Key 26. D 27. E 28. A 29. A 30. E 31. D 32. C 33. B 34. D 35. C 36. C 37. E 38. D 39. E 40. B 41. D 42. E 43. B 44. B 45. D 46. B 47. E 48. A 49. B 50. C

Page 22: AL Physics 1981-2004 MC

83 AL Physics/M.C./P.1

1983 Hong Kong Advanced Level Examination AL Physics

Multiple Choice Question

1. A ball bounces up and down from the floor. Which of the following graphs shows the variation of its velocity v with time t?

A.

t

v

0 B.

t

v

0

C.

t

v

0

D.

t

v

0

E.

t

v

0 2. A man is walking due east at 1.00 m/s on the

deck of a ship streaming north at 1.73 m/s. In

what direction will the man be walking relative to the surface of the earth?

A. N 30º E B. N 60º E C. NE D. N 26.9º E E. N 40.9º E 3.

P

I IIIII

IV

V

A boy is whirling a stone, tied to a piece of

string, in a vertical circle as shown above. The string suddenly breaks at P. Which of the paths (I - V) represents a possible path for the stone from just before the string breaks until the stone hits the ground?

A. I B. II C. III D. IV E. V 4. A spaceship burns fuel and moves with

constant acceleration in a straight line. Which of the graphs below best represents the variation of its momentum p with time t?

A.

t

p

0 B.

Page 23: AL Physics 1981-2004 MC

83 AL Physics/M.C./P.2

t

p

0 C.

t

p

0 D.

t

p

0 E.

t

p

0 5.

v

t0

The velocity v of a particle varies with time t as

shown. Which of the following graphs best represents the variation of the displacement s of the particle with time t?

A.

s

t0

B.

s

t0

C.

s

t0 D.

s

t0 E.

s

t0 6.

a5a

a3

4

Z

X Y

XYZ is a rigid framework of rods of negligible

mass. Three small bodies, each of mass M, are attached to the framework, one at each of the points X, Y and Z. The moment of inertia of the framework about an axis through X perpendicular to XYZ is

A. 12 Ma². B. 25 Ma². C. 27 Ma². D. 98 Ma². E. 144 Ma². 7.

2 m

3 m

hX

Y

Z

Page 24: AL Physics 1981-2004 MC

83 AL Physics/M.C./P.3

A particle is released from X and slips down a smooth curve to Y, at the edge of a table 2 m high, where it travels horizontally. It then leaves the table and travels freely under gravity until it hits the ground at Z, at a horizontal distance of 3 m from Y. The vertical distance, h, of X above Y is

A. 9/8 m. B. 4/3 m. C. 2 m. D. 3 m. E. 4 m. 8. A body is moving with simple harmonic

motion about point O. Which of the following graphs represents the variation of its displacement x from O, with its acceleration a?

A.

a

x

0

B.

a

x

0

C.

a

x

0

D.

a

x

0

E.

a

x

0

9.

F

W A man pulls with force F on a rope passing

over a pulley of radius r and moment of inertia I and raises a weight W at constant speed v. The kinetic energy of the system shown is

A. Wv²/2g + Iv²/2. B. Wv²/2g + Iv²/2r². C. Wv²/2 + Iv²/2. D. Wv²/2 + Iv²/2r². E. Wv²/2 + Ir²v²/2. 10. A sphere X of mass m, travelling with speed u,

makes a head-on collision with a similar sphere Y which is at rest. After the collision the velocities of X and Y are v1 and v2 respectively. Which one of the following is a possible pair of values for v1 and v2?

v1 v2 A. -u 2u B. u/4 3 u/4 C. 3 u/4 u/4 D. u/√2 u/√2 E. u/2 (u√3)/2 11. The capillary rise of water in a glass tube is 5

cm. When the same tube is dipped into a second liquid, of density three times the density of water, the capillary rise is 2.5 cm. The angle of contact is zero in each case. The ratio of the surface tension of the second liquid to that of water is

Page 25: AL Physics 1981-2004 MC

83 AL Physics/M.C./P.4

A. 1 : 6. B. 2 : 3. C. 3 : 2. D. 3 : 1. E. 6 : 1. 12.

X

Y

Z A pipe X of cross-sectional area 24 cm²

branches into two smaller pipes, Y of area 15 cm² and Z of area 6 cm². An incompressible liquid flows through the pipes and travels at a speed of 0.4 m/s in X and 0.6 m/s in Y. What is the speed of the fluid in Z?

A. 0.1 m/s B. 0.2 m/s C. 1.0 m/s D. 1.5 m/s E. 1.6 m/s 13.

object

lens screen

glassslab

A converging lens forms a sharp image on a

screen as shown. A glass slab of thickness t is inserted between the lens and the screen. In order for the image to be again sharply focussed, it is necessary to move the screen a small distance d

A. away from the lens; d decreases when t

increases. B. away from the lens; d increases when t

increases. C. towards the lens; d decreases when t

increases. D. towards the lens; d increases when t

increases. E. away from the lens; d is independent of t.

14.

i

A

CB When light is incident on the face AB of a

prism as shown, it may pass through the face AC, with total deviation d, or it may suffer total internal reflection at AC. Which of the graphs below best represents the variation of d with i? (t.i.r. represents total internal reflection)

A.

i

d

t.i.r.

90o0 B.

i

d

t.i.r.

90o0 C.

i

d

t.i.r.

90o0 D.

i

d

t.i.r.

90o0 E.

Page 26: AL Physics 1981-2004 MC

83 AL Physics/M.C./P.5

i

d

t.i.r.

90o

t.i.r.

0 15.

2 f

converging lens

screen

Two arrows are drawn as shown on a screen,

placed at a distance 2 f from a converging lens of focal length f. Which of the following diagrams correctly represents the image seen when the screen is viewed through the lens?

A.

B.

C.

D.

E.

16. An ambulance with its siren sounding a note of

constant frequency fs drives along a straight

road. Which of the graphs below best represents the variation of frequency f heard by a man standing close to the road as the ambulance drives past?

A.

time

f

fs

B.

time

f

fs

C.

time

f

fs

D.

time

f

fs

E.

time

f

fs

17. A beam of electrons crossing a vacuum tube

constitutes a current. Which of the following changes would double the magnitude of the current?

A. Doubling the potential difference across

the electron gun which produces the electrons.

B. Halving the area of cross-section of the beam.

C. Halving the length of the tube. D. Doubling the electron speed. E. Doubling the electron kinetic energy.

Page 27: AL Physics 1981-2004 MC

83 AL Physics/M.C./P.6

18. Tow parallel wires repel each other with a

force F when the same current passes though them. If the current is doubled and the distance between the wires is also doubled, the force of repulsion will then be

A. F/4. B. F/2. C. F. D. 2F. E. 4F. 19. A meter has a scale marked -20 µA to +100 µA

and its resistance is 24 Ω. In order to convert it to an ammeter with a positive full-scale deflection of +1.0 mA, it is necessary to add a resistance R

A. in parallel, R = 2.4 Ω B. in series, R = 2.7 Ω C. in parallel, R = 2.7 Ω D. in series, R = 3.3 Ω E. in parallel, R = 3.3 Ω 20. Two parallel metal plates are placed

horizontally with a separation of 0.05 m. A p.d. of 2.0 kV is connected across the plates. An oil drop with a charge of -1.6 × 10-19 C is observed to remain at rest between the plates. (Assume g = 10 m/s².) The mass of the drop is

A. 4.0 × 10-25 kg. B. 1.6 × 10-18 kg. C. 1.6 × 10-16 kg. D. 6.4 × 10-16 kg. E. 6.4 × 10-14 kg. 21. A rectangular coil PQRS is driven with

constant velocity in a direction perpendicular to a uniform magnetic field as shown.

RQ

SP Which of the following statements is correct? A. An induced current it flowing in the coil in

the clockwise direction. B. An induced current is flowing in the coil in

the anticlockwise direction.

C. An electromagnetic force acts on the side PQ in a direction opposing its motion.

D. There is no induced current flowing in the coil.

E. The magnitude of the magnetic flux through the coil changes with time.

22.

X Y

8 Ω

In the above circuit, X and Y are identical cells,

of e.m.f. 10 V and internal resistance 4 Ω. What is the current passing through the 8 Ω resistor?

A. zero B. 5/6 A C. 1 A D. 5/3 A E. 2 A 23.

G

R 2 V

40 Ω Ω50

0.4 A0.4 A

In the circuit shown, no current flows through

the galvanometer G. If the internal resistance of the cell is negligible, what is the value of R?

A. 0.89 Ω B. 1.3 Ω C. 4.4 Ω D. 5.0 Ω E. 5.3 Ω 24. An alternating current I = I0 sin ωt flows in a

coil of inductance L and resistance R. A back e.m.f. E is induced in the coil, where E is equal to

A. IωL.

B. LdI

dt.

C. I(ωL - R).

Page 28: AL Physics 1981-2004 MC

83 AL Physics/M.C./P.7

D. LdI

dtIR− .

E. LdI

dtIR+ .

25. Which of the following graphs best shows the

change of current I with time t when an electric motor connected to a d.c. source is switched on? (The magnetic field is supplied by a permanent magnet.)

A.

t

I

0 B.

t

I

0

C.

t

I

0

D.

t

I

0 E.

t

I

0 26.

L

Xdynamo

A dynamo is connected to an a.c. ammeter X

and a large inductor L as shown. The ammeter reading is I. If the resistance of the circuit is negligible, and the rate of rotation of the dynamo is doubled, the ammeter reads

A. I/2. B. I. C. I√2. D. 2 I. E. 4 I. 27. An alternating voltage V is applied to an LCR

series circuit, where the inductive reactance XL, the capacitive reactance XC and the resistance R satisfy the relation

XL = 2 XC = 2 R.

What is the phase difference between V and the

potential difference VC across the capacitor? A. π/4 B. π/2 C. 3 π/4 D. π E. 5 π/4 28. A radioactive source of gamma rays has a half-

life of 2 days. A Geiger counter placed 3 m from the source initially has a count-rate of 1440 per minute. After 6 days the counter is moved back to a distance of 9 m from the source, and its count-rate, in counts per minute, is then

A. 20. B. 60. C. 180. D. 320. E. 360. 29.

3 V

6 Va.c.

Page 29: AL Physics 1981-2004 MC

83 AL Physics/M.C./P.8

For the circuit shown above, which of the

graphs shown below best represents the variation of current I with time t?

A.

t

I

0 B.

t

I

0 C.

t

I

0 D.

t

I

0

E.

t

I

0

30.

L1 L2

E1 E2 The diagram shows a transistor circuit with two

similar light bulbs L1 and L2. The bulb L2 lights up brightly, but L1 does not glow at all. Which of the following could be a possible reason for this?

A. The filament of L1 is burnt out.

B. The cell E1 should be connected the other way round.

C. The cell E2 should be connected the other way round.

D. The collector current is very much less than the emitter current.

E. The base current is very much less than the emitter current.

31. A wire, of force constant k, has an extension e

when it is supporting a weight W. If the elastic limit is not exceeded, the energy stored in the wire is equal to

(1) ½ × stress × strain. (2) ½ × W × e. (3) ½ × k × e². A. (1), (2) and (3) B. (1) and (2) only C. (2) and (3) only D. (1) only E. (3) only 32. A cylinder containing air is fitted with an

airtight, frictionless piston maintained at a constant temperature. The piston is moved very slowly inwards, until the volume of the cylinder has halved. Which of the following quantities has doubled?

(1) The average speed of the gas molecules in

the cylinder. (2) The average momentum of the gas

molecules in the cylinder. (3) The average force exerted by the gas

molecules on the piston. A. (1), (2) and (3) B. (1) and (2) only C. (2) and (3) only D. (1) only E. (3) only 33. Heat energy must be supplied to a pan of water

at 100 ºC to keep it boiling. This energy replaces the heat energy lost to the surroundings, and also supplies energy to

(1) increase the average kinetic energy of the

molecules. (2) increase the average potential energy of

the molecules. (3) allow the water vapour to expand against

atmospheric pressure.

Page 30: AL Physics 1981-2004 MC

83 AL Physics/M.C./P.9

A. (1), (2) and (3) B. (1) and (2) only C. (2) and (3) only D. (1) only E. (3) only 34. Theoretically, the zero on the absolute

temperature scale is the point where a fixed mass of an ideal gas has

(1) zero pressure. (2) zero volume. (3) infinite density. A. (1), (2) and (3) B. (1) and (2) only C. (2) and (3) only D. (1) only E. (3) only 35. Water has the saturated vapour pressure p at

100 ºC. When a beaker of pure water was heated, it did not start to boil until the temperature was 100.5 ºC. The reason for this could have been

(1) the air above the water was saturated with

water vapour. (2) the external pressure was more than p. (3) the walls of the vessel were so smooth that

bubble formation could not start. A. (1), (2) and (3) B. (1) and (2) only C. (2) and (3) only D. (1) only E. (3) only 36.

XA B

C

S

The above arrangement is used in a version of

Young’s experiment. X is a monochromatic light source. A, B and C are narrow parallel slits. Bright fringes are observed on the screen S. What would happen if the whole set-up were placed in water instead of air?

(1) The central bright fringe will become dark.

(2) The fringes on both sides become multi-coloured.

(3) The separation between the fringes decreases.

A. (1), (2) and (3) B. (1) and (2) only C. (2) and (3) only D. (1) only E. (3) only 37.

M

M

2

1

v 2

v 1i

Light travels in media M1 and M2 with speeds

v1 and v2 respectively. When light travelling from medium M1 strikes a boundary between the two media with angle of incidence i, it suffers total internal reflection. This indicates that

(1) v1 is less than v2. (2) sin i is less than v1/v2. (3) sin i is greater than v2/v1. A. (1), (2) and (3) B. (1) and (2) only C. (2) and (3) only D. (1) only E. (3) only 38. When monochromatic light is incident

normally on a wedge-shaped thin film, an interference pattern may be seen by reflection. Which of the following changes would increase the number of fringes per unit length as seen by an observer?

(1) increasing the wavelength of the light (2) increasing the angle of the wedge (3) increasing the refractive index of the film

material A. (1), (2) and (3) B. (1) and (2) only C. (2) and (3) only D. (1) only E. (3) only 39. In a standing wave in a closed pipe,

Page 31: AL Physics 1981-2004 MC

83 AL Physics/M.C./P.10

(1) some air molecules do not vibrate. (2) no energy comes out of the pipe. (3) in the fundamental mode of vibration air

molecules in the middle of the pipe oscillate through a larger amplitude than molecules at any other part of the pipe.

A. (1), (2) and (3) B. (1) and (2) only C. (2) and (3) only D. (1) only E. (3) only 40. Which of the following statements about the

coulomb is/are correct? (1) When one coulomb of charge flows across

a potential difference of one volt, one joule of energy is released.

(2) The force exerted on a charge of 1 coulomb in an electrostatic field of 1 volt/metre is 1 newton.

(3) 1 coulomb is the charge on 1 mole of electrons.

A. (1), (2) and (3) B. (1) and (2) only C. (2) and (3) only D. (1) only E. (3) only 41.

+ _

V

coil

L, RS

A steady potential difference V is applied

across a coil of inductance L and resistance R, connected to a switch S, as shown. After closing the switch S, the current increases and takes time t to reach half its steady maximum value. The time t can be shortened by

(1) inserting a piece of soft iron inside the

solenoid. (2) applying a higher potential difference to

the circuit. (3) adding a resistor in series with the coil. A. (1), (2) and (3) B. (1) and (2) only C. (2) and (3) only D. (1) only E. (3) only

42.

G

V

QPR

E

T

The potentiometer circuit shown is used to find

the internal resistance of the cell E. At balance, the galvanometer pointer does not deflect, and NO current flows through

(1) the potentiometer wire PQ. (2) the resistor R. (3) the galvanometer G. A. (1), (2) and (3) B. (1) and (2) only C. (2) and (3) only D. (1) only E. (3) only 43. A 10 V, 5 W light bulb in a set of Christmas

tree lights (which consists of 20 bulbs in series) burns out and Jimmy goes to buy a replacement. When he gets back, he finds that although the new bulb is marked 5 W, the light it gives is very dim, although the other bulbs light up brightly. Which of the following is a possible reason for this?

(1) The supply voltage has dropped below

200 V. (2) The current through the circuit is very

much less than 0.5 A. (3) The bulb is designed to work at a voltage

higher than 10 V. A. (1), (2) and (3) B. (1) and (2) only C. (2) and (3) only D. (1) only E. (3) only 44. A signal is applied to the Y plates of an

oscilloscope, with the time base running. the trace on the screen is as shown in Figure (i).

Page 32: AL Physics 1981-2004 MC

83 AL Physics/M.C./P.11

Figure (i) Figure (ii) Which of the following adjustments would

change the trace to that shown in Figure (ii)? (1) Doubling the Y sensitivity. (2) Doubling the sweep speed of the time

base. (3) Doubling the frequency of the signal. A. (1), (2) and (3) B. (1) and (2) only C. (2) and (3) only D. (1) only E. (3) only 45. The resonant frequency of a series LCR circuit

is f0. When an alternating signal V of frequency f is applied to this circuit, a current I flows in it. Which of the following is/are correct?

(1) There is no phase difference between I and

V when f = f0. (2) I leads V when f is very much less than f0. (3) V leads I when f is very much greater than

f0. A. (1), (2) and (3) B. (1) and (2) only C. (2) and (3) only D. (1) only E. (3) only 46.

C

R K

d.c.

A capacitor and a resistor are connected in

series to a d.c. voltage supply as shown. At time t = 0, the key K is closed. Which of the following statements is/are true?

(1) At any time t the sum of the potential

differences across C and R is a constant. (2) The capacitor is almost fully charged after

a time t = 5 CR.

(3) The final charge on the capacitor does not depend on the position of R in the circuit.

A. (1), (2) and (3) B. (1) and (2) only C. (2) and (3) only D. (1) only E. (3) only 47. The allowed energy levels for an atom are En =

-W/n², where W is a constant and n is any positive integer. The frequency of a photon corresponding to the energy transition from n = m + 1 to n = m

(1) is directly proportional to W. (2) decreases as m increases. (3) increases as the temperature increases. A. (1), (2) and (3) B. (1) and (2) only C. (2) and (3) only D. (1) only E. (3) only 48.

E

C

The above figure shows a photoelectric cell. E

is the emitter and C is the collector. The stopping potential for photoelectrons depends on

(1) the nature of E. (2) the distance between E and C. (3) the surface area of E. A. (1), (2) and (3) B. (1) and (2) only C. (2) and (3) only D. (1) only E. (3) only 49. The binding energy per nucleon varies with

mass number as shown below:

mass

number

bindingenergy /nucleon

P

Q

R

Page 33: AL Physics 1981-2004 MC

83 AL Physics/M.C./P.12

If nuclear energy were to be generated by the fusion of the nuclei of an element X, which of the points P, Q and R would represent possible positions of X on the graph?

(1) P (2) Q (3) R A. (1), (2) and (3) B. (1) and (2) only C. (2) and (3) only D. (1) only E. (3) only 50. When alpha particles are fired in a beam at a

thin sheet of gold, they are scattered though a

wide range of angles, from zero to 180º from the incident beam. This experiment shows that they gold atom has

(1) electrons in different orbits. (2) a volume consisting mainly of empty

space. (3) a small, dense, charged nucleus. A. (1), (2) and (3) B. (1) and (2) only C. (2) and (3) only D. (1) only E. (3) only

- End of Paper -

Page 34: AL Physics 1981-2004 MC

83 AL Physics/M.C./P.13

Question No. Key 1. B 2. A 3. B 4. D 5. B 6. B 7. A 8. C 9. B 10. B 11. C 12. A 13. B 14. A 15. D 16. D 17. D 18. D 19. C 20. D 21. D 22. C 23. C 24. B 25. E

Question No. Key 26. B 27. C 28. A 29. C 30. E 31. C 32. E 33. C 34. A 35. C 36. E 37. D 38. C 39. D 40. B 41. E 42. E 43. E 44. E 45. A 46. A 47. B 48. D 49. B 50. C

Page 35: AL Physics 1981-2004 MC

84 AL Physics/M.C./P.1

1984 Hong Kong Advanced Level Examination AL Physics

Multiple Choice Question

1.

P

A horizontal force P is applied to a wooden

block at rest on a rough horizontal table. P is increased uniformly from zero. Which of the following graphs best describes the variation of the frictional force F acting on the block with P?

A.

P

F

0 B.

P

F

0 C.

P

F

0 D.

P

F

0 E.

P

F

0 2. An object is thrown horizontally from the top

of a cliff at a speed 20 m/s. If the gravitational

acceleration g = 10 m/s², what will be the speed of the object after 3 s?

A. 20 m/s B. 25 m/s C. 30 m/s D. 36 m/s E. 50 m/s 3. In an elastic collision between two bodies, and

in the absence of external forces, A. kinetic energy and momentum are both

conserved. B. kinetic energy is not conserved but

momentum is conserved. C. kinetic energy is conserved but momentum

is not conserved. D. neither kinetic energy nor momentum is

conserved. E. kinetic energy is not conserved and the

momentum changes direction. 4.

P

The diagram represents the rear view of a

motor car moving, away from an observer on a level road at a constant speed around a bend of which the centre of curvature is at P. Which of the arrows best represents the direction of the resultant of the forces exerted by the road on the car?

A. B. C.

D.

Page 36: AL Physics 1981-2004 MC

84 AL Physics/M.C./P.2

E.

5. A circular loop, of mass M and radius r, is

hung over a peg, and makes small oscillations in its own plane of period 2 s. If the moment of inertia of the loop about the axis of oscillation is 2 Mr² and the gravitational acceleration g = 10 m/s², then the value of r is

A. 0.51 m. B. 0.68 m. C. 0.76 m. D. 1.01 m. E. 2.03 m. 6. The tip of each prong of a tuning fork, emitting

a sound wave of frequency 250 Hz, has an amplitude of 0.5 mm. What is the speed of each tip when its displacement is 0.4 mm?

A. 0.47 m/s B. 0.79 m/s C. 7.85 m/s D. 740 m/s E. 1.23 m/s 7. A system oscillates under the influence of an

external periodic driving force. Which of the following statements is INCORRECT?

A. In steady state the system vibrates at the

frequency of the driving force. B. The amplitude of vibration becomes very

large when the frequency of the driving force is close to the natural frequency of vibration of the system.

C. The amplitude of vibration remains finite if damping forces are present.

D. A resonance the displacement of the system is in phase with the driving force.

E. At resonance the power transferred from the driving force to the system is a maximum.

8. The velocity v of a small steel ball falling in a

viscous liquid inside a long vertical tube varies with time t, as follows:

t

v

0 Which of the following graphs best represents

the variation of the viscous force F acting on the body?

A.

t

F

0 B.

t

F

0 C.

t

F

0 D.

t

F

0 E.

t

F

0 9.

Page 37: AL Physics 1981-2004 MC

84 AL Physics/M.C./P.3

atmosphericpressure P

air

water

X

Water, of density ρ, rises up a capillary tube of

radius r to a height h. If the angle of contact is zero and the surface tension is γ , then the pressure at X is

A. P - ρgh. B. P. C. P + ρgh. D. P + ρgh - 2γ/r. E. P + ρgh + 2γ/r. 10. Equal and steadily increasing forces are

applied to each of the following three wires: a carbon fibre, a steel wire and a brass wire. Numerical values for the Young modulus E, the ultimate tensile stress S, and the area of cross-section A of the wires are:

E/1010 N/m² S/ 108 N/m² A/10-6 m² carbon fibre

40 17 0.10

steel wire

21 10 0.30

brass wire

12 40 0.05

Which of the wires is the stiffest and which of

the wires will break first? Choose the combination satisfying both criteria.

Stiffest wire Wire which will break first A. carbon fibre steel wire B. steel wire carbon wire C. brass wire steel wire D. carbon fibre carbon fibre E. brass wire brass wire 11. A fixed mass of gas at s.t.p. occupies a volume

of 1 m³. The gas is heated and allowed to expand to a final volume of 2 m³ with its pressure doubled. The average kinetic energy of the gas molecules is

A. reduced to one quarter of its initial value. B. halved. C. unchanged.

D. doubled. E. increased four times. 12. An astronomical telescope has an objective of

focal length 40 cm, and an eyepiece of focal length 2 cm. It is used to look at a distant object when its lenses are set 42 cm apart. the final image seen is

A. upright, virtual and at infinity. B. inverted, virtual and at infinity. C. upright, real and at infinity. D. inverted, real and at infinity. E. upright, virtual and at the least distance of

distinct vision. 13.

IIIV

IVIII

L

X YP

Q

The diagram shows two incoming parallel rays

of light P and Q which pass through a thin converging lens L. The ray XY after passing through the lens will pass through the point

A. I. B. II. C. III. D. IV. E. V. 14.

Fig. 1

Fig. 2

Fig. 3

light source observer

Three polaroid sheets are arranged in the three

different ways as shown in Figures 1, 2 and 3 and an observer looks towards the light source as shown in the diagrams. The bold lines indicate the directions of the transmission axes

Page 38: AL Physics 1981-2004 MC

84 AL Physics/M.C./P.4

in each case. Which of the following statements correctly describes the effect of the combination?

Figure 1 Figure 2 Figure 3 A. opaque opaque opaque B. opaque opaque transparent C. opaque transparent opaque D. opaque transparent transparent E. transparent opaque transparent 15. Newton’s rings produced when a biconvex lens

rests on a plane glass plate are observed using a travelling microscope. If the biconvex lens is very slowly moved vertically upwards from the lower glass plate, which of the following would be observed?

A. The central spot remains dark all the time. B. The rings disappear immediately. C. The rings move towards the centre. D. The rings move out from the centre. E. The rings are no longer concentric. 16. Young’s slits are used to produce interference

fringes with light of wavelength 600 nm. A thin sheet of mica of refractive index 1.6 is placed in front of one of the slits and the centre of the fringe-system is displaced through 8 fringe widths. The thickness of the mica is

A. 120 nm. B. 3 000 nm. C. 4 000 nm. D. 7 700 nm. E. 8 000 nm. 17. In the tuning of a violin string, a pitch pipe of

frequency 427 Hz was blown at the same time as the string was plucked and 5 beats were heard every 2 seconds. Then the violin string was slightly tightened with a fine-adjustment screw. When the pitch pipe and the string were sounded together again no beats were heard. The initial frequency of the note produced by the string before any adjustment was made must have been

A. 422.0 Hz. B. 424.5 Hz. C. 429.5 Hz. D. 432.0 Hz. E. 437.0 Hz.

18. A sound reproduction system produces a sound level of 95 dB above threshold. Assuming the threshold of hearing to be 10-12 W/m², 95 dB corresponds to an intensity of

A. 5.6 × 10-8 W/m². B. 0.00316 W/m². C. 0.0316 W/m². D. 1.12 W/m². E. 6.3 × 104 W/m². 19.

Ω

Ω

Ω

Ω

Ω

99.9 k

9.9 k100

0.1001

1.01

0 - 1 mA3

2

1

1

2

3

Q

P

inputterminals

The diagram shows the circuit of a multimeter

with ranges 0 - 0.1 A, 0 - 1 A, 0 - 10 V and 0 - 100 V. What must be the positions of the range switches P and Q if the current to be measured is about 0.5 A?

P Q A. 1 1 B. 1 2 C. 1 3 D. 2 1 E. 2 2 20.

RB

P QS

X Y

G

The potentiometer circuit above was set up to

determine the e.m.f. of a thermocouple XY. PQ is a uniform resistance wire. As the sliding contact S is moved along PQ towards Q, the galvanometer deflection decreases continuously in the same direction, but does not become zero. What is the reason?

Page 39: AL Physics 1981-2004 MC

84 AL Physics/M.C./P.5

A. The e.m.f. of the thermocouple is too small.

B. The resistance R is too small. C. The resistance R is too large. D. The e.m.f. of the battery B is too large. E. The galvanometer resistance is too small. 21.

SYX

12 V 3 Ω

Ω5

V

A high resistance voltmeter is connected

between points X and Y in the circuit shown. When the switch S is open, the reading on the voltmeter is approximately

A. 0 V. B. 1.5 V. C. 2.5 V. D. 4.0 V. E. 12 V. 22. 1 000 turns of wire are wound on a solenoid of

length 0.30 m and area of cross-section 3.2 × 10-4 m². When the solenoid carries a current of 1.5 A, the magnetic flux through the solenoid is

A. 6.0 × 10-7 Wb. B. 2.0 × 10-6 Wb. C. 5.7 × 10-4 Wb. D. 2.0 × 10-3 Wb. E. 6.3 × 10-3 Wb. (µ0 = 1.26 × 10-6 H/m) 23. A sinusoidal voltage is generated by an a.c.

dynamo. If the speed of rotation of the dynamo is doubled, what will happen to the frequency and the amplitude of the voltage formed?

frequency amplitude A. no change no change B. doubled no change C. no change doubled D. doubled halved E. doubled doubled

24.

Q

P

A sheet of dielectric occupies part of the space

between the parallel plates of an evacuated capacitor. P is a point in the dielectric and Q is a point in the vacuum between the plates. Neglecting edge effects, the electric filed strength at P when the capacitor is charged is

A. zero. B. the same as that at Q. C. equal in magnitude but opposite in

direction to that at Q. D. less than that at Q. E. greater than that at Q. 25. A transistor is used for current amplification in

the common emitter configuration. The measured currents through the emitter, collector and base of the transistor are ie, ic and ib respectively. What are the possible values of ic/ie and ie/ib as obtained from the measurement of the various currents?

ic/ie ie/ib A. 0.98 50 B. 1 1 C. 0.5 50 D. 1.02 50 E. 50 0.98 26. A power station supplies electrical power to a

user. The power generated by the station is 1200 kW and is transmitted at 132 000 V. If the resistance of the lines connecting the power station to the user is 550 Ω , the power available to the user is about

A. 700 kW. B. 1 155 kW. C. 1 195 kW. D. 1 200 kW. E. 31 680 kW. 27.

Page 40: AL Physics 1981-2004 MC

84 AL Physics/M.C./P.6

N

A large aluminium disc mounted on a

horizontal axle is spun in the clockwise direction between the poles of a powerful horseshoe magnet. Which of the following diagrams shows how the eddy currents flow in the disc?

A.

B.

C.

D.

E.

28.

10 cm 10 cm

Figure (a) Figure (b) A step-down transformer of turns-ratio 20 : 1

has its primary winding connected to the mains (50 Hz) and its secondary winding connected across the Y plates of an oscilloscope, with the time-base freely running. The trace observed on the screen is as shown in Figure (a). the transformer is now disconnected from the oscilloscope. With the same time base setting, a signal of unknown frequency f is connected across the Y plates, and the trace is as shown in Figure (b). The value of f is

A. 6.25 Hz. B. 20.0 Hz. C. 125 Hz. D. 150 Hz. E. 2 500 Hz. 29.

RL

V

VRVL

(3)

(4)

(2)

(1)

(6)(5)

Figure (a) Figure (b) In Figure (a), an alternating voltage V is

applied across an inductor L and a resistor R connected in series. VL represents the instantaneous voltage across the inductor, and VR represents the voltage across the resistor. In the phasor diagram in Figure (b), the phasor representing VR points in the direction (1). In which of the directions shown in Figure (b) will the phasors representing VL and V point?

VL V A. (1) (1) B. (2) (3) C. (3) (2) D. (5) (6) E. (6) (5)

Page 41: AL Physics 1981-2004 MC

84 AL Physics/M.C./P.7

30.

load resistance

RCinput

The above figure shows a half-wave rectifier

with a smoothing circuit. The time constant of C and R should be

A. small compared with the time of one cycle. B. equal to half the time of one cycle. C. equal to the time of one cycle. D. equal to twice the time of one cycle. E. large compared with the time of one cycle. 31. The ionisation energy of an atom in its ground

state is A. the energy required to separate all the

electrons from the remainder of the atom. B. the maximum energy required to separate

one electron from the remainder of the atom.

C. the minimum energy required to separate one electron from the remainder of the atom.

D. the minimum energy required to add one electron to the atom.

E. the minimum energy required for an electron to change its state while at the same time remaining attached to the nucleus.

32. For the photoelectric effect, which of the

following is the correct relationship between the energy E of a photon, the work function w of the surface which it strikes, and the maximum kinetic energy K of the emitted photoelectron?

A. E = w + K B. E = w - K C. E = K - w D. K = 2(w + E) E. w = ½(K + E) 33. An alpha particle collides with a stationary

helium nucleus ( 24 He ) in a cloud chamber.

Which of the following diagrams represents the most probable set of tracks?

A.

α

αHe2

4

B.

α

α

He24

C.

α

α

He24

D.

α

α He24

E.

α

α He24

34. The minimum wavelength of the X-rays

emitted from a hot cathode X-ray tube is controlled by

A. the cathode temperature. B. the nature of the target. C. the anode-cathode voltage. D. the size of the target. E. the length of the X-ray tube. 35. 3He is an isotope of helium. If the masses of a

proton, a neutron and a 3He nucleus are, respectively, 938.3 MeV/c², 939.6 MeV/c² and 2808.5 MeV/c², the binding energy of 3He is

Page 42: AL Physics 1981-2004 MC

84 AL Physics/M.C./P.8

A. zero. B. 7.7 MeV. C. 9.0 MeV. D. 930.6 MeV. E. 947.3 MeV. 36. In a controlled thermal fission reactor, the

function of the moderators is to reduce A. the speed of the neutrons released on

fission. B. the rate of production of the neutrons. C. the energy generated in the nuclear

reactor. D. the amount of radioactive radiations

produced in the nuclear reactor. E. the rate of disintegration of the 235U

nucleus. 37.

screen

R

d

p

A beam of electrons, travelling with uniform

momentum p, is incident on a narrow slit of width d. A fluorescent screen is placed at a distance R from the opening. What is the width of the central maximum of the diffraction pattern observed on the screen?

(h = Planck constant) A. d/2 B. 2d² / R C. 2h / p D. 2hd / Rp E. 2hR /dp 38. Two bodies are allowed to fall freely from the

same location, one of which is released a short time before the other. If the air resistance can be neglected, while they are falling,

(1) the two bodies have the same acceleration. (2) their speeds always differ by the same

amount. (3) their distance of separation is always the

same. A. (1), (2) and (3)

B. (1) and (2) only C. (2) and (3) only D. (1) only E. (3) only 39. An ice-skater spinning on one foot brings her

arms as close as possible to her body. In doing so which of the following quantities must have increased?

(1) The angular velocity of rotation of the

skater. (2) The moment of inertia of the skater. (3) The angular momentum of the skater. A. (1), (2) and (3) B. (1) and (2) only C. (2) and (3) only D. (1) only E. (3) only 40. Taking the Earth to be a perfect sphere of

uniform density rotating about its polar axis, which of the following statements concerning the observed acceleration due gravity, g, at the surface of the Earth is NOT true?

(1) g at the equator is smaller than that at the

poles. (2) If the rate of rotation of the Earth slows

down, g at the equator increases. (3) If the radius of the Earth increases with its

density remaining unchanged, g at the poles decreases.

A. (1), (2) and (3) B. (1) and (2) only C. (2) and (3) only D. (1) only E. (3) only 41.

r

V

0 a b

The figure above shows the variation of the

potential energy V between two atoms with the distance r separating them. Which of the following statements is/are correct?

Page 43: AL Physics 1981-2004 MC

84 AL Physics/M.C./P.9

(1) The atoms will experience a repulsive

force when r < a. (2) The atoms will experience an attractive

force when r > b. (3) The atoms can be in equilibrium when r =

a. A. (1), (2) and (3) B. (1) and (2) only C. (2) and (3) only D. (1) only E. (3) only 42.

The above figure shows the stream lines for air

flowing past the wing of an aeroplane. Which of the following is/are correct?

(1) The pressure above the wing is greater

than that below the wing. (2) The speed of the air flow above the wing

is greater than that below the wing. (3) The pressure difference between locations

above and below the wing increases when the density of air increases.

(Assume the same speed of air flow.) A. (1), (2) and (3) B. (1) and (2) only C. (2) and (3) only D. (1) only E. (3) only 43. A candidate places an electrical heater in the

middle of a lump of ice and measures the time it takes to just melt all of the ice. The power of the electric heater and the initial mass of ice were measured and found to be 12 W and 0.10 kg, respectively. From these measurements, he calculates the specific latent heat of fusion of ice, but finds that his answer is lower than expected. This result is possibly due to the fact that

(1) energy has been lost to the surroundings. (2) the mass of ice is actually more than the

measured value.

(3) the power of the heater is actually more than the measured value.

A. (1), (2) and (3) B. (1) and (2) only C. (2) and (3) only D. (1) only E. (3) only 44. One mole of an ideal gas of volume V1 is at an

initial pressure P1 and temperature T1. If the gas undergoes an isothermal expansion, so that its volume increases to V2.

(1) there is no heat exchange between the gas

and the surroundings. (2) the external work done is equal to P1(V2 -

V1). (3) the internal energy of the gas remains

unchanged. A. (1), (2) and (3) B. (1) and (2) only C. (2) and (3) only D. (1) only E. (3) only 45.

N

vc

The above graph shows the distribution of

speeds (v) of the molecules in a constant mass of gas. Which of the following statements is/are correct?

(1) The value v = c at which the peak of the

curve occurs increases when the temperature rises.

(2) The peak of the curve rises when the temperature rises.

(3) Provided that the temperature does not change, the kinetic energy of each molecules is fixed.

A. (1), (2) and (3) B. (1) and (2) only C. (2) and (3) only D. (1) only E. (3) only

Page 44: AL Physics 1981-2004 MC

84 AL Physics/M.C./P.10

46. Light travels between two media X and Y. If the refractive index of X is greater than the refractive index of Y, which of the following is/are possible ray diagram(s)?

(1)

YX

(2)

YX

(3)

YX

A. (1), (2) and (3) B. (1) and (2) only C. (2) and (3) only D. (1) only E. (3) only 47. Which of the following statements about wave

motion is correct? (1) Diffraction cannot be exhibited by

longitudinal waves. (2) Refraction prevents the formation of

shadows with perfectly sharp edges. (3) Reflection is sometimes accompanied by a

phase change of π radians. A. (1), (2) and (3) B. (1) and (2) only C. (2) and (3) only D. (1) only E. (3) only 48.

ZXY

The diagram shows a pattern of electric field

lines in which X, Y and Z are points marked on one of the field lines. It would be correct to say that

(1) X is at a higher potential than Z. (2) a negative charge placed at Z would

accelerate to the left along the tangent to the field line at Z.

(3) the force exerted on a charge at Y would be greater than if the charge were placed at X.

A. (1), (2) and (3) B. (1) and (2) only C. (2) and (3) only D. (1) only E. (3) only 49. A capacitor (of negligible resistance) and a

solenoid (whose resistance is not negligible) are connected in series with an a.c. supply. The resonant frequency of the circuit can be increased by

(1) replacing the solenoid with one of lower

resistance, but the same inductance. (2) replacing the capacitor with one of greater

capacitance. (3) replacing the solenoid with one of lower

inductance, but the same resistance. A. (1), (2) and (3) B. (1) and (2) only C. (2) and (3) only D. (1) only E. (3) only 50. The time base of a cathode ray oscilloscope (1) provides a sawtooth voltages of variable

frequency across the X-plates. (2) enables the electron beam to be deflected

across the screen at different speeds. (3) varies the number of electrons reaching

the screen and hence controls the brightness.

Page 45: AL Physics 1981-2004 MC

84 AL Physics/M.C./P.11

A. (1), (2) and (3) B. (1) and (2) only C. (2) and (3) only

D. (1) only E. (3) only

- End of Paper -

Question No. Key 1. D 2. D 3. B 4. D 5. A 6. A 7. D 8. A 9. A 10. D 11. E 12. B 13. B 14. B 15. C 16. E 17. B 18. B 19. B 20. C 21. E 22. B 23. E 24. D 25. A

Question No. Key 26. B 27. D 28. C 29. C 30. E 31. C 32. A 33. D 34. C 35. B 36. A 37. E 38. B 39. D 40. E 41. B 42. C 43. E 44. E 45. D 46. D 47. E 48. A 49. E 50. B

Page 46: AL Physics 1981-2004 MC

85 AL Physics/M.C./P.1

1985 Hong Kong Advanced Level Examination AL Physics

Multiple Choice Question

1.

Q

PA

B

Two bodies P and Q are connected by a light

string A. The weights of P and Q are 4 N and 10 N respectively. P is connected to the roof by another light string B. If string B is cut and the bodies allowed to fall, the net force acting on P during free fall is

A. 0 N. B. 4 N. C. 6 N. D. 10 N. E. 14 N. 2. A lift of mass M carries a man of mass m.

When the lift is being hauled upward by a rope, the lift rises with an acceleration a. The reaction between the man and the floor of the lift is

A. mg. B. m (g - a). C. m (g + a). D. m (g + a) - Ma. E. m (g + a) + Ma. 3. A motorcyclist going round a corner on a level

road leans over at an angle to the horizontal. The reason for this is

A. to allow his weight to exert a torque about

the contact point on the ground. B. to increase the frictional force between the

motorcycle and the road. C. to lower his centre of mass. D. to provide the centripetal force.

E. to reduce the radius of curvature of his path.

4. Which of the following statements is true of the

acceleration of a particle oscillating with S.H.M.?

A. It is always in the opposite sense to the

velocity of the particle. B. It varies linearly with the frequency of

oscillation. C. It has its smallest magnitude when the

speed of the particle is greatest. D. It decreases as the potential energy

increases. E. Its magnitude is a minimum when the

displacement of the particle is a maximum. 5.

F = mg

R

A string wraps around a uniform cylinder of

radius R and mass m. A constant tension F = mg is maintained in the string causing the cylinder to rotate about its cylindrical axis. Given that the moment of inertia of the cylinder about its axis is mR²/2, the angular acceleration of the cylinder is

A. 0. B. gR. C. g/(2R). D. g/R. E. 2 g/R. 6. If v1 is the minimum speed for a projectile to

escape from the earth and v2 is the orbital speed of a satellite circling close to the earth, then v1/v2 =

Page 47: AL Physics 1981-2004 MC

85 AL Physics/M.C./P.2

A. 1/√2. B. 1/2. C. 1. D. √2. E. 2. 7. One end of a capillary tube of length 10 cm is

dipped into water. The water level rises by 5 cm inside the tube. if the capillary tube is now taken out and reinserted into the water so that only 4 cm of the tube is above the water surface,

A. the water inside the tube will not rise. B. the water inside the tube will rise to the 2

cm level. C. the water inside the tube will rise to the 4

cm level. D. the water will flow out of the tube

continuously. E. the water will shoot up to a height of 5 cm. 8. A liquid of density 1.00 × 103 kg/m³ flows

along a horizontal pipe whose cross-sectional area changes from 50 × 10-6 m² to 25 × 10-6 m². Manometers (using the same liquid) are attached to the two sections of the pipe and the vertical difference between the levels of liquid in them is 50 mm. The rate of flow of the liquid mass in the pipe must be

A. 0.22 × 10-2 kg/s. B. 1.44 × 10-2 kg/s. C. 2.50 × 10-2 kg/s. D. 2.89 × 10-2 kg/s. E. 5.01 × 10-2 kg/s. (g may be taken to be 10 m/s².) 9. In a bromine diffusion experiment, liquid

bromine is introduced into the bottom of a glass column containing air. Diffusion of bromine occurs and the ‘half-brown’ level reaches a distance d from the bottom of the column after time t. If the r.m.s. speed of bromine molecules is c, then the mean free path of the molecules is

A. ct. B. d² / (ct). C. c² t² / d.

D. √(cdt). E. d / √ (ct). 10.

P

T

P

T

A BS

1

1

2

2

A and B are two identical containers connected by a tap S initially closed. A contains an ideal gas at a pressure P1 and a temperature T1. B contains the same gas at a pressure P2 and a temperature T2. The tap S is then opened. If the temperatures of the containers A and B remain constant at T1 and T2 respectively, the final pressure of the gas mixture will be

A. (P1 + P2)/2. B. (P1 + P2). C. (P1T1 + P2T2) / (T1 + T2). D. (P1T2 + P2T1) / (T1 + T2).

E. ( )( )PT

PT

T T1

1

2

21 2+ + .

11. A vessel of volume 1 × 10-3 m³ contains 0.72 g

of an ideal gas at a pressure of 1 × 105 Pa. The r.m.s. velocity of the gas molecules is

A. 20 m/s. B. 110 m/s. C. 340 m/s. D. 650 m/s. E. 3400 m/s. 12.

0 20 40 60 80 10020

40

60

80

100

120

temp / oC

length/mmQ

P

Page 48: AL Physics 1981-2004 MC

85 AL Physics/M.C./P.3

The graph shows the variations of the lengths of the liquid columns of two liquid-in-glass thermometers with the temperature being measured on the ideal gas scale. What are the Celsius temperatures obtained with these liquid-in-glass scales which correspond to 60 ºC on the ideal gas scale?

Thermometer P Thermometer Q A. 50 ºC 60 ºC B. 60 ºC 60 ºC C. 60 ºC 70 ºC D. 70 ºC 60 ºC E. 100 ºC 90 ºC 13. External work W is done on a real gas and no

heat energy is allowed to enter or leave the gas. If the change in internal energy of the gas is ∆U, then

A. ∆U = 0. B. 0 < ∆U < W. C. ∆U = W. D. W < ∆U < 2 W. E. ∆U = 2 W. 14. White light is directed normally onto a

diffraction grating and the diffracted light is observed through the telescope of a spectrometer. The appearance of the zeroth order and the first order diffraction pattern will look like:

A.

W W W

B.

WR B B R

continuous spectrum C.

WB R R B

continuous spectrum D.

WR B B R

line spectrum E.

WR BR

line spectrum

B

(W : white, R : red, B : blue) 15.

concavemirror

lamp

The diagram above shows a small electric lamp

fixed on a wall and positioned on the principal axis of a concave mirror of radius of curvature 12 cm. If the mirror is 24 cm from the wall, which of the following statements best describes the appearance of the reflected light on the wall?

A. It is the form of a dot on the lamp. B. It is a circular patch on the lamp and the

same size as the lamp. C. It is a circular patch round the lamp but

smaller in size than the mirror. D. It is a circular patch round the lamp and

the same size as the mirror. E. It is a circular patch round the lamp but

larger in size than the mirror.

Page 49: AL Physics 1981-2004 MC

85 AL Physics/M.C./P.4

16. Two signal generators are connected to display

the formation of beats on a C.R.O. screen. For one particular setting of the two signal generators, the following pattern is observed on the C.R.O.:

2.5 cm If the C.R.O. time base is set at 0.2 ms/cm, the

beat frequency is A. 100 Hz. B. 200 Hz. C. 400 Hz. D. 1 000 Hz. E. 2 000 Hz. 17.

air

water

normal

ultra-soundbeam

A beam of ultrasound is being emitted from a

submarine under water towards the water surface. Which of the following statements is true?

A. The refracted beam leaving the surface

will bend away from the normal. B. The refracted beam will bend towards the

normal. C. The refracted beam will travel in the same

direction as the incident beam. D. Total internal reflection will occur. E. The refracted beam will travel along the

water surface. 18. The surface of a material of refractive index

1.8 is coated with a thin film of liquid of refractive index 1.5 and thickness 200 nm. White light falls normally on the thin film. Which of the following light wavelengths (in air) is not reflected from the thin film?

A. 400 nm B. 450 nm C. 600 nm D. 750 nm E. 800 nm 19. The image of a distant star produced by an

astronomical telescope is a diffraction pattern. If the effective diameter of the objective lens is reduced by one-half by covering its outer parts with a stop, the area of the central maximum of the diffraction pattern is

A. decreased by a factor of 4. B. halved. C. unchanged. D. doubled. E. increased by a factor of 4. 20. When parallel light is incident at the Brewster

angle in air on the surface of a glass block, A. the light is totally reflected. B. the reflected light is partially polarised. C. the transmitted light is unpolarised. D. the reflected and refracted wavefronts are

at right angles to each other. E. the angle of incidence is equal to the angle

of refraction. 21. A photograph is taken with a camera. The

exposure time required with an aperture of f-4 is 1 s. The exposure time required with an aperture of f-8 is

A. 0.25 s. B. 0.50 s. C. 1 s. D. 2 s. E. 4 s. 22. Which of these statements concerning electric

potential is INCORRECT? A. Potential is a scalar quantity. B. The potential difference between two

points, expressed in volts, is numerically equal to the change in the energy, expressed in joules, when a coulomb of charge is moved from one point to the other.

Page 50: AL Physics 1981-2004 MC

85 AL Physics/M.C./P.5

C. The potential is zero whether the electric field is zero.

D. The potential due to a point charge varies as 1/r, where r is the distance from the point charge.

E. The potential gradient at a point is proportional to the strength of the electric field at that point.

23. The energy of a 2 µF capacitor when charged

to a potential difference V is E. The energy of a 4 µF capacitor when charged to a potential difference 3 V is

A. 3 E. B. 6 E. C. 9 E. D. 12 E. E. 18 E. 24. Which one of the following combinations has

the least capacitance? A.

0.25 F 0.20 F0.50 Fµ µ µ

B.

0.50 Fµ

0.25 Fµ 0.20 Fµ C.

0.50 Fm

0.25 Fµ

0.20 Fµ D.

0.50 Fµ 0.25 Fµ

0.20 Fµ

E.

0.50 Fµ

0.25 Fµ

0.20 Fµ 25. Which of the following graphs best represents

the variation of the magnetic flux density B along the axis of a long current-carrying solenoid, with the distance x from the centre of the solenoid along the axis?

A.

x

B

0 B.

x

B

0 C.

x

B

0 D.

x

B

0

Page 51: AL Physics 1981-2004 MC

85 AL Physics/M.C./P.6

E.

x

B

0 26. An ammeter is connected in series with an

electric motor which is running freely, and the reading noted. When the motor raises a load at a steady speed, the reading of the ammeter will

A. remain the same. B. increase to a higher value. C. decrease to a lower value. D. increase to a maximum value and then

decrease. E. decrease to a minimum value and then

increase. 27.

intopaperB

A particle, of mass 3.2 × 10-26 kg and charge -1.6 × 10-19 C, enters a uniform magnetic field of flux density 0.08 T at a speed of 105 m/s, as shown. It will

A. pass undeviated through the magnetic

field. B. be deflected upward in a circular arc of

radius 0.25 m. C. be deflected upward in a circular arc of

radius 0.50 m. D. be deflected downward in a circular arc of

radius 0.25 m. E. be deflected downward in a circular are of

radius 0.50 m. 28.

disc

bar magnet

A bar magnet is freely suspended so that it

hangs horizontally above a flat aluminium disc which is rotating in a horizontal plane. The bar magnet will

A. rotate in the same direction as the disc. B. rotate in the opposite direction to the disc. C. remain at rest. D. oscillate to and fro in the horizontal plane. E. oscillate to and fro in the vertical plane. 29.

Y

L

XRi

L is a pure inductor, in series with a resistor R. The current i in the circuit is initially zero at time t = 0 and afterwards it rises linearly to reach a maximum at t = T; it then falls linearly to zero in the same time. Which of the following waveforms represents the variation of the potential V at point X with respect to time t?

A.

t

V

0 B.

t

V

0 C.

t

V

0

D.

Page 52: AL Physics 1981-2004 MC

85 AL Physics/M.C./P.7

t

V

0

E.

t

V

0

30. A black box, known to contain an electrical

device, is connected in series with a resistor R. The following circuit is set up:

50 Hza.c.

blackbox

R

ground

Y

Y

TodoublebeamCRO

1

2

The pattern observed on the double beam C.R.O. is shown below:

Y1Y2

The electrical device inside the box is A. a capacitor. B. a diode. C. an inductor. D. a resistor. E. a transistor. 31. Protactinium extracted from a solution of

uranyl nitrate decays with a half-life of 72 s. The value of the decay constant is

A. 9.6 × 10-3 s. B. 9.6 × 10-3 s-1.

C. 0.014 s-1. D. 49.9 s. E. 49.9 s-1. 32.

R6 VC

BE

A

P

40 Ω

10 kd.c.

+

_

In the transistor circuit shown above, the

voltages VAE, VPE, VCE and VBE were measured and tabulated as follows:

VAE VPE VCE VBE

3.6 V 2.6 V 2.5 V 0.6 V

What are the values for the base current IB and

the collector current IC? IB/mA IC/mA A. 0.2 7.8 B. 0.2 27.5 C. 0.26 7.8 D. 0.26 27.5 E. 0.68 34.0 33. In a gold-foil scattering experiment, an α -

particle and a proton, having the same kinetic energy, collide head-on with gold-nuclei. The ratio of the distance of closest approach of the α-particle to that of the proton is

A. 1 : 4. B. 1 : 2. C. 1 : 1. D. 2 : 1. E. 4 : 1. 34.

Page 53: AL Physics 1981-2004 MC

85 AL Physics/M.C./P.8

+ +

A dish containing a strong α-source is placed

inside a gold leaf electroscope containing dry air. If the gold-leaf is originally positively charged, what will happen to if after a few minutes?

A. It will increases in divergence. B. It will increases in divergence and then

decrease. C. It will collapse. D. It will collapse and then rediverge. E. There will be no change in divergence. 35.

X Y

The figure above shows two adjacent lines in the spectrum of a hot gas. Line X is brighter than line Y because

A. line X has a higher frequency. B. line X has a greater wavelength. C. line X originates in the hotter part of the

gas. D. for line X, more electrons undergo

transitions between the two states involved in the emission.

E. for line X, electrons undergo transitions between two states of greater energy difference.

36. Which of the following pairs of quantities have

the same dimensions? (1) pressure and Young modulus (2) angular momentum and Planck constant (3) coefficient of viscosity and gravitational

constant A. (1), (2) and (3) B. (1) and (2) only C. (2) and (3) only

D. (1) only E. (3) only 37.

r

F

0a b

The graph shows how the force F between two

atoms in a solid varies with the distance r between them. Which of the following statements about the distances a and b marked on the graph is/are correct?

(1) a is the smallest possible separation of the

atoms. (2) b is the equilibrium separation of the two

atoms. (3) The stiffness of the solid depends on the

slope of the curve near a. A. (1), (2) and (3) B. (1) and (2) only C. (2) and (3) only D. (1) only E. (3) only 38. According to the kinetic theory of gases, at a

given temperature, the molecules of all gases have the same

(1) average speed. (2) average kinetic energy. (3) average intermolecular potential energy. A. (1), (2) and (3) B. (1) and (2) only C. (2) and (3) only D. (1) only E. (3) only 39.

Page 54: AL Physics 1981-2004 MC

85 AL Physics/M.C./P.9

L

IS

A converging lens L and a lamp S are arranged

as shown. The rays from S converge at a point I after passing through L. Which of the following operations could enable a parallel beam of light to emerge from the lens L?

(1) moving the lens L to the left until parallel

rays are obtained (2) replacing L by a lens with less converging

power (3) placing a diverging lens of suitable focal

length in front of the lens L A. (1), (2) and (3) B. (1) and (2) only C. (2) and (3) only D. (1) only E. (3) only 40. In a compound microscope, (1) the final image produced is virtual. (2) the final image produced is erect. (3) the focal lengths of both the objective and

the eyepiece must be long in order to produce high magnification.

A. (1), (2) and (3) B. (1) and (2) only C. (2) and (3) only D. (1) only E. (3) only 41. A train of plane wavefronts is incident upon a

diverging lens. F denotes one of the focal points of the lens. Which of the following diagrams represent(s) what might happen to the wavefronts?

(1)

F

(2)

F

(3)

F

A. (1), (2) and (3) B. (1) and (2) only C. (2) and (3) only D. (1) only E. (3) only 42. A sound source S approaches an observer O at

a constant speed v. Which of the following statements is/are correct?

(1) The frequency of the source as heard by O

increases linearly with time as S approaches O.

(2) The sound appears louder and louder as S approaches O.

(3) The wavelength of the wave appears to be shorter as S approaches O.

A. (1), (2) and (3) B. (1) and (2) only C. (2) and (3) only D. (1) only E. (3) only 43.

C2C1

A charged capacitor C1 is connected to an

uncharged capacitor C2 as shown. If E1 = the total enrgy stored in C1 and C2 before

connection, and

Page 55: AL Physics 1981-2004 MC

85 AL Physics/M.C./P.10

E2 = the total energy stored in C1 and C2 after connected,

which of the follwoing is/are correct? (1) E1 = E2. (2) After connection, the p.d. across C1 is

equal to the p.d. across C2. (3) After connection, the charges stored in C1

and C2 are in the ratio

capacitance of

capacitance of

C

C1

2

.

A. (1), (2) and (3) B. (1) and (2) only C. (2) and (3) only D. (1) only E. (3) only 44. The e.m.f. of a battery is equal to (1) the total electrical power it generates

divided by the current it delivers. (2) the total electrical energy it releases, per

coulomb of charge when connected to an external circuit.

(3) the potential difference across its terminals when it is on open circuit.

A. (1), (2) and (3) B. (1) and (2) only C. (2) and (3) only D. (1) only E. (3) only 45.

insulting thread

source ofhigh voltage

+

_

G

A conducting ball is suspended between two

metal plates connected through a sensitive centre-zero galvanometer to a source of high p.d. The ball shuttles back and forth between the plates, making alternate contacts with each

plate. Which of the following statements is/are correct?

(1) The ball carries charges, sometimes

positive, and sometimes negative, (2) If the separation of the metal plates

decreases, the frequency of oscillation of the ball increases.

(3) The galvanometer shows a current flowing always in the same direction.

A. (1), (2) and (3) B. (1) and (2) only C. (2) and (3) only D. (1) only E. (3) only 46.

C BA

X R

GY

In the above circuit, C denotes the balance

position on the potentiometer wire AB. Which of the following procedures can shift C towards the end B?

(1) replacing the driving cell X by one with a

larger e.m.f. (2) adding a resistance in series with the

galvanometer G. (3) increasing the resistance of the rheostat R. A. (1), (2) and (3) B. (1) and (2) only C. (2) and (3) only D. (1) only E. (3) only 47. The ionisation potential of a hydrogen atom is

13.6 V. Which of the following energy levels is/are possible for the atom?

(1) - 1.51 eV (2) - 3.40 eV (3) - 6.80 eV A. (1), (2) and (3)

Page 56: AL Physics 1981-2004 MC

85 AL Physics/M.C./P.11

B. (1) and (2) only C. (2) and (3) only D. (1) only E. (3) only 48. The photons emitted by a laser (1) are of same frequency. (2) travel in the same direction. (3) are in phase. A. (1), (2) and (3) B. (1) and (2) only C. (2) and (3) only D. (1) only E. (3) only 49.

A

battery

C

X

microammeter

red light

Red light shines on the photoelectric cell C as shown. If the reading of the microammeter is zero, this may be explained by the fact that

(1) the e.m.f. of the battery is too small. (2) the intensity of the light is too low. (3) electrode X is made of a material with too

great a work function. A. (1), (2) and (3) B. (1) and (2) only C. (2) and (3) only D. (1) only E. (3) only 50. Which of the following phenomena can

provide direct evidence for the existence of discrete electron energy levels in atoms?

(1) electron diffraction (2) photo-electric emission (3) the spectrum of light from a sodium lamp A. (1), (2) and (3) B. (1) and (2) only C. (2) and (3) only D. (1) only E. (3) only

- End of Paper -

Page 57: AL Physics 1981-2004 MC

85 AL Physics/M.C./P.12

Question No. Key 1. B 2. C 3. A 4. C 5. E 6. D 7. C 8. D 9. B 10. D 11. D 12. C 13. C 14. B 15. E 16. E 17. B 18. A 19. E 20. D 21. E 22. C 23. E 24. A 25. D

Question No. Key 26. B 27. D 28. A 29. D 30. A 31. B 32. B 33. D 34. C 35. D 36. B 37. E 38. C 39. A 40. D 41. E 42. C 43. C 44. A 45. A 46. E 47. B 48. A 49. E 50. E

Page 58: AL Physics 1981-2004 MC

86 AL Physics/M.C./P.1

1986 Hong Kong Advanced Level Examination AL Physics

Multiple Choice Question 1.

2 kg

3 kg

30o

An object of mass 3 kg is placed on a smooth

plane inclined at 30º to the horizontal. It is connected by a light string passing over a frictionless pulley to another object of mass 2 kg, as shown above. Given that g = 10 m/s², when the system is released, the tension in the string will be

A. 18 N. B. 20 N. C. 24 N. D. 25 N. E. 30 N. 2.

30o 30o C

B

A

M

10 m

A small block M of mass 1 kg is transported

across a small hill along the road ABC by an applied force F which is always parallel to the road. The speed of M is kept constant throughout the journey and the kinetic friction between the block and the road is 2.60 N. The total work done by F in transporting M from A to C is

A. zero. B. 104 J. C. 152 J. D. 200 J. E. 304 J. 3. A football player is running at a velocity of 3

m/s due north. After a violent collision with another player, he is moving at a velocity of 4 m/s due east. Which of the following arrows best represents the direction of his change of velocity?

A.

E

S

W

N

B.

C.

D.

E. 4. A sphere X moving with velocity u on a

smooth horizontal plane makes a head-on collision with another sphere Y of the same mass which is initially at rest. If the collision is perfectly elastic, which of the following statements is/are correct?

(1) Kinetic energy is conserved in this

collision. (2) Linear momentum is conserved in this

collision. (3) X and Y stick together and move off with

the same velocity after the collision. A. (1), (2) and (3) B. (1) and (2) only C. (2) and (3) only D. (1) only E. (3) only 5.

Page 59: AL Physics 1981-2004 MC

86 AL Physics/M.C./P.2

h

P

A particle P is placed at a height h on a smooth

loop-the-loop track, as shown. The radius of the loop is r. P is now released from rest. If P is to complete the loop, the minimum value of h should be

A. 2 r. B. 2.5 r. C. 3 r. D. 4 r. E. 5 r. 6.

ba

E

K

0x

An object is attached to a light spring which

does not obey Hooke’s law. The mass is set oscillating so that the system has a constant total mechanical energy E. The graph above shows the variation of the kinetic energy K of the mass with the extension x of the spring. The object will experience a force of maximum magnitude at

A. x = 0 only. B. x = a only. C. x = b only. D. x = (a + b)/2 only. E. both x = a and x = b. 7.

Y

Q

P

X

The moments of inertia of a circular loop,

when rotated in turn about 3 different axes, are shown in the following table:

axis moment

of inertia

XY I1 PQ I2

an axis through Y and perpendicular to the plane of

the loop

I3

Which of the following is correct? A. I1 > I2 > I3 B. I2 > I1 > I3 C. I2 > I3 > I1 D. I3 > I1 > I2 E. I3 > I2 > I1 8.

S

a br rP 3

A satellite S moves around a planet P in an

elliptical orbit. The ratio of the speed of the satellite at point a to that at point b is

A. 1 : 9. B. 1 : 3. C. 1 : 1. D. 3 : 1. E. 9 : 1. 9.

r

FL

M

N

P

Q

Two neighbouring molecules with separation r

experience a force F between them. The graph shows how F varies with r. Which of the following statements is/are correct?

(1) Hooke’s law follows from the linearity of

the region LMN. (2) ε is the equilibrium separation of the

molecules.

Page 60: AL Physics 1981-2004 MC

86 AL Physics/M.C./P.3

(3) Thermal expansion can be explained by the fact that the curve NPQ is not symmetrical about P.

A. (1), (2) and (3) B. (1) and (2) only C. (2) and (3) only D. (1) only E. (3) only 10. Which of the following statements is/are

correct? (1) The surface tension of a liquid increases

with temperature. (2) Liquids tend to reduce their surface area as

much as possible. (3) Molecules in the surface layer of a liquid

have more potential energy than those inside the liquid.

A. (1), (2) and (3) B. (1) and (2) only C. (2) and (3) only D. (1) only E. (3) only 11. In a bromine diffusion experiment, the half-

brown level rises by 5 cm in 240 s. If the mean free path of bromine in air is 10-7 m, the number of collisions made by a bromine molecule within this period of time will be

A. 5 × 105. B. 5 × 107. C. 2.5 × 1011. D. 5 × 1011. E. 7 × 1011. 12. Two metal rods of the same Young modulus Y

and the same cross-sectional area A, but of different lengths L and 2L respectively, are joined together end-to-end to form a rod of length 3L. What are the Young modulus and the force constant of the composite rod?

Young modulus Force constant A. 2Y 3AY/(2L) B. 2Y 3AY/L C. 2Y AY/(3L) D. Y 3AY/L E. Y AY/(3L)

13. The electrical resistance of a certain device is given by R = a + bt², where t is the temperature indicated by a Celsius-scale mercury thermometer, and a, b are constants. If the device is calibrated at 0 ºC and 100 ºC, how will its performance, when used as a thermometer, compare with that of the mercury thermometer?

A. It will read the same temperature for all

temperatures. B. It will give a lower reading at all

temperatures. C. It will give a lower reading for

temperatures between 0 ºC and 100 ºC. D. It will give a lower reading for

temperatures above 100 ºC. E. It will never read the same temperature. 14.

N 2 N

container X container Y Container X holds N molecules of a certain gas

at atmospheric pressure. Container Y has the same volume as X and holds 2N molecules of the same gas. Both containers are at the same temperature. The ratio of the average number of collisions per second with the walls in X to those in Y is

A. 1 : √2. B. 1 : 2. C. 1 : 1. D. 2 : 1. E. 4 : 1. 15.

1 2

B

CA

P / Pa

V / 10-3 m3

105

1052 x

T1T2

The figure above shows two curves for a given

mass of gas at temperature T1 and T2. If the symbols P, V and T stand for the pressure, volume and absolute temperature of the gas, which of the following statements is FALSE?

Page 61: AL Physics 1981-2004 MC

86 AL Physics/M.C./P.4

A. Temperature T1 is twice temperature T2. B. PV/T is constant for all the points on the

two curves. C. When the gas expands from state B to state

C as represented by the curve BC, no energy is transferred from the surroundings to the gas.

D. No work is done by the gas when it changes from state A to state B along the line AB.

E. To take the gas from state A to state C, 100 J of energy is needed.

16. At 0 ºC temperature and 1.00 × 105 N/m²

pressure, the density of a gas is 0.179 kg/m³. The r.m.s. speed of the gas molecules at 91 ºC will be

A. 231 m/s. B. 470 m/s. C. 1 290 m/s. D. 1 490 m/s. E. 1 730 m/s. 17. For an ideal gas expanded isothermally, A. the work done by the gas is equal to the

heat energy added to it. B. the work done on the gas is equal to the

heat energy rejected by it. C. the work done on the surroundings is zero. D. the heat energy transferred to the gas is

zero. E. the internal energy of the gas increases. 18. When a parallel beam of white light passes

through a glass lens, it is separated into rays of different colours. For which colour of light does the lens have the greatest focal length and for which colour of light does glass have the greatest refractive index?

The lens has the

greatest focal length for

Glass has the greatest refractive

index for A. blue light blue light B. blue light red light C. yellow light yellow light D. red light blue light E. red light red light

19. A converging lens of focal length 12.5 cm is

used as a magnifying glass with the final image

formed at infinity. If the least distance of distinct vision is 25 cm, the angular magnification is

A. 1/2. B. 1. C. 2. D. 4. E. infinite. 20. A real object is placed in front of a convex

mirror of focal length f. Images are formed by the convex mirror for various object distances u. If the image distances are denoted by v, which of the graphs below shows the variation of |v| with |u| ?

A.

|u|

f

f

v

||

||

| |

B.

|u|

f

f

v

||

||

| |

C.

|u|

f

f

v

||

||

| |

D.

|u|

f

f

v

||

||

| |

E.

Page 62: AL Physics 1981-2004 MC

86 AL Physics/M.C./P.5

|u|

f

f

v

||

||

| |

21.

glass

water

air

Y

X

Z

θ

A glass vessel in the shape of a triangular

prism is filled with water, and light is incident normally on the face XY. If the refractive indices for water and glass are 4/3 and 3/2 respectively, total internal reflection will occur at the glass-air surface XZ only for sin θ greater than

A. 1/2. B. 2/3. C. 3/4. D. 8/9. E. 16/27. 22. In the diffraction of light round an obstacle, the

angle of diffraction is increased when A. the wavelength of the incident light wave

is increased. B. the wavelength of the incident light wave

is decreased. C. the amplitude of the incident light wave is

increased. D. the amplitude of the incident light wave is

decreased. E. the width of the obstacle is increased. 23. When a beam of unpolarised light travelling in

air falls on the surface of a block of glass, it is possible to find an angle of incidence such that

A. none of the light is reflected. B. all of the light is reflected. C. the reflected light is completely plane

polarised. D. the transmitted light is completely plane

polarised.

E. the reflected light and transmitted light are both completely plane polarised.

24. Lens Nature Focal length I converging 40 cm II converging 10 cm III diverging 40 cm IV diverging 10 cm A student is given the above 4 lenses, and

asked to make a telescope which will give erect magnified final images. How should he choose the lenses?

objective eyepiece A. I II B. I III C. I IV D. II III E. II IV 25. Two stretched wires are tuned to vibrate

transversely at the same frequency of 1 200 Hz. When the tension in one of the wires is reduced by 1%, beats are heard as both wires vibrate. The beat frequency is

A. 3 Hz. B. 6 Hz. C. 12 Hz. D. 24 Hz. E. 1 188 Hz. 26. An aeroplane flies horizontally at a low

altitude with a constant speed of 300 m/s. It transmits a radio signal of frequency 30 MHz as it passes a receiving station. What is the difference in the frequencies received a long time before and a long time after the passage of the plane?

A. 10 Hz. B. 30 Hz. C. 60 Hz. D. 120 Hz. E. 150 Hz. 27. If two independent sources, each separately at

a noise level of 70 dB, are sounded together, they will produce a noise level of

A. 35 dB. B. 70 dB. C. 73 dB.

Page 63: AL Physics 1981-2004 MC

86 AL Physics/M.C./P.6

D. 90 dB. E. 140 dB. 28.

X

Y

Two waveforms X and Y are displayed on a

C.R.O. screen. Which of the following statements is correct?

A. X leads Y by a phase difference of π/4. B. X leads Y by a phase difference of π/2. C. Y leads X by a phase difference of π/4. D. Y leads X by a phase difference of π/2. E. There is no phase difference between X

and Y. 29. An electric current is flowing in a copper wire

which is part of a circuit. The electrons in the copper wire

A. were at rest until the switch was closed but

now move along the wire with a drift velocity of a few millimetres per second.

B. were at rest until the switch was closed but now move along the wire at a speed approaching the speed of light.

C. had random motion onto which was imposed a drift velocity of a few millimetres per second when the switch was closed.

D. had random motion onto which was imposed a drift velocity of a few kilometres per second when the switch was closed.

E. had random motion until the switch was closed but now move along the wire at a speed approaching the speed of light.

30.

QP

In the capacitor arrangement shown in the

diagram, plate P has been changed and connected to the electroscope. If plate Q is moved towards plate P, the gold-leaf will

A. diverge a lot more. B. diverge a little more. C. remain where it is. D. fall slightly. E. fall completely. 31. Permittivity of free space = 8.85 × 10-12 F/m.

A conducting sphere of radius 0.1 m carries a positive charge of 10-4 C. A particle P of mass 2 × 10-5 kg carrying a negative charge of 10-10 C is released from rest at a distance of 1 m from the centre of the sphere. If the force due to gravity is neglected, the velocity of P when it strikes the surface of the sphere will be

A. 2.8 m/s. B. 9.0 m/s. C. 29.8 m/s. D. 81.0 m/s. E. 890 m/s. 32.

2 Ω

2 Ω

2 Ω

2 m H

2 µF

16 V

When a steady state is reached, the current

delivered by the battery is A. zero. B. 4.0 A. C. 5.0 A. D. 5.3 A. E. 8.0 A.

Page 64: AL Physics 1981-2004 MC

86 AL Physics/M.C./P.7

33.

YX

R

B

Z

V

S

In a potentiometer experiment, a balance point

was found on the potentiometer wire XY at Z. After switch S had remained closed for some time, it was found that the contact at Z had to be moved towards Y to maintain the balance. The most likely reason was that

A. XY was getting warm and its resistance

was increasing. B. R was getting warm and its resistance was

increasing. C. the galvanometer coil was getting warm

and its resistance was increasing. D. the cell B was running down. E. the cell V was running down. 34.

8 Ω 8 Ω 8 Ω

8 Ω 8 Ω

8 Ω 8 Ω8 Ω

X Y

In the circuit above, the equivalent resistance

across XY is A. 0 Ω. B. 10 Ω. C. 20 Ω. D. 40 Ω. E. 64 Ω. 35.

G

SP

Q R

x

y

B

The figure above shows 4 resistors, of

resistance P, Q, R and S in a Wheatstone bridge circuit. Which of the following statements is/are correct when the bridge is balanced?

(1) The p.d. across R is equal to the p.d.

across S. (2) The potential at point x is equal to the

potential at point y. (3) The current through G will still be zero

when the battery and the galvanometer are interchanged.

A. (1), (2) and (3) B. (1) and (2) only C. (2) and (3) only D. (1) only E. (3) only 36.

0 1 2 3

t /ms

Vs

(a)

V s V

10 µ F

10 Ω

(b) A square wave voltage Vs, as shown in Figure

(a), is applied to a circuit containing a 10 µF capacitor and a 10 Ω resistor connected in series. Which of the following graphs best represents the variation of the p.d. V across the resistor with time t?

A.

V

t / ms01 2

B.

V

t / ms01 2

C.

Page 65: AL Physics 1981-2004 MC

86 AL Physics/M.C./P.8

V

t / ms01 2

D.

V

t / ms01 2

E.

V

t / ms01 2

37.

A short bar magnet passes through an air-cored

solenoid of similar length with uniform velocity. Which of the following graphs best represents the variation of the current I in the solenoid with time t?

A.

t

I

0

B.

I

0 t C.

t

I

0 D.

I

0 t E.

t

I

0 38.

C LR

In an LCR-series circuit carrying a sinusoidal

alternating current, the voltages across the resistor and the capacitor are 4.0 V r.m.s. and 2.0 V r.m.s. respectively. If the applied voltage is 10.0 V r.m.s., then the r.m.s. voltage across the inductor will be

A. 4.0 V. B. 8.9 V. C. 9.8 V. D. 11.2 V. E. 13.8 V. 39.

Page 66: AL Physics 1981-2004 MC

86 AL Physics/M.C./P.9

1 cm

An alternating current passes through a resistor

R of resistance 10 Ω. The voltage across the resistor is measured by a C.R.O. The figure above shows the waveform on the screen. The y-sensitivity of the C.R.O. is 2 V/cm. The average power dissipated in the resistor is

A. zero. B. 0.1 W. C. 0.4 W. D. 0.8 W. E. 1.6 W. 40. X and Y are two long solenoids having the

same length and the same cross-sectional area. They are wound on the same type of core material. X has 500 turns and Y has 1 000 turns. If the self-inductance of X is 0.5 H, the self-inductance of Y is

A. 0.5 H. B. 0.7 H. C. 1.0 H. D. 2.0 H. E. 4.0 H. 41.

voltage

time

V2V1

The inputs to the X and Y plates of a C.R.O. are

V1 and V2, as shown above. Which of the diagrams below best represents the display on the screen?

A.

B.

C.

D.

E.

42. Which of the following circuits can give a half-

wave rectification waveform on a C.R.O. ? A.

C.R.O.To

B.

C.R.O.To

C.

C.R.O.To

D.

C.R.O.To

E.

Page 67: AL Physics 1981-2004 MC

86 AL Physics/M.C./P.10

C.R.O.To

43.

R

3 V

6 Vcb

e

A

S

X

Y In the above circuit, the sliding contact S is

moved between X and Y to give different voltages across ce. Which of the following graphs best represents the variation of the collector current Ic with the voltage Vce across the emitter and the collector?

A.

Vce

Ic

0 B.

Vce

Ic

0 C.

Vce

Ic

0 D.

Vce

Ic

0 E.

Vce

Ic

0 44.

+6 V

V

0 V

RRR

R

RInput 1

Input 2

In the above circuit, the reading of the

voltmeter is zero. What should be the voltages applied at Input 1 and Input 2 respectively?

Input 1 Input 2 A. 0 V 0 V B. 0 V 6 V C. 3 V 3 V D. 6 V 0 V E. 6 V 6 V 45.

thincopper

foil

fluorescentscreen

electronbeam

In order to observe a diffraction ring pattern on

the screen in the arrangement shown above, which of the following conditions must be met?

Page 68: AL Physics 1981-2004 MC

86 AL Physics/M.C./P.11

A. The copper foil must be a single crystal specimen.

B. The copper foil must be of uniform thickness.

C. The electron beam must strike the foil at normal incidence.

D. The electron beam must be polarised. E. The electron beam must be approximately

monoenergetic. 46.

I

II

III

IV

V

E

E1

In the diagram above, E1 and E∞ represent (to

scale) the energy levels of a hydrogen atom in its ground state and the ionised state, respectively. Which of the drawn lines represents the energy level of the atom in its first excited state?

A. I B. II C. III D. IV E. V 47. When a beam of light of intensity I and

frequency f is shone on the surface of a metal connected to earth, 200 electrons are ejected form the surface per second. If a light beam of intensity 2 I and frequency 2 f is used, the

number of electrons ejected from the metal per second will be

A. 50. B. 100. C. 200. D. 400. E. 800. 48. When fission occurs in a heavy nucleus, the

two nuclei produced (1) are stable. (2) contains more protons than neutrons. (3) have more binding energy per nucleon

than the original nucleus. A. (1), (2) and (3) B. (1) and (2) only C. (2) and (3) only D. (1) only E. (3) only 49. In an alpha-particle scattering experiment,

alpha particles having the same kinetic energy collide head-on with 47

107 Ag and 1327 Al nuclei

respectively. The ratio of the distance of closest approach for 47

107 Ag to that for 1327 Al is

A. 0.25. B. 0.28. C. 1.00. D. 3.62. E. 3.96. 50. Which of the following best represents the

order of magnitude of the radius of a nucleus? A. 10-2 m B. 10-6 m C. 10-10 m D. 10-14 m E. 10-18 m

- End of Paper -

Page 69: AL Physics 1981-2004 MC

86 AL Physics/M.C./P.12

Question No. Key 1. A 2. B 3. A 4. B 5. B 6. B 7. E 8. D 9. B 10. C 11. C 12. E 13. C 14. B 15. C 16. D 17. A 18. D 19. C 20. A 21. C 22. A 23. C 24. C 25. B

Question No. Key 26. C 27. C 28. B 29. C 30. D 31. B 32. B 33. D 34. B 35. A 36. C 37. A 38. D 39. D 40. D 41. E 42. E 43. E 44. A 45. E 46. D 47. C 48. E 49. D 50. D

Page 70: AL Physics 1981-2004 MC

87 AL Physics/M.C./P.1

1987 Hong Kong Advanced Level Examination AL Physics

Multiple Choice Question

1. Steel ball bearings, each of mass m, are fired at the rate of n ball bearings per second towards a fixed vertical steel block with a horizontal speed v. They rebound from the block with the same speed. The average force exerted on the block is

A. zero. B. mnv. C. 2mnv. D. mgnv. E. 2mgnv. 2. A parachutist of mass m falls in air under the

influence of gravity. The air resistance is equal to bv, where v is his speed and b is a constant. After falling a height s from rest, he reaches a terminal speed u. His kinetic energy at that instant is

A. mgs. B. mgs - bus. C. mgs - (m³g²)/(2b²). D. mgs + (m³g²)/(2b²). E. m³g²/(2b²). 3. A particle performs S.H.M. between two points

A and B with period T. If O is the centre of oscillation, the shortest time for the particle to move from point B to the mid-point of OA is

A. T/3. B. 3T/8. C. 2T/5. D. 2T/3. E. 3T/4. 4.

3 kg2 kg

axle

An object of mass 3 kg is tied to another object

of mass 2 kg with a string passing over a fixed smooth pulley. The weight of the pulley is

negligible. When the objects move under the action of gravity, the vertical upward force acting on the axle of the pulley is

A. zero. B. 10 N. C. 24 N. D. 48 N. E. 50 N. 5.

10 m

A

BC

A sphere is projected downwards from A with

a speed of 10 m/s at an angle of 30º to the horizontal. The sphere rebounds from the ground, first at B and then at C. If the collisions are perfectly elastic, the horizontal distance BC is equal to

A. 8.7 m. B. 10.0 m. C. 26.0 m. D. 43.5 m. E. 100.0 m. 6. A space capsule is launched with speed u from

the surface of the Earth to a maximum height above the ground equal to the radius of the Earth. A rocket is then fired horizontally which keeps the space capsule revolving in a circular orbit round the earth at that altitude with speed v. The ratio u : v is equal to

A. 1 : 2. B. 1 : √2. C. 1 : 1. D. √2 : 1. E. 2 : 1. 7. A toy car has a lead flywheel of moment of

inertia 0.001 kgm² attached to the axle of its rear wheels. The flywheel is now accelerated to rotate at 150 revolutions per minute and the toy car is allowed to move on a table. If the

Page 71: AL Physics 1981-2004 MC

87 AL Physics/M.C./P.2

effective decelerating force experienced by the car is 0.025 N, the car will stop after travelling a distance

A. 1.23 m. B. 2.47 m. C. 3.00 m. D. 4.93 m. E. 6.00 m. 8. A sphere and a cylinder, each having the same

mass and radius, are released together, side by side, at the top of an inclined plane and roll down along lines of greatest slope, without slipping. It is observed that the sphere reaches the bottom first. Which of the following statements is/are correct?

(1) The angular acceleration of each is the

same. (2) The cylinder has a greater moment of

inertia. (3) The kinetic energy of each is the same at

the bottom. A. (1), (2) and (3) B. (1) and (2) only C. (2) and (3) only D. (1) only E. (3) only 9. A metal wire is gradually loaded until the

elastic limit is exceeded, and then gradually unloaded. Which of the following graphs best represents the variation of stress with strain?

A.

strain

stress

0 B.

strain

stress

0 C.

strain

stress

0 D.

strain

stress

0 E.

strain

stress

0 10. In an experiment to measure the Young

modulus for steel in the form of a long wire, each measurement is made with the same percentage error. Which of the following measurements contributes the greatest error to the final result?

A. the applied force B. the diameter of the wire C. the extension of the wire D. the length of the wire E. The above 4 measurements will make an

equal contribution. 11.

h

B

A

r 2 r 12 2

Water, of density ρ , is introduced into a

capillary U-tube as shown. A and B are points just below the corresponding meniscuses. If the angle of contact is zero and the surface tension is T, then the pressure difference of the points A and B will be

Page 72: AL Physics 1981-2004 MC

87 AL Physics/M.C./P.3

A. 2 2

1 2

Tr

Tr

gh− + ρ and the pressure at A is

higher.

B. 2 2

1 2

Tr

Tr

gh− + ρ and the pressure at B is

higher. C. ρgh and the pressure at A is higher. D. ρgh and the pressure at B is higher.

E. 2 2

1 2

T

r

T

r− and the pressure at A is higher.

12.

S T

fluidflow

The above diagram shows a Pitot-static tube

situated in a moving fluid. A manometer connected to S and T shows a difference h in the liquid levels. If

v = the velocity of the moving fluid, d = the density of the moving fluid, ρ = the density of the liquid in the manometer, then v² is equal to A. 2ρgh/d. B. 2dgh/ρ. C. ρgh/d. D. dgh/ρ. E. ρdgh. 13.

0 2 4 6

100

200a b

c

/10-3 m3

/kPap

V

An ideal gas undergoes the processes marked

abc in the above p-V diagram. If the total heat energy supplied to the gas during the processes is 3100 J, the change in internal energy of the gas will be

A. -3900 J. B. -2700 J. C. 2300 J. D. 2700 J. E. 3500 J. 14. Two closed vessels X and Y contain equal

masses of an ideal gas. X has a greater volume than Y. When the temperature T changes, which of the following represents the variation of the pressure P of the gas in each vessel with temperature T?

A.

X

Y

P

/ CoT B.

Y

X

P

/ CoT C.

X, Y

P

/ CoT D.

X

Y

P

/ CoT E.

Y

X

P

/ CoT 15.

Page 73: AL Physics 1981-2004 MC

87 AL Physics/M.C./P.4

R R0 0

P Q

The variations of properties P, Q and R of

three different substances with temperature may be used to define three different temperature scales. The variations of P with R and of Q with R are shown above. Which of the following is/are correct?

(1) Temperature scales defined in terms of Q

and R will agree only at the fixed points. (2) Temperature scales defined in terms of P

and R will agree at all points. (3) Temperature scales defined in terms of P

and Q will not agree at any point. A. (1), (2) and (3) B. (1) and (2) only C. (2) and (3) only D. (1) only E. (3) only 16. An ideal gas inside a container without thermal

insulation is compressed very slowly to 70% of its original volume. While the gas is being compressed, there is a heat energy transfer of 5000 J. Which of the following statements is/are correct?

(1) Heat energy is transferred out of the

system. (2) Work done on the gas during compression

is equal to 5000 J. (3) The internal energy increases by 5000 J. A. (1), (2) and (3) B. (1) and (2) only C. (2) and (3) only D. (1) only E. (3) only 17.

c

N (c )

The graph shows the distribution of molecular

speeds c for a gas at room temperature. N(c) represents the number of molecules ∆N in a small range of speeds c to c + ∆c so that ∆N = N(c) ∆c. As the temperature is increased,

A. the peak position will move to the right

and its height will increase. B. the peak position will move to the right

and its height will decrease. C. the peak position will move to the left and

its height will increase. D. the peak position will move to the left and

its height will decrease. E. the peak position and its height will both

remain unchanged. 18. A stroboscope is used to freeze a wave pattern.

When the flashing frequency of the stroboscope is slightly reduced, the wave pattern appears to

A. move forward. B. move backward. C. move forward and then backward. D. move backward and then forward. E. remain stationary. 19.

P

A wave pulse is moving with uniform speed

along a rope attached to a fixed wall. A graph of the vertical displacement s against time t for a point P on the rope would be:

A.

0

s

t

B.

Page 74: AL Physics 1981-2004 MC

87 AL Physics/M.C./P.5

0

s

t

C.

0

s

t

D.

0

s

t

E.

0

s

t

20.

IO

M

Rays from a point light source O are reflected

by concave mirror M and converge to a point I as shown. Which of the following operations would enable a parallel beam of light to be reflected from M?

(1) moving the mirror M towards O (2) replacing M by a concave mirror of shorter

focal length (3) placing a converging lens of suitable focal

length between O and M A. (1), (2) and (3) B. (1) and (2) only C. (2) and (3) only

D. (1) only E. (3) only 21.

I

L

O

A real image I of an object O is formed by a

converging lens L as shown above. A diverging lens L1 is placed between L and I such that a real image I’ is obtained. Which of the following statements is/are correct?

(1) I’ is also an inverted image. (2) I’ is larger than I. (3) I’ is further away from L than I. A. (1), (2) and (3) B. (1) and (2) only C. (2) and (3) only D. (1) only E. (3) only 22.

A distant object consisting of two arrows is

viewed through an astronomical refracting telescope consisting of two converging lenses. Which of the following corresponds to the image seen?

A.

B.

Page 75: AL Physics 1981-2004 MC

87 AL Physics/M.C./P.6

C.

D.

E.

23.

detector

ofdirection

unpolarisedwave

z

yx

O

An unpolarised microwave travels along the

positive y-axis. A metal grid is placed in the xz-plane. The electromagnetic wave detected by the detector is

A. of zero intensity. B. unpolarised. C. plane-polarised in the yz-plane. D. plane-polarised in the xz-plane. E. plane-polarised in the xy-plane. 24.

MSO

A

B A plane mirror M is illuminated by

monochromatic light from a slit S. The virtual image of S by reflection and S itself act as 2 coherent sources and the interference pattern is observed on the screen AOB at a distance from

the mirror. Which of the following statements about the interference pattern on the screen is/are correct?

(1) No interference pattern can be seen in the

region OB on the screen. (2) As the mirror M moves downward, the

separation of the fringes decreases. (3) As the mirror M moves horizontally away

from the screen, the separation of the fringes increases.

A. (1), (2) and (3) B. (1) and (2) only C. (2) and (3) only D. (1) only E. (3) only 25. An open tube of length L and a string of length

L produce the same fundamental note. However when each is sounded independently the sound produced can be distinguished readily. This is because

A. the string produces some harmonics which

are not produced by the open pipe. B. the open pipe produces some harmonics

which are not produced by the string. C. they produce completely different

harmonics. D. the pipe produces longitudinal standing

waves while the string produces transverse standing waves.

E. they produce the same harmonics but the amplitudes of corresponding harmonics are different.

26. Two identical sound sources send out sound

waves to an observer. One of the sources is now moved away from the observer. The observer then hears alternate loud and weak signals, and loud signals are detected whenever the source moves through a distance of 0.50 m. If the speed of the source is 1/10 of the speed of sound, the wavelength emitted by the stationary source is

A. 0.25 m. B. 0.45 m. C. 0.50 m. D. 0.55 m. E. 1.00 m.

Page 76: AL Physics 1981-2004 MC

87 AL Physics/M.C./P.7

27. If 3/4 of the sound energy produced by a typewriter is absorbed by a sponge rubber pad placed underneath, the sound level produced will fall by

A. 0.25 dB. B. 0.75 dB. C. 3.00 dB. D. 6.00 dB. E. 12 dB. 28.

µF

µFµF

µF

3

1

4

2

C

DB

A

A voltage of 1200 V is applied across AB in

the capacitor network shown above. The voltage across CD is found to be 450 V. If after some time, the voltage across CD suddenly jumps to 600 V, which capacitor(s) has been shorted?

A. 1 µF B. 2 µF C. 3 µF D. 4 µF E. All 4 capacitors have been shorted. 29.

10 20 30 400

5

10

i /m A

t /s

A capacitor C is charged to a certain pd. and

then discharged through a resistor R. The variation of the current i with time t is shown in the above graph. Which of the following is/are correct?

(1) The time constant of the circuit is about 15 s.

(2) The area under the graph is proportional to the energy stored in the capacitor.

(3) If the resistance of R is doubled, the current at t = 0 will also be doubled.

A. (1), (2) and (3) B. (1) and (2) only C. (2) and (3) only D. (1) only E. (3) only 30.

X

YZ

W

-Q

-Qq

+Q

Three charges +Q, -Q and -Q are fixed at the

corners W, X and Y respectively of a square as shown. A forth charge, q, is fixed at Z, after which the charge at X experiences a NET electrostatic force indicated by the arrow. q is equal to

A. +Q. B. +2 Q. C. +4 Q. D. + 2 Q. E. +2 2 Q. 31.

V

ABS

3 V

1 Ω

In the above figure, AB is a resistance wire of

uniform cross-section, and S is a sliding contact. The 3 V battery has negligible internal resistance, and V is a high resistance voltmeter. Which of the following graphs shows the correct variation of voltage measured by V when the contact S is moved from A to B?

Page 77: AL Physics 1981-2004 MC

87 AL Physics/M.C./P.8

A.

3 V

A B B.

3 V

A B C.

3 V

A B D.

3 V

A B E.

3 V

A B 32.

G

R

40 Ω Ω20

0.5 A0.5 A

2 V

In the circuit shown, no current flows through

the galvanometer. If the internal resistance of the cell is negligible, the value of R is

A. 4.3 Ω.

B. 8 Ω. C. 10 Ω. D. 32 Ω. E. 160 Ω. 33.

X YB

A

2 Ω

The above potentiometer circuit is used to

calibrate an ammeter A. XY is the potentiometer wire and the potential drop across the wire is 0.02 V/cm. B is the balance point and XB = 75 cm. The current passing through the ammeter A is

A. 0.40 A. B. 0.75 A. C. 1.33 A. D. 1.50 A. E. 3.00 A. 34.

SX

YR

An a.c. supply S of frequency 50 Hz is

connected to a resistor R via two long, parallel, straight metal wires X and Y, as shown. The force between X and Y

A. is equal to zero. B. always attracts. C. always repels. D. sometimes attracts and sometimes repels;

the frequency of variation is 50 Hz. E. sometimes attracts and sometimes repels;

the frequency of variation is 100 Hz. 35.

Page 78: AL Physics 1981-2004 MC

87 AL Physics/M.C./P.9

0.5 a

0.4 a0.3 a

P Q

R

0 x

y

P and Q represent two long, straight, parallel,

conducting wires separated by a distance of 0.5 a, as shown in the figure above. Each of them carries a current I flowing into the plane of the paper. The x-component of magnetic density at the point R is

A. zero.

B. µπ0

21 17

Ia

× . .

C. µπ0

24 00

Ia

× . .

D. µπ0

24 17

Ia

× . .

E. µπ0

25 38

Ia

× . .

36.

constantcurrent

PQ

S

N

The figure shows the essential parts of an

apparatus to demonstrate the Hall effect. Which of the following statements is/are correct?

(1) In the arrangement above, the Hall p.d. is

developed across PQ. (2) The magnitude of the Hall potential is

greater if the applied magnetic flux density is increased.

(3) The magnitude of the Hall potential is less if the width PQ of the specimen is decreased.

A. (1), (2) and (3) B. (1) and (2) only C. (2) and (3) only D. (1) only E. (3) only

37.

N S

C

G

A narrow coil C connected to a sensitive

galvanometer G is placed between the poles of a powerful electromagnet, with its plane normal to the magnetic field which is uniform. Initially the electromagnet is on, then it is switched off at time ts. Which of the following graphs best represents the variation of the galvanometer current i with time t?

A.

t

i

0ts

B.

t

i

0ts

C.

t

i

0ts

D.

t

i

0ts

Page 79: AL Physics 1981-2004 MC

87 AL Physics/M.C./P.10

E.

t

i

0ts

38.

field

loop

AB

The uniform magnetic field shown pointing

into, and acting perpendicular to, the plane of the paper is confined to a cylindrical volume of radius 8 cm. The magnetic flux density now decreases at a constant rate of 0.01 T/s. A circular loop of radius 10 cm is placed so that its plane is perpendicular to the magnetic field, as shown. What will be the magnitude and direction of the induced e.m.f. in the loop?

Magnitude Direction A. 2.0 × 10-4 V from A to B via the loop B. 2.0 × 10-4 V from B to A via the loop C. 3.1 × 10-4 V from A to B via the loop D. 3.1 × 10-4 V from B to A via the loop E. there is no induced e.m.f. 39.

t

V

0

voltage

The voltage of an a.c. source varies with time t

as shown. The r.m.s. value of the applied voltage is

A. zero. B. V/2. C. V/√2. D. V. E. V√2.

40.

primarycoil

secondarycoil

The above diagram shows a step-down voltage

transformer. Which of the following is/are correct?

(1) The dotted line indicates the path of the

eddy current. (2) If the terminals of the secondary coil are

shorted, the primary current will increase. (3) The number of turns in the primary is

higher than that of the secondary. A. (1), (2) and (3) B. (1) and (2) only C. (2) and (3) only D. (1) only E. (3) only 41. A 120 V, 60 W lamp is run from a 240 V, 50

Hz mains supply using a capacitor connected in series with the lamp and supply. What is the theoretical value of the capacitor required to operate the lamp at its normal rating?

A. 3.8 µF B. 6.6 µF C. 7.7 µF D. 13.3 µF E. 83.3 µF 42.

RC

In the above a.c. smoothing circuit, the ripple

on the current passing through the load R can be reduced by increasing

(1) the load resistance R. (2) the capacitance C.

Page 80: AL Physics 1981-2004 MC

87 AL Physics/M.C./P.11

(3) the a.c. supply frequency. A. (1), (2) and (3) B. (1) and (2) only C. (2) and (3) only D. (1) only E. (3) only 43. A NPN transistor is operated as a linear

voltage amplifier and the output voltage is displayed on a CRO screen as shown:

V1.5 V

R

R6 V

Vin

b

L

out

Vdc

Vpp3 V

0 time

Vout

What changes will occur when RL is slightly

increased? Vdc Vpp A. decreases increases B. decreases decreases C. increases increases D. increases decreases E. no change increases 44.

output

IN 2

IN 1

A certain logic gate with two inputs IN 1, IN 2

and an output is shown above. The variation of the input and output voltages are as follows:

0

6

0

6

6

0

output/V

IN 2/V

IN 1/V

time

time

time

What is the logic gate? A. NOT gate B. OR gate C. NOR gate D. AND gate E. none of the above 45.

0 V

+6 V

15 kΩ

2.2 kΩ

VoutVin= 0.8 V

0.7 V

In the above circuit, if the current amplification

factor β = 130, what is the output voltage Vout? A. 0 V B. 3.0 V C. 4.1 V D. 4.9 V E. 6 V 46. The timebase of a C.R.O. is set at 5 ms/cm.

An a.c. supply of frequency 50 Hz is now applied to the y-plates of the C.R.O. If the width of the screen of the C.R.O. is 10 cm, how many cycles can be displayed on the screen?

A. 0.25 B. 0.5 C. 1 D. 2.5

Page 81: AL Physics 1981-2004 MC

87 AL Physics/M.C./P.12

E. 5 47.

= 1

= 2

= 3 = 4

n

n

nn

The figure shows the four lowest energy levels

of a hydrogen atom. The hydrogen atom is excited from ground state to the energy level n = 3 when an electron collides inelastically with it. What is the minimum energy required for the electron to do this?

(ionisation potential of hydrogen = 13.6 V) A. 4.9 eV B. 12.1 eV C. 12.8 eV D. 15.1 eV E. 20.0 eV 48.

p.d.

current

III

III

IV_ + The above figure shows the currents observed

in a photocell circuit as a function of the p.d. between the plates of the photocell when light beams I, II, III and IV were each directed in

turn at the cathode. Which of the beams has the highest frequency?

A. I B. II C. III D. IV E. they all have the same frequency. 49.

gold

foil

R

QP

α

In an experiment on α -particle scattering, α -

particles are directed onto a gold foil, and detectors are placed at positions P, Q and R as shown. What is the distribution of α-particles as recorded at P, Q and R respectively?

P Q R A. all none none B. none none all C. most some none D. most some few E. few some most 50. Which of the following experiments provide

direct evidence for the existence of energy levels in atoms?

A. the Chadwick experiment B. the Franck-Hertz experiment C. the Geiger-Marsden experiment D. the Millikan experiment E. none of the above

- End of Paper -

Page 82: AL Physics 1981-2004 MC

87 AL Physics/M.C./P.13

Question No. Key 1. C 2. E 3. A 4. D 5. C 6. D 7. D 8. C 9. B 10. B 11. D 12. A 13. C 14. E 15. B 16. B 17. B 18. A 19. D 20. D 21. A 22. A 23. E 24. B 25. E

Question No. Key 26. B 27. D 28. D 29. D 30. E 31. C 32. C 33. B 34. C 35. D 36. B 37. A 38. A 39. D 40. C 41. C 42. A 43. A 44. E 45. C 46. D 47. B 48. B 49. E 50. B

Page 83: AL Physics 1981-2004 MC

88 AL Physics/M.C./P.1

1988 Hong Kong Advanced Level Examination AL Physics

Multiple Choice Question

1.

water

current ZY

X

A boy wished to swim across a river with

parallel banks as shown in the diagram above. Assuming the water current is flowing at 0.5 m/s, and the swimming speed of the boy is 1 m/s, along which direction should the boy swim if he wishes

(1) to reach the opposite bank in the shortest

time? (2) to take the shortest route to the opposite

bank? (1) shortest time (2) shortest route A. X Y B. X Z C. Y X D. Y Z E. Z Y 2. A smooth block of mass 2 kg slides down a

wedge. The wedge, of mass 10 kg, is placed on a horizontal table, and its inclined plane makes an angle of 30º with the horizontal. If the wedge remains stationary all the time, the normal reaction of the table acting on the wedge is

A. 15 N. B. 85 N. C. 100 N. D. 115 N. E. 120 N. 3. A football player is running at a velocity of 3

m/s due north. After a violent collision with another player, he is moving at a velocity of 4 m/s due east. Which of the following arrows best represents the direction of his change of velocity?

A.

E

S

W

N

B.

C.

D.

E. 4. A force F is applied to an initially stationary

particle from time t = 0. F varies sinusoidally with time t as shown in the following graph:

t

F

0

Which of the following graphs best represents

the variation of the subsequent velocity v of the particle with t?

A.

Page 84: AL Physics 1981-2004 MC

88 AL Physics/M.C./P.2

t

v

0

B.

t

v

0

C.

t

v

0

D.

t

v

0 E.

t

v

0 5.

time

velocity

0

The graph shown is a possible representation

of the relation between velocity and time for the motion of

(1) a mass, attached to a spiral spring, pulled

vertically downwards and then released.

(2) the spot on the screen of a CRO when a linear time base is operating.

(3) a ball dropped vertically onto a hard, smooth and horizontal surface, where it undergoes elastic collisions.

A. (1), (2) and (3) B. (1) and (2) only C. (2) and (3) only D. (1) only E. (3) only 6.

A car of mass m travels into a region where the

track is an arc of a vertical circle of radius r. At the bottom of this arc, the car travels at speed v. At this position the vertical force exerted upwards by the track on the car is

A. mv²/r. B. mg. C. mv²/r - mg. D. mg - mv²/r. E. mg + mv²/r. 7.

mS 1 S 2

A block of mass m is attached to two identical

springs S1 and S2 as shown. The force constant of the springs is k. If the block is made to execute simple harmonic motion, the period will be

A. 24

π mk

.

B. 22

π mk

.

C. 2π mk

.

D. 2 2π mk

.

Page 85: AL Physics 1981-2004 MC

88 AL Physics/M.C./P.3

E. 2 4π mk

.

8. A satellite moving round the Earth in a circular

orbit of radius R has a period T. What would the period be if the orbit were of radius R/4?

A. T/8 B. T/4 C. T/2 D. 2T E. 4T 9. A body is suspended by a string and allowed to

swing as a simple pendulum. When it is moved from the north pole to the equator, its period will

A. remain constant. B. decrease. C. increase. D. decrease and then increase. E. increase and then decrease. 10.

strain

stress

wire 1

wire 2

wire 3

Three wires of different material, but of the

same length and cross-sectional area are stretched until they break. Their stress/strain curves are shown in the figure above. If E1, E2 and E3 represent the energy required to break wire 1, wire 2 and wire 3 respectively, which of the following is correct?

A. E1 > E2 > E3 B. E3 > E2 > E1 C. E2 > E1 > E3 D. E2 > E3 > E1 E. E3 > E1 > E2 11.

0 separation rr0

-E

potentialenergy

The graph above shows how the potential

energy between molecules of a substance varies with their separation. Which of the following is an INCORRECT inference from the graph?

A. No resultant force acts on each molecule

when r = r0. B. The force between molecules is repulsive

when r < r0. C. The average separation between the

molecules is r0. D. The energy required to separate two

molecules completely is E. E. The larger the value of E, the higher is the

melting point of the substance. 12. For smoke particles undergoing Brownian

motion in air, the motion of the smoke particles is mainly caused by

A. air convention currents. B. the interaction between oxygen and

nitrogen molecules. C. collisions between air molecules. D. collisions between smoke particles. E. collisions between air molecules and

smoke particles. 13. If the surface tension of a soap solution is T,

the work done in increasing the radius of a soap bubble from a to 2a is

A. 2 π a² T. B. 4 π a² T. C. 8 π a² T. D. 12 π a² T. E. 24 π a² T. 14. An ideal gas is contained in two metal

cylinders A and B connected by a tap, which is

Page 86: AL Physics 1981-2004 MC

88 AL Physics/M.C./P.4

initially closed. The volume and pressure of the gas in the cylinders are as follows:

pressure/Pa volume/m³ A 5 × 105 11 × 10-3 B 2 × 105 4 × 10-3 When the tap connecting the two cylinders is

opened, what will be the final pressure in the vessel? You may assume that the temperature remains constant.

A. 2.4 × 105 Pa. B. 3.5 × 105 Pa. C. 4.2 × 105 Pa. D. 5.0 × 105 Pa. E. 6.9 × 105 Pa. 15. A piston is SLOWLY pushed into a metal

cylinder containing an ideal gas. Which of the following statements is INCORRECT?

A. The mass of the gas remains the same. B. The pressure of the gas increases. C. The number of the molecules per unit

volume increases. D. The average speed of gas molecules

increases. E. The frequency of collision of the gas

molecules with the piston increases. 16. The energy which must be supplied to 1 mole

of gas initially at absolute temperature T, pressure p and volume V, to raise its temperature by 1K, is Q1 at constant pressure and Q2 at constant volume. The value of Q2 is

A. Q1. B. Q1 + pVT. C. Q1 - pVT. D. Q1 - pV/T. E. Q1 + pV/T. 17. An ideal gas is at temperature T. If the mass of

a gas molecule = m, the molar gas constant = R and the Avogadro constant = NA, then the r.m.s. speed of the molecules is

A. 3RT m/ .

B. RT mN A/ ( ) .

C. 3RT mN A/ ( ) .

D. RTN mA / .

E. 3RTN mA / . 18. Which of the following is NOT an assumption

in deriving the kinetic theory of gases? A. The volume of the molecules is negligible

compared with the volume of the gas. B. Attractive forces between the molecules

are negligible. C. The duration of a collision is negligible

compared with the time between collisions.

D. Collisions with the walls of the container and with other molecules cause no change in the average kinetic energy of molecules.

E. The molecules suffer negligible change of momentum on collision with the walls of the container.

19. The mean free path of air molecules is equal to A. the average diameter of an air molecule. B. the average separation between air

molecules. C. the average distance between collisions of

air molecules. D. the average distance travelled by an air

molecule in unit time. E. the distance travelled by an air molecule in

free space before coming to rest. 20. An inexpansible vessel contains 1.2 kg of gas

at 300 K. What is the mass of gas expelled from the vessel if it is heated from 300 K to 400 K under constant pressure?

A. 0.25 kg B. 0.3 kg C. 0.6 kg D. 0.75 kg E. 0.9 kg 21.

Page 87: AL Physics 1981-2004 MC

88 AL Physics/M.C./P.5

0.2 m

0.2 m

When a pin is moved along the principal axis

of a small concave mirror, the image position coincides with the object at a point 0.5 m from the mirror. If the mirror is placed at a depth of 0.2 m in a transparent liquid, the same phenomenon occurs when the pin is placed 0.4 m from the mirror. The refractive index of the liquid is

A. 6/5. B. 5/4. C. 4/3. D. 3/2. E. 5/3. 22. A projector lens produces a clear, enlarged

image of a slide on a screen. How could a clear image with greater magnification be obtained?

screen

movement lens movement

A. farther away closer to the slide B. farther away none C. closer away from the slide D. none closer to the slide E. closer closer to the slide

23.

When monochromatic light is incident

normally on a wedge-shaped thin air film, an interference pattern may be seen by reflection. Which of the following is/are correct?

(1) Parallel fringes are observed. (2) If water is introduced into the region

between the plates, the fringe separation decreases.

(3) If the angle of the wedge is increased, the fringe separation decreases.

A. (1), (2) and (3) B. (1) and (2) only C. (2) and (3) only D. (1) only E. (3) only 24.

light source MS

Polarized light is obtained by passing a narrow

beam of unpolarized light from source S through a tank of water to which a drop of milk has been added. Which of the following statements is/are correct?

(1) Light from source S must be

monochromatic. (2) Completely polarized light is detected at

position M. (3) The drop of milk provides particles to

scatter the light. A. (1), (2) and (3) B. (1) and (2) only C. (2) and (3) only D. (1) only E. (3) only 25.

P QAO

A loudspeaker at O produces a progressive

sound wave of frequency 330 Hz which propagates along OA with a speed of 330 m/s. The phase difference between the air vibrations at P and Q, 0.5 m apart, is

A. dependent on the distance OP. B. zero. C. 0.5 radians. D. π/2 radians. E. π radians.

Page 88: AL Physics 1981-2004 MC

88 AL Physics/M.C./P.6

26.

displacement

0a a a32 x

The figure above shows the variation of the

displacement of air molecules along the x-axis in a standing sound wave at a particular time. At what positions will the pressure remain constant with respect to time?

A. x = 0 and x = 2a only B. x = a and x = 3a only C. x = 0 and x = a only D. x = 2a and x = 3a only E. x = 0, x = a, x = 2a and x = 3a 27. A moving train sounds a whistle of frequency

500 Hz. The apparent frequency heard by an observer standing close to the railroad is 462 Hz. If the speed of sound in air is 300 m/s, the train is moving at a speed of

A. 23 m/s away from the observer. B. 23 m/s towards the observer. C. 25 m/s away from the observer. D. 25 m/s towards the observer. E. 28 m/s away from the observer. 28. If the sound level of a source increases by 6

dB, the power emitted by the source will have its initial value multiplied by approximately

A. log10 6. B. 2. C. 4. D. 6. E. 106. 29. Which of the following is/are equivalent to a

unit energy? (1) electron-volt (2) kilowatt-hour (3) volt-coulomb A. (1), (2) and (3)

B. (1) and (2) only C. (2) and (3) only D. (1) only E. (3) only 30.

a

a

a

a

+q

+q -2

-2

q

q

Four particles carrying charges + q, + q, -2 q

and -2 q are placed at the vertices of a square of side a. The electric potential at the centre of the square is

A. zero.

B. − 22 0

qaπε

.

C. − 2

0

qaπε

.

D. + 22 0

qaπε

.

E. + 2

0

qaπε

.

31.

- 400 V

- 300 V

- 200 V

- 100 VP Q

R

The diagram shows points of equal potential

joined as equipotential lines. Which of the following statements is/are correct?

(1) The electric field at P is in a direction

tangential to the line passing through P. (2) The electric field is the same at the points

P and Q. (3) Work has to be done in moving an

electron from point P to point R. A. (1), (2) and (3) B. (1) and (2) only

Page 89: AL Physics 1981-2004 MC

88 AL Physics/M.C./P.7

C. (2) and (3) only D. (1) only E. (3) only 32. If a metal conductor, of cross-sectional area A,

has n free electrons per unit volume, each carrying a charge e and moving with a drift velocity v, the

(1) current density will be nev. (2) current flow will be evA. (3) drift velocity will increase with

temperature. A. (1), (2) and (3) B. (1) and (2) only C. (2) and (3) only D. (1) only E. (3) only 33.

vertical+

_

H

h

Y

X

A potential difference V is maintained between

plates X and Y, separated by a distance h. A particle of mass m and positive charge q enters the region between X and Y through the hole H with negligible velocity. If it makes no collisions on the way, it will strike X with kinetic energy.

A. mgh. B. qV. C. qV + mgh. D. qV - mgh. E. mgh - qV. 34. A parallel-plate capacitor is charged by a

battery and the energy stored in the capacitor is E. The connections to the battery are then removed. If one of the plates is moved so that the plate-separation is halved, the electrical energy stored in the capacitor will

A. decrease to E/4. B. decrease to E/2. C. remain the same. D. increase to 2E. E. increase to 4E.

35.

BC

R

S

Using the vibrating reed switch S, in the above

circuit, the capacitor C is charged from battery B and completely discharged through the resistor R with a frequency of 25 Hz. When a CRO is connected across R, the waveform will look like:

A.

B.

C.

D.

E.

36.

Page 90: AL Physics 1981-2004 MC

88 AL Physics/M.C./P.8

0.40.3

P Q

R

0 x

y

a

a a

0.5 P and Q represent two long, straight, parallel,

conducting wires separated by a distance of 0.5 a, as shown in the figure above. Each of them carries a current I flowing into the plane of the paper. The magnitude of the y-component of the magnetic induction at point R is

A. zero.

B. µπ0

2117

Ia

× ( . ).

C. µ

π0

24.00).

I

a× (

D. µ

π0

24.17

I

a× ( ).

E. µ

π0

25 38).

I

a× ( .

37. A particle of mass m and charge q moves in a

circular orbit in a magnetic field B. The time taken to complete a single orbit is

A. Bq/(2πm). B. 2πm/(Bq). C. 2mq/(Bπ). D. Bm/(2πq). E. 2π/(mBq). 38.

M

N

H Ky

x

I1

I2

In the figure shown, MN is a fixed long

conductor, carrying a current I1. HK is another conductor perpendicular to MN. When a current I2 is allowed to pass through HK in the direction shown, the force on HK

A. acts in the +y direction.

B. acts in the -y direction. C. acts in the +x direction. D. acts in the -x direction. E. is zero. 39.

P Q

L Two coaxial solenoids P and Q are arranged as

shown. The cross-sectional areas of P and Q are A1 and A2 respectively. P contains N1 turns while Q contains N2 turns. Q is now connected to a power supply so that the current through it rises at a uniform rate S. The maximum e.m.f. induced in P is

A. µ0N1N2A1S. B. µ0N1N2A2S. C. µ0N1N2A1S/L. D. µ0N1N2A2S/L. E. zero. 40.

If the pattern shown above appears on the CRO

screen when two sinusoidal a.c. voltages are applied across the x-plates and the y-plates, what is the ratio of the frequency of the voltage across the x-plates to that across the y-plates?

A. 1 : 4 B. 1 : 2 C. 1 : 1 D. 2 : 1 E. 4 : 1 41. A resistor R and a capacitor C are connected in

series with an a.c. supply. The r.m.s. applied voltage and the r.m.s. current are V and I, respectively. If the resistance of R is one half

Page 91: AL Physics 1981-2004 MC

88 AL Physics/M.C./P.9

of the total impedance of the circuit, the power consumed in the circuit will be

A. IV/4. B. IV/2. C. IV/ 3 . D. IV. E. 3 IV/2. 42.

fieldintopaper

wire frame

A rectangular wire frame surrounds a uniform

magnetic field which is confined to a square region as shown in the diagram. The magnetic field is situated at the centre of the frame and is perpendicular to the plane of the paper. If the frame moves to the right with a uniform velocity, which of the graphs below best represents the variation of the induced current i with time t?

(The clockwise direction of the current is taken

as positive.) A.

0 t

i

B.

0 ti

C.

0 t

i

D.

0 ti

E.

0 t

i

43.

E

E

E

E

E

1

2

3

4

5

The diagram shows the first five energy levels

of an atom. Which of the spectra below best corresponds to the transitions indicated?

low highfrequency

A.

B.

C.

D.

Page 92: AL Physics 1981-2004 MC

88 AL Physics/M.C./P.10

E.

44. An α-source originally consisted entirely of the

element polonium. After the emission of a single α-particle, each polonium atom becomes an atom of lead. At the end of two years, the source was found to contain 98% lead and 2% polonium. At the end of one year, the sample would have had the approximate composition:

A. 14% lead, 86% polonium. B. 25% lead, 75% polonium. C. 50% lead, 50% polonium. D. 75% lead, 25% polonium. E. 86% lead, 14% polonium. 45.

Light falls on the photo-sensitive metal surface

of a photocell. A battery and a sensitive meter are connected to the photocell as shown. Which of the following statements is correct?

A. The number of electrons emitted from the

metal surface per second is proportional to the potential difference between the metal surface and the anode.

B. No current is observed in the meter until after a considerable time, when the metal surface has heated up.

C. The maximum energy of the electrons emitted is proportional to the intensity of light.

D. The maximum kinetic energy of the electrons emitted is independent of the particular metal used.

E. No current is observed in the meter unless the frequency of light is above a minimum value.

46.

wavelength

intensity

minλ The graph above shows the spectrum of X-rays

from an X-ray tube. When the accelerating potential is increased, what will happen to λmin and the wavelengths of the characteristic lines?

λmin wavelengths of characteristics lines A. decreased unchanged B. decreased decreased C. decreased increased D. increased unchanged E. increased decreased 47.

alphaparticles

berylliumparaffinwax

ionisationchamber

X Y

The figure above shows the experimental set-

up used by Chadwick to demonstrate the existence and properties of neutrons. Radiation X is produced when beryllium is bombarded by alpha-particles. Radiation Y is produced when X bombards paraffin wax. Which of the following statements concerning this experiment is NOT true?

A. X is not affected by magnetic fields. B. The current registered by the ionisation

chamber increases when the sheet of paraffin wax is removed.

C. Y is a stream of protons. D. Both momentum and mechanical energy is

conserved for the collision of X on paraffin wax to produce Y.

E. The mass of X is found to be approximately that of a proton.

Page 93: AL Physics 1981-2004 MC

88 AL Physics/M.C./P.11

48. In a controlled thermal fission reactor, the use of the control rods will NOT affect

A. the speed of the neutrons released on

fission. B. the rate of production of the neutrons. C. the energy generated in the nuclear

reactor. D. the amount of radioactive radiations

produced in the nuclear reactor. E. the rate of disintegration of the 235U

nucleus. 49. If electron A has twice the kinetic energy of

electron B, what is the ratio of the de Broglie wavelength of electron A to that of electron B?

A. 1 : 2 B. 1 : √2 C. 1 : 1 D. √2 : 1 E. 2 : 1

50.

electronsource

sheet X

fluorescentscreen

In an evacuated tube, a beam of electrons is

accelerated through a potential difference V and strikes a graphite sheet X. Rings are seen on a fluorescent screen positioned behind X. Which of the following statements is/are correct?

(1) Sheet X acts as a diffraction grating. (2) The rings indicate the wave nature of

electrons. (3) When V is increased, the rings are wider

apart. A. (1), (2) and (3) B. (1) and (2) only C. (2) and (3) only D. (1) only E. (3) only

- End of Paper -

Page 94: AL Physics 1981-2004 MC

88 AL Physics/M.C./P.12

Question No. Key 1. D 2. D 3. A 4. D 5. E 6. E 7. D 8. A 9. C 10. D 11. C 12. E 13. E 14. C 15. D 16. D 17. C 18. E 19. C 20. B 21. D 22. A 23. A 24. E 25. E

Question No. Key 26. B 27. C 28. C 29. A 30. B 31. E 32. D 33. C 34. B 35. B 36. A 37. B 38. A 39. D 40. B 41. B 42. C 43. E 44. E 45. E 46. A 47. B 48. A 49. B 50. B

Page 95: AL Physics 1981-2004 MC

89 AL Physics/M.C./P.1

1989 Hong Kong Advanced Level Examination AL Physics

Multiple Choice Question 1. A micrometer screw gauge is used to measure

the diameter of a piece of wire. The following readings were obtained:

mean zero reading -0.05 ± 0.02 mm,

and mean apparent diameter +1.05 ± 0.02 mm. The diameter of the wire should be written as A. 1.00 ± 0.02 mm. B. 1.00 ± 0.04 mm. C. 1.10 ± 0.00 mm. D. 1.10 ± 0.02 mm. E. 1.10 ± 0.04 mm. 2.

waterstill water

A wooden block of density 800 kg/m³ and

volume 1.0 m³ is fastened to the bottom of a fresh-water pond as shown above. If the string suddenly breaks, the initial acceleration of the block will be close to

A. 0.25 m/s². B. 1.25 m/s². C. 2.5 m/s². D. 10 m/s². E. 12.5 m/s². 3. Two ships X and Y travel with equal speeds. X

moves North and Y moves East. Which of the following best gives the direction of the velocity of X relative to Y?

A.

N

B.

C.

D.

E.

4.

spring A

spring B

2 N

3 N Two objects of weights 2 N and 3 N are

suspended from a fixed point by two identical light springs A and B as shown in the diagram. The force constants of the springs are both 1 N/cm. What are the extensions of springs A and B?

Page 96: AL Physics 1981-2004 MC

89 AL Physics/M.C./P.2

Extension of spring A

Extension of spring B

A. 5 cm 3 cm B. 5 cm 2 cm C. 3 cm 2 cm D. 3 cm 5 cm E. 2 cm 3 cm

5.

30o 30o

Y

SP

X

In the figure above, X and Y are blocks of mass

1 kg and 2 kg respectively. S is a spring balance of negligible mass and P is a smooth pulley fixed at the top of two smooth inclined planes. What is the reading of S when X is held stationary?

A. 5 N B. 10 N C. 15 N D. 20 N E. 30 N 6. A ball bounces up and down on the ground in

the vertical direction. Which of the following graphs best describes the variation of its acceleration a with time t?

A.

0 t

a

B.

0 t

a

C.

0 t

a

D.

0 t

a

E.

0 t

a

7. A man weighs an object with a spring balance

in a lift. Before the lift moves the scale reads 50 N. The lift goes down and then stops. The reading on the scale is

A. 50 N throughout the journey. B. more than 50 N when the lift starts, and

remains steady until it comes to rest. C. less than 50 N when the lift starts, and

remains steady until it comes to rest. D. more than 50 N as the lift starts, and less

than 50 N as it comes to rest. E. less than 50 N as the lift starts, and more

than 50 N as it comes to rest. 8.

30o

B

A

A smooth conical container rotates about the

axis AB as shown. A marble remains at rest relative to the container at a radial distance r

Page 97: AL Physics 1981-2004 MC

89 AL Physics/M.C./P.3

from the axis. If the velocity of the marble is v, then v² is equal to

A. gr sin 30º. B. gr tan 30º. C. gr / tan 30º. D. gr cos 30º. E. gr / cos 30º. 9.

30 cm

Diagram NOT to scale

An object performs simple harmonic motion.

A ticker-tape stuck to it records its positions at 0.020 s intervals for the first half cycle. The maximum speed of the object is

A. 0.71 m/s. B. 2.36 m/s. C. 2.83 m/s. D. 4.71 m/s. E. 9.42 m/s. 10. A particle of mass 0.2 kg moves with S.H.M.

of amplitude 0.05 m. If the total energy of the particle is 0.004 J, then its period of motion is

A. π/4 s. B. π/2 s. C. 3 π/4 s. D. π s. E. 2 π s. 11. Assuming the Earth to be a perfect sphere,

what would its angular velocity of rotation have to be for an object at the equator to be weightless (i.e. to give a spring balance reading of zero)?

(Radius of the Earth = 6.4 × 106 m.) A. 2.4 × 10-12 rad/s B. 1.6 × 10-6 rad/s C. 1.3 × 10-3 rad/s D. 8.0 × 102 rad/s E. 6.4 × 105 rad/s

12.

a

a

X Y

Earth

2 Two points X and Y are at distances a and 2a

from the centre of the Earth. The gravitational potential at X is -8 kJ/kg. When a 1 kg mass is taken from X to Y the work done on the mass is

A. -4 kJ. B. -2 kJ. C. +2 kJ. D. +4 kJ. E. +8 kJ. 13.

O

P

Q

hh

2

A ball rolls down an inclined plane. The ball is

first released from rest from P and then later from Q. Which of the following statements is/are correct?

(1) The ball takes twice as much time to roll

from Q to O as it does to roll from P and O.

(2) The acceleration of the ball at Q is twice as large as the acceleration at P.

(3) The ball has twice as much K.E. at O when rolling from Q as it does when rolling from P.

A. (1), (2) and (3) B. (1) and (2) only C. (2) and (3) only D. (1) only E. (3) only 14.

Page 98: AL Physics 1981-2004 MC

89 AL Physics/M.C./P.4

An air bubble is blown at the end of a capillary

tube dipping into a liquid of surface tension T. The excess pressure needed to blow the bubble

(1) is greatest at the beginning when one starts

to blow. (2) is least when the bubble is a hemisphere. (3) is directly proportional to T. A. (1), (2) and (3) B. (1) and (2) only C. (2) and (3) only D. (1) only E. (3) only 15.

A

C

Baxis of pipe

liquid flow

An incompressible viscous liquid flows along a

horizontal pipe of uniform cross-sectional area under streamline conditions. Which of the following statements is/are correct?

(1) The pressure at point A is greater than that

at point B. (2) The speed of liquid flow is the same at

points A and B. (3) The speed of liquid flow at C is less than

that at B. A. (1), (2) and (3) B. (1) and (2) only C. (2) and (3) only D. (1) only E. (3) only 16. Two vessels of equal volume both contain an

ideal gas and are connected by a tube of negligible volume. Initially both vessels are at temperature T0 and pressure P0. One vessel is maintained at T0, while the temperature of the

other is raised to T. The new pressure is then given by

A. P0. B. T P0 / T0. C. T0 P0 / T. D. 2 T P0 / (T + T0). E. P0 (T + T0) / 2 T0. 17. Two different gases A and B are contained in

two identical vessels. If the ratio of their molecular masses and absolute temperatures are respectively 8 : 1 and 2 : 1, the ratio of their r.m.s. molecular speeds will be

A. 1 : 4. B. 1 : 2. C. 1 : 1. D. 2 : 1. E. 4 : 1. 18. The internal energy of an ideal gas at

temperature T is 3 CT/2, where C is a constant. Given that R = the molar gas constant, NA = Avogadro constant, k = Boltzmann constant, then for an ideal gas containing N molecules, the constant C is equal to

A. NA R. B. NA k. C. NR / NA. D. Nk / NA. E. NR. 19. 1 mole of an ideal mono-atomic gas expands at

a constant pressure of 105 Pa from a volume of 0.02 m³ to a volume of 0.03 m³. The increase in internal energy of the gas is

A. 500 J. B. 667 J. C. 1 000 J. D. 1 500 J. E. 2 500 J. 20. A rotating disc, with a small hole near its edge

is illuminated by a lamp which flashes at a rate of 50 times per second. The hole is observed to be moving backwards slowly relative to the

Page 99: AL Physics 1981-2004 MC

89 AL Physics/M.C./P.5

actual direction of rotation of the disc. The disc is probably rotating at

A. 24 revolutions per second. B. 26 revolutions per second. C. 50 revolutions per second. D. 98 revolutions per second. E. 102 revolutions per second. 21. The coated lens of a camera appears purple in

daylight. According to the manufacturers, the coating has a refractive index of 1.25 and the material of the lens has a refractive index of 1.50. If the wavelength of yellow light in air is 520 nm, the approximate thickness of the coating is

A. 104 nm. B. 130 nm. C. 180 nm. D. 210 nm. E. 260 nm. 22. Light of wavelength λ is incident normally on a

diffraction grating with p lines per millimetre. the second-order diffraction maximum is at an angle θ from the central position. For a second grating with 3 p lines per millimetre illuminated normally by light of wavelength 5 λ /4, the angle between the first-order diffraction maximum and the central position is φ. Which of the following relations is correct ?

A. sin φ = (5 sin θ)/12 B. sin φ = sin (5 θ/12) C. sin φ = sin (15 θ/4) D. sin φ = (15 sin θ)/8 E. sin φ = sin (15 θ/8) 23. If the threshold of hearing is 10-12 W/m², a

sound level of one microwatt per m² is above threshold by

A. 6 dB. B. 11 dB. C. 60 dB. D. 110 dB. E. 120 dB. 24. A stationary radar source emits waves of

frequency f, and wavelength λ which are

reflected from an object moving towards the source at a speed u. The reflected waves reaching a receiver standing near to the radar source will have an apparent wavelength of

A. λ - 2u/f. B. λ - u/f. C. λ. D. λ + u/f. E. λ + 2u/f. 25.

MA B

+ _

Two large metal plates A, B are oppositely

charged and placed a small distance apart. A piece of metal M is placed centrally between the plates. Which of the following graphs shows the variation of electric field intensity E from A to B?

A.

A B

E

B.

A B

E

C.

A B

E

D.

Page 100: AL Physics 1981-2004 MC

89 AL Physics/M.C./P.6

A B

E

E.

A B

E

26. An isolated spherical conductor of radius r is

charged to a potential V. The total electrical energy stored is

A. V²/(8πε0 r). B. V²/(4πε0 r). C. V²/(2πε0 r). D. V² × (2πε0 r). E. V² × (4πε0 r). 27. A capacitor is charged to a p.d. of 100 V and

then connected across a resistor. After 1 s, the p.d. across the capacitor is 80 V. After another second, the p.d. across the capacitor will be

A. 40 V. B. 56 V. C. 60 V. D. 64 V. E. 70 V. 28.

100

V

A2 V K

Ω In the above circuit, V is a voltmeter of high

internal resistance and A is an ammeter of low internal resistance. What is the voltmeter reading when (a) switch K is open, (b) switch K is closed?

K open K closed A. 0 V 0 V B. 0 V 2 V C. 1 V 1 V D. 2 V 0 V E. 2 V 2 V 29.

BA

1

1

2

1

Ω

Ω

Ω

Ω

In the above circuit, the equivalent resistance

between A and B is A. 2/7 Ω. B. 2/5 Ω. C. 2/3 Ω. D. 2 Ω. E. 5 Ω. 30.

E E

X

Y

I

II

III

IV

A particle carrying a negative charge is free to

move in a uniform electric field E. If the particle starts with a certain velocity from point X, which of the paths shown could represent the route which the particle would follow from X to Y?

A. I and II only B. I and IV only C. II and III only D. III and IV only E. I, II, III and IV 31.

Page 101: AL Physics 1981-2004 MC

89 AL Physics/M.C./P.7

10 V

A BX

10 Ω In the above circuit, AB is a metre-wire of

resistance 10 Ω. When X is moved to the mid-point of AB, the p.d. across AX will be

A. 2.5 V. B. 4.0 V. C. 5.0 V. D. 6.0 V. E. 7.5 V. 32.

x

yr s

B

p

qI

A circular loop carrying a current I is placed in

a uniform magnetic field B in the xy plane as shown. If the loop is free to move, the magnetic forces will cause it to

A. rotate about the y-axis as indicated by p. B. rotate about the y-axis as indicated by q. C. rotate about the x-axis as indicated by r. D. rotate about the x-axis as indicated by s. E. remain stationary. 33. Which of the following statements about a d.c.

motor is FALSE? A. The back e.m.f. developed dissipates the

input energy in Joule heating. B. If the motor is stopped by some

obstruction, the current through the coil will increase.

C. The direction of rotation can be predicted by Fleming’s left hand rule.

D. A single-coil motor will not start when it is switched on, if the plane of the coil is perpendicular to the magnetic field.

E. An ideal d.c. motor develops a back e.m.f. equal to the applied e.m.f.

34.

L V

p qt

The circuit diagram shows an alternating

potential difference connected across an inductor L, and the graph shows the variation of the source voltage V with time t. Which of the following is/are correct?

(1) At time p, both the current through L and

the p.d. across it are zero. (2) The current through L and the p.d. across

it are 180º out of phase. (3) At time q, the current through L is at its

maximum value. A. (1), (2) and (3) B. (1) and (2) only C. (2) and (3) only D. (1) only E. (3) only 35. A length of wire is bent to form a circular coil

of one turn. The coil is placed in a uniform magnetic field with its plane normal to the direction of the field. The flux linkage through the coil is φ. The same length of wire is now bent to form a double loop of smaller radius. The flux linkage is now

A. φ/4. B. φ/2. C. φ. D. 2φ. E. 4φ. 36.

EK

A B

Page 102: AL Physics 1981-2004 MC

89 AL Physics/M.C./P.8

Two rectangular wire loops A and B are placed

in the same plane. Loop A includes a battery E and switch K, which is initially open. If K is suddenly closed, what is the direction of the induced current in loop B? Is the force between the two loops attractive or repulsive?

Direction of current Nature of force A. clockwise attractive B. clockwise repulsive C. anticlockwise attractive D. anticlockwise repulsive E. no current no net force 37.

A B

1 kΩ

200 Ω

Which of the following graphs best represents

the variation with time t of the current I through the segment AB in the above circuit?

A.

t

I

B.

t

I

C.

t

I

D.

t

I

E.

t

I

38.

6 V r.m.s.

An ideal inductor of reactance 1 Ω and an ideal

resistor of resistance 2 Ω are joined in series across the terminals of a 6 V r.m.s. a.c. source as shown. The power dissipated in the circuit is

A. 8 W. B. 12 W. C. 14.4 W. D. 16.1 W. E. 18 W. 39.

R

D

In the above circuit, D is an ideal diode, R is a

resistor of resistance 5 kΩ and the alternating e.m.f. is represented by the equation

E = 10 cos (5 π t), Where E is in volts and t is in seconds. The

power dissipated in R is A. zero. B. 2.5 mW. C. 5 mW.

Page 103: AL Physics 1981-2004 MC

89 AL Physics/M.C./P.9

D. 10 mW. E. 20 mW. 40.

6 VR

Q

P

5 V50 Hz

100 Ω 10 Ω

15 k Ω

Ib

Ic

A CRO is used to display the current transfer

characteristic curve of a transistor. How should points P, Q and R be connected to the X-input, Y-input and the earth terminal of the CRO?

X-input Y-input Earth A. P Q R B. P R Q C. Q P R D. Q R P E. R P Q 41. A gold-leaf electroscope is positively charged.

Which of the following objects when placed near the cap, will cause the leaf of the electroscope to fall?

(1) a lighted match (2) a strong α-source (3) an earthed metal plate A. (1), (2) and (3) B. (1) and (2) only C. (2) and (3) only D. (1) only E. (3) only 42. When a beam of white light passes through

iodine vapour, the spectrum of the emergent light shows dark lines. Which of the following statements is/are correct?

(1) The iodine vapour absorbs from the light

all frequencies except those which it emits. (2) The iodine vapour emits less energy than it

absorbs.

(3) The iodine vapour absorbs the same frequencies as it emits.

A. (1), (2) and (3) B. (1) and (2) only C. (2) and (3) only D. (1) only E. (3) only 43. A spherical drop of oil of mass m and radius r

carries an electrical charge q. It is placed in air of viscosity η between two horizontal parallel plates. When an electric field E is set up between the plates the drop is driven upwards, reaching a terminal velocity v. If the upthrust due to displaced air is negligible, which of the following is an expression for v?

A. mg / (6 π r η) B. 2mg / (9 π r η) C. (mg - qE)/ (6 π r η) D. (qE - mg) / (6 π r η) E. (qE + mg) / (6 π r η) 44. An atom emits light of wavelengths 122 nm

and 103 nm when one of its electrons returns to its ground state from its first and second excited states, respectively. The wavelength of light emitted when the electron passes from the second excited state to the first excited state is

A. 19 nm. B. 112 nm. C. 113 nm. D. 225 nm. E. 661 nm. 45. When light of wavelength 4.0 × 10-7 m is

incident on the surface of a metal, the kinetic energy of the electrons emitted has a maximum value of 3.0 × 10-19 J. What is the longest wavelength of light which would cause electrons to be emitted form the metal?

( Planck constant = 6.6 × 10-34 Js, speed of light in air = 3.0 × 108 m/s) A. 6.6 × 10-7 m B. 1.0 × 10-6 m C. 2.5 × 10-6 m D. 9.8 × 105 m E. 1.5 × 106 m

Page 104: AL Physics 1981-2004 MC

89 AL Physics/M.C./P.10

46.

S

R

T

A neutral particle decays in a short time into a

proton and a negative particle. The initial and subsequent paths of the particles in a magnetic field acing into the plane of the paper are shown above. Which of the following gives the respective paths for the neutral particle, the proton and the negative particle?

Neutral

particle The proton Negative

particle A. R S T B. R T S C. S R T D. S T R E. T R S

47. Which of the following statements is/are true

for beta particles originating from nuclear disintegrations?

(1) Beta particles travel at the speed of light. (2) Emitted beta particles from a nuclide have

a continuous energy distribution. (3) Beta decay is accompanied by the

emission of neutrinos. A. (1), (2) and (3) B. (1) and (2) only C. (2) and (3) only D. (1) only E. (3) only

48. A Geiger-Muller tube and a scaler record

approximately 50 counts per second due to a radioactive source. What is the shortest counting time required so that the percentage error of the counts will not exceed 1%?

A. 100 s B. 200 s C. 1 000 s D. 5 000 s E. 10 000 s 49. A helium atom, a hydrogen atom and a neutron

have masses of 4.003 u, 1.008 u and 1.009 u (unified atomic mass units) respectively. Assuming that hydrogen atoms and neutrons can fuse to form helium, the binding energy of a helium nucleus is

A. 0.0031 u. B. 0.031 u. C. 1.017 u. D. 2.014 u. E. 2.017 u. 50. If the kinetic energy of an electron is 10.0 eV,

what is the wavelength of the de Broglie wave associated with it?

( Planck constant = 6.63 × 10-34 Js, Charge of electron = 1.60 × 10-19 C, Mass of electron = 9.11 × 10-31 kg ) A. 4.1 × 10-16 m B. 3.9 × 10-10 m C. 5.5 × 10-10 m D. 2.3 × 1014 m E. 4.5 × 1014 m

- End of Paper -

Page 105: AL Physics 1981-2004 MC

89 AL Physics/M.C./P.11

Question No. Key 1. E 2. C 3. C 4. A 5. B 6. C 7. E 8. C 9. B 10. B 11. C 12. D 13. E 14. E 15. A 16. D 17. B 18. C 19. D 20. D 21. A 22. D 23. C 24. A 25. B

Question No. Key 26. D 27. D 28. D 29. A 30. A 31. B 32. D 33. A 34. E 35. B 36. D 37. A 38. C 39. C 40. D 41. A 42. E 43. D 44. E 45. B 46. D 47. C 48. B 49. B 50. B

Page 106: AL Physics 1981-2004 MC

90 AL Physics/M.C./P.1

1990 Hong Kong Advanced Level Examination AL Physics

Multiple Choice Question

1. The formula T² = 4 π² l/g is used to calculate the acceleration due to gravity g.

If the maximum percentage error of l = 2 % the maximum percentage error of T = 5%, then the maximum percentage error for g will

be A. 3%. B. 8%. C. 12%. D. 23%. E. 27%. 2. When a man is running due north, he feels that

the wind is blowing towards him from the east. What is the actual direction of the wind?

A. from the east B. from north-east C. from south-east D. from south-west E. from north-west 3. A uniform rope has a length l and a mass m. It

is held stationary with 4/5 of the length of the rope lying on a smooth horizontal table surface and with the remaining 1/5 hanging freely over the edge. The minimum work required to pull the whole rope onto the table surface is

A. 0.01 mgl. B. 0.02 mgl. C. 0.1 mgl. D. 0.2 mgl. E. mgl. 4.

d

h

M

u

θgun

A hunter aims his gun at a target which is at

rest at point M, and his gun makes an angle θ with the horizontal. Exactly as the gun is fired the target drops from M with zero initial velocity. If the bullet is to strike the target, the angle θ depends on

(1) u, the initial speed of the bullet. (2) h, the vertical height of the target above

the level of the gun. (3) d, the horizontal distance of the gun from

the target. A. (1), (2) and (3) B. (1) and (2) only C. (2) and (3) only D. (1) only E. (3) only 5.

vP

A wheel rolls horizontally along the ground

without slipping. The speed of the centre of mass of the wheel is v. The instantaneous speed of point P relative to the ground is

A. zero. B. v. C. v √2. D. 2 v. E. 4 v.

Page 107: AL Physics 1981-2004 MC

90 AL Physics/M.C./P.2

6. A spring of unstretched length 10.0 cm, has

one end fixed to the ceiling. A mass is suspended at the other end, and the extension so produced is 3.0 cm. When the mass is set to rotate in a horizontal circular path the length of the spring is 16.0 cm. The angle between the spring and the vertical is

A. 15º. B. 30º. C. 45º. D. 60º. E. 75º. 7. An object moves vertically with simple

harmonic motion just behind a wall. From the other side of the wall the object is visible in each cycle for 2.0 s and hidden behind the wall for 6.0 s. The maximum height reached by the object relative to the top of the wall is 0.30 m. The amplitude of the motion is

A. 0.18 m. B. 0.51 m. C. 0.60 m. D. 1.02 m. E. 1.20 m. 8. Two wires X and Y of the same length and of

the same elastic metal are each stretched to the same tension. The diameter of wire X is half that of wire Y. The ratio of the elastic potential energy stored in wire X to that stored in wire Y is

A. 1 : 1. B. 1 : 2. C. 1 : 4. D. 2 : 1. E. 4 : 1. 9. Two spherical soap bubbles of the same

material have radii 40 mm and 60 mm. They coalesce so that they share a common surface. If the radii of the bubbles remain unchanged, the radius of curvature of the common surface is

A. 40 mm. B. 50 mm. C. 60 mm.

D. 120 mm. E. 240 mm. 10. A piston of radius r and length l is pushed

through a cylinder covered with a thin layer of oil of viscosity η. The effective thickness of the oil layer is d. When the speed of the piston is u, the resistive force it experiences is:

A. 2πrlηu/d. B. 2πrlηud. C. πr2lηu/d. D. πr2lηud. E. 2πrlηu. 11. In fluid dynamics, which of the following

assumptions is/are used in deriving Bernoulli’s equation?

(1) The fluid undergoes streamline flow. (2) The fluid is compressible. (3) Viscous forces act on the fluid. A. (1), (2) and (3) B. (1) and (2) only C. (2) and (3) only D. (1) only E. (3) only 12. Which of the following will decrease when the

temperature increases? (1) the surface tension of a liquid (2) the viscosity of a liquid (3) the drift velocity of electrons in a metal A. (1), (2) and (3) B. (1) and (2) only C. (2) and (3) only D. (1) only E. (3) only 13. Container X holds pure hydrogen gas while

container Y holds pure oxygen gas. If the hydrogen molecules in X have the same r.m.s. speed as the oxygen molecules in Y, which of the following conclusions may be drawn?

A. The gas in X has a higher temperature than

Y. B. The gas in X has a higher pressure than Y.

Page 108: AL Physics 1981-2004 MC

90 AL Physics/M.C./P.3

C. The gas in Y has a higher temperature than X.

D. The gas in Y has a higher pressure than X. E. The gases in X and Y are at the same

temperature. 14. A fixed mass of an ideal gas is enclosed in a

cylinder by a piston. When the piston is pushed quickly into the cylinder, which of the following physical properties of the gas will increase?

(1) pressure (2) temperature (3) internal energy A. (1), (2) and (3) B. (1) and (2) only C. (2) and (3) only D. (1) only E. (3) only 15. A rubber band is of unstretched length l and

force constant k. When it is stretched to a length 2 l, the speed of transverse waves on it is v. What will be the speed of transverse waves when it is stretched to a length 3 l?

A. v B. v √2 C. 3v/2 D. v √3 E. 2v 16.

Pdeep

shallow

V

IV

IIIII

IQ

The figure shows wave crests moving in the

direction of the arrow towards the interface PQ between a shallow region and a deep region as shown in the figure above. Which of the lines shown may represent one of the wave crests in the deep region?

A. I B. II

C. III D. IV E. V 17.

A disc with a spot on it rotates at a constant

speed in a darkened room. A students shines a stroboscopic lamp on it. When the flashing rate is 8 flashes per second, the disc appears stationary with two spots on it as shown. When the flashing rate is reduced to 2 flashes per second, the disc would appear to be stationary with

A. no spot. B. one spot. C. two spots. D. four spots. E. eight spots. 18. Which of the following represent the

approximate noise levels (i) in a quiet school library? (ii) near the airport when an aircraft is taking

off and flying overhead? (i) (ii) A. 30 dB 60 dB B. 60 dB 90 dB C. 30 dB 90 dB D. 90 dB 60 dB E. 60 dB 30 dB 19. The intensity of a sound wave is proportional

to the square of the amplitude of the wave. If two waves of the same frequency are superimposed in phase, the total intensity is proportional to

A. the mean value of the intensities of the two

waves. B. the square of the sum of the two

amplitudes. C. the square of the mean value of the two

amplitudes.

Page 109: AL Physics 1981-2004 MC

90 AL Physics/M.C./P.4

D. the square of the difference of the two amplitudes.

E. the sum of the intensities of the two waves. 20. In a simple astronomical telescope, under

normal adjustment, which of the following statements is/are correct?

(1) The first image is formed at the focal

plane of the objective. (2) The first image is real and inverted. (3) The focal length of the objective is longer

than that of the eyepiece. A. (1), (2) and (3) B. (1) and (2) only C. (2) and (3) only D. (1) only E. (3) only 21.

30o

A boat travels in shallow water, in which

waves of all wavelengths travel at a speed of 4.0 m/s. What is the speed of the boat if the bow wave generated by the boat has an apex angle of 30º?

A. 2.0 m/s B. 2.3 m/s C. 4.0 m/s D. 6.9 m/s E. 8.0 m/s 22.

The figure shows two pulses travelling to the

right along a rope. The right-hand end of the rope is fixed to a wall. The following figures represent predicted positions of the pulses at later times (where appropriate, the arrows indicate the direction of a pulse). Which of the following could NOT arise from the initial given condition?

(1)

(2)

(3)

A. (1), (2) and (3) B. (1) and (2) only C. (2) and (3) only D. (1) only E. (3) only 23.

G

L1

L2

A

B 2 loudspeakers L1 and L2 are connected to a

signal generator G. A microphone is moved along the line AB and the variation in intensity is noted. Which of the following statements concerning the above arrangement is/are correct?

(1) If the separation of the 2 loudspeakers is

less than the wavelength of the sound emitted, no alternation of maxima and minima can be detected along AB.

(2) If the frequency of the sound waves emitted is increased, the separation between adjacent maxima along AB will be increased.

(3) If the 2 loudspeakers are vibrating in antiphase, no alternation of maxima and minima will be detected along AB.

A. (1), (2) and (3) B. (1) and (2) only

Page 110: AL Physics 1981-2004 MC

90 AL Physics/M.C./P.5

C. (2) and (3) only D. (1) only E. (3) only 24. Sound waves of frequency f are emitted by a

source S. When S is moved with speed u (relative to the ground) towards a stationary observer O, a rise in pitch of ∆f is detected. Which of the following statements is/are correct?

(1) The speed of sound waves relative to the

observer is unaffected by the motion of S. (2) If both S and O move in the same direction

with speed u, no rise in pitch will be detected.

(3) If S is at rest with O moving towards it at speed u, the rise in pitch will also be ∆f.

A. (1), (2) and (3) B. (1) and (2) only C. (2) and (3) only D. (1) only E. (3) only 25. When parallel light is incident at the Brewster

angle in air on the surface of a glass block, which of the following statements is/are correct?

(1) The refracted light is plane-polarised. (2) The reflected light is plane-polarised. (3) The reflected ray and the refracted ray are

at right angles to each other. A. (1), (2) and (3) B. (1) and (2) only C. (2) and (3) only D. (1) only E. (3) only 26. White light diffracted by a single slit falls on a

white screen. Which of the following statements is/are correct?

(1) The centre of the diffraction pattern is

white. (2) The first minimum is closer to the centre

for red light than for blue light. (3) The central band width is increased as the

slit width is increased. A. (1), (2) and (3)

B. (1) and (2) only C. (2) and (3) only D. (1) only E. (3) only 27.

+ _

A light conducting sphere is hanged from a

long insulating thread between oppositely charged metal plates connected to a high voltage supply. If the sphere is given a positive charge, it will

A. move to the positive plate and stick to it. B. move to the negative plate and stick to it. C. remain still. D. oscillate, touching each plate in turn,

beginning with the negative plate. E. oscillate, touching each plate in turn,

beginning with the positive plate. 28.

small hole

+Q

The arrangement above shows two concentric

hollow metal spheres of inner radius b and outer radius a. A charge +Q is given to the inner sphere and the outer sphere is earthed. What is the work done in bringing a small positive charge q from infinity to the surface of the inner sphere?

A. zero B. qQ/(4πε0a) C. qQ/(4πε0b) D. qQ/[4πε0(a - b)] E. qQ(1/b - 1/a)/(4πε0)

Page 111: AL Physics 1981-2004 MC

90 AL Physics/M.C./P.6

29.

b

b

bb

P

P

2

1

A uniformly charged wire has the form of a

circular loop of radius b. P1 and P2 are two points on the axis of the loop. P1 is at a distance b from the loop centre and P2 is at a distance 2 b from the loop centre. At P1, P2, the potentials are V1, V2 respectively. V2/V1 is equal to

A. 1/3. B. 2/5. C. 1/2. D. √2/√5. E. 4π. 30.

A parallel-plate capacitor is connected to a

battery as shown. What will happen if the separation of the plates is increased?

capacitance voltage charge A. decreases decreases decreases B. decreases unchanged decreases C. decreases decreases increases D. increases unchanged decreases E. increases unchanged increases 31.

12 V

B1

B2

r.m.s.a.c.

Bulbs B1, B2 are connected to an a.c. supply

and both bulbs are lit. If the a.c. supply is replaced by a 12 V d.c. supply with negligible internal resistance, what will happen to the brightness of each bulb?

B1 B2 A. unchanged increases B. unchanged decreases C. decreases increases D. decreases decreases E. increases unchanged 32.

1A

3 V, 1 Ω

YX

The figure above shows part of a circuit which

carries a current of 1 A from X to Y through a cell of e.m.f. 3 V and internal resistance 1 Ω. The potential difference between X and Y is

A. 0 V. B. 1 V. C. 2 V. D. 3 V. E. 4 V. 33.

I

6 V

3 3 3Ω Ω Ω

In the above circuit, the battery has negligible

internal resistance. The current I is A. 2/3 A. B. 1 A.

Page 112: AL Physics 1981-2004 MC

90 AL Physics/M.C./P.7

C. 4/3 A. D. 2 A. E. 6 A. 34. Which of the following circuits is best used for

the measurement of a low resistance R? (Polarities of meter terminals are given in the

diagram.) A.

V

A+ _

+ _

R

B.

V

A+_

+ _

R

C.

V

A+ _

+_

R

D.

V

A+_

+_

R

E.

V

A+ _

+ _

R

35.

SR

G

A BC

E

In the potentiometer circuit shown, S is a

source whose e.m.f. is to be measured. R is a resistor to protect the galvanometer G. In the experiment, a student finds that G deflects in the same direction for any position of C, giving maximum deflection at B and minimum deflection (not zero) at A. Which of the following is the possible reason?

A. S is connected with wrong polarity. B. The resistance of R is too high. C. There is contact resistance at A and B. D. The e.m.f. of the driver cell E is too low. E. The driver cell E has a large internal

resistance. 36. A d.c. motor is connected to a 12 V battery.

Which of the following statements is correct? (1) As the motor gathers speed, the current

through the motor increases. (2) As the motor gathers speed, the torque

acting on the rotating coil increases. (3) If the efficiency is 75% at steady state, the

back e.m.f. is then 9 V. A. (1), (2) and (3) B. (1) and (2) only C. (2) and (3) only D. (1) only E. (3) only 37. A cell, a switch, a resistor and an inductor are

connected in series to form a circuit. At time t = 0 the switch is closed. Which of the following best represents the variation of the current I in the circuit with time t?

A.

t

I

0 B.

Page 113: AL Physics 1981-2004 MC

90 AL Physics/M.C./P.8

t

I

0 C.

t

I

0 D.

t

I

0 E.

t

I

0 38. The self-inductance of an air-core solenoid can

be increased by (1) increasing the current through it. (2) increasing the frequency of the supply. (3) placing a ferro-magnetic material inside

the solenoid. A. (1), (2) and (3) B. (1) and (2) only C. (2) and (3) only D. (1) only E. (3) only 39.

0

+5 V

10 k Ω

1 kΩ

The circuit above shows an NPN transistor and two resistors 10 kΩ and 1 kΩ connected to a 5 V d.c. supply. The current gain of the transistor is 100. What is the value of the collector current?

A. 5 µA B. 500 µA C. 5 mA D. 50 mA E. 500 mA 40. Electrical power is transmitted at high voltage

over long distances because (1) a larger amount of energy can be

transmitted per unit charge. (2) a smaller current is required for a fixed

amount of power delivered. (3) less power is lost through heating during

transmission. A. (1), (2) and (3) B. (1) and (2) only C. (2) and (3) only D. (1) only E. (3) only 41.

d

d

θ

A beam of X-rays of wavelength λ is incident

upon a crystal containing atoms arranged in planes of separation d. The beam makes an angle θ with the planes as shown, an intense diffracted X-ray beam is produced. Which of the following is equal to a small integer?

A. d sin θ / λ B. λ / (d sin θ) C. 2 λ sin θ / d D. 2 d sin θ / λ E. λ / (2 d sin θ) 42.

Page 114: AL Physics 1981-2004 MC

90 AL Physics/M.C./P.9

a

a21_

Two sinusoidal alternating potential

differences Vx and Vy of the same frequency are applied respectively to the x and y plates of a C.R.O with the time base switched off. The trace on the screen is an ellipse as shown above. What is the phase difference between Vx and Vy?

A. 30º B. 45º C. 60º D. 120º E. 150º 43.

00

VV

timetime

1 2

Potential difference V1 and V2 are applied to

the X-plates and Y-plates of a C.R.O. respectively. The trace on the screen is

A.

B.

C.

D.

E.

44. A high constant potential difference is applied

across the electrodes of a discharge tube containing air. A vacuum pump gradually exhausts air from the tube. Which of the following will possibly happen?

(1) Positive ions move towards the anode. (2) Some air molecules are ionised. (3) X-rays are emitted from the anode. A. (1), (2) and (3) B. (1) and (2) only C. (2) and (3) only D. (1) only E. (3) only 45.

QP S S1 2

A mixed stream of ions (of different charges,

polarities and speeds) travels along PQ and passes through a narrow slit S1. In the region between S1 and S2, an electric field E and a magnetic field of flux density B are directed normally to each other. The E-field acts vertically downward and the B-field acts into the plane of the paper. Ions that are undeflected and emerge from slit S2 must have the same

Page 115: AL Physics 1981-2004 MC

90 AL Physics/M.C./P.10

A. polarity. B. speed. C. quantity of charge. D. mass. E. charge to mass ratio. 46.

hr

A

Bball

3-dimensionalmodel 'hill'(symmetric inhorizontal plane)

The figure shows the set-up for a gravitational

analogue simulation of alpha-particle scattering by a thin metal sheet. What should be the relationship between the height of the ‘hill’ h and the radius of horizontal cross-sections r?

A. hr = constant B. hr² = constant C. h²r = constant D. h²r³ = constant E. h³r² = constant 47. The photoelectric effect occurs when

monochromatic light falls upon a metal surface in a photocell. What happens when the light intensity increases?

A. More electrons are emitted with

unchanged speed. B. More electrons are emitted with increased

speed. C. The same number of electrons is emitted

with increased speed. D. More photons are emitted from the

surface. E. Photons of greater energy are emitted from

the surface. 48. A radioactive source is placed in front of a GM

tube connected to a counter. Various absorbers are placed between the source and the GM tube and the count-rate recorded. The following results were obtained:

Absorber Counts per minute no absorber 711 a sheet of paper 508 5 mm thick of aluminium sheet 493 25 mm of thick lead block 218 It can be deduced from these results that the

radiation(s) emitted by the source is/are A. α and γ rays only B. β and γ rays only C. α rays only D. β rays only E. γ rays only 49.

4

3

E

E

E

The diagram shows the energy levels of a

certain atom. When an electron changes energy from 4E to E, a photon of wavelength λ is emitted. Which of the following wavelengths of photons could be produced by other transitions between the energy levels shown?

A. λ/3 and 2 λ/3 B. λ/3 and 3 λ C. 2 λ/3 and 3 λ/2 D. 2 λ/3 and 3 λ E. 3 λ/2 and 3 λ 50. In a collision between an electron and an atom

which leads to excitation of the atom without ionization, which of the following is/are correct?

(1) An orbital electron escapes from the

nucleus. (2) An orbital electron acquires energy. (3) The energy transferred appears later as

electromagnetic radiation. A. (1), (2) and (3) B. (1) and (2) only C. (2) and (3) only D. (1) only

Page 116: AL Physics 1981-2004 MC

90 AL Physics/M.C./P.11

E. (3) only

- End of Paper - Question No. Key 1. C 2. C 3. B 4. C 5. C 6. D 7. D 8. E 9. D 10. A 11. D 12. A 13. C 14. A 15. D 16. D 17. B 18. C 19. B 20. A 21. E 22. E 23. D 24. B 25. C

Question No. Key 26. D 27. D 28. E 29. D 30. B 31. A 32. E 33. E 34. A 35. A 36. E 37. E 38. E 39. C 40. A 41. D 42. A 43. B 44. C 45. B 46. A 47. A 48. A 49. E 50. C

Page 117: AL Physics 1981-2004 MC

91 AL Physics/M.C./P.1

1991 Hong Kong Advanced Level Examination AL Physics

Multiple Choice Question

1. In which of the following situations is the magnitude of the normal reaction of the supporting surface, R, equal to the weight of the body, mg?

(1) At rest on a rough inclined plane. (2) On the floor of a spacecraft in circular

orbit around the earth. (3) On the floor of a lift moving upwards with

uniform velocity. A. (1), (2) and (3) B. (1) and (2) only C. (2) and (3) only D. (1) only E. (3) only 2. An object of mass m slides with constant

acceleration a down a plane making an angle θ with the horizontal. The frictional force acting on the object is

A. mg - ma B. mg - ma sin θ C. mg sin θ - ma D. (mg - ma) sin θ E. (mg + ma) sin θ 3.

m

m

v

v

θ

θ

A gas atom of mass m moving with a uniform

speed v makes an elastic collision with the wall of the container as shown in the diagram. What is the magnitude of the change in the momentum of the gas atom?

A. 2mv B. mv sin θ C. mv cos θ D. 2mv sin θ E. 2mv cos θ 4. A spacecraft of mass 4.0 × 104 kg was

travelling on its way to the moon with the rocket motors shut down. At the instant when it was travelling at 1500 m/s, the rocket motors were turned on for 5 seconds to make a course correction. If the rocket gave a thrust of 1.0 × 105 N at right angles to the direction of travel, through what angle would the flight path of the spacecraft be turned?

A. 1.6 × 10-3 rad B. 5.7 × 10-3 rad C. 8.3 × 10-3 rad D. 1.6 × 10-2 rad E. 8.3 × 10-2 rad 5. A point mass is attached to the lower end of a

light spring fixed at the upper end. The mass is made to oscillate vertically. If the potential energy of the system is taken to be zero when the mass is at its equilibrium position, the speed of the mass at the equilibrium position is directly proportional to the square root of

(1) the amplitude of oscillation. (2) the total energy of the system. (3) the maximum potential energy of the

system. A. (1), (2) and (3) B. (1) and (2) only C. (2) and (3) only D. (1) only E. (3) only 6.

Page 118: AL Physics 1981-2004 MC

91 AL Physics/M.C./P.2

A

B

H

Terminalplatform

20 m30 m

The diagram shows part of the route of a roller-

coaster in an amusement park. The cart descends from H, completes a circular loop A and moves to B. If the cart of passengers is to complete the central circular track safely, what is the minimum velocity of the cart at the bottom of the circular track A? (Assume that there is no friction between the cart and the track.)

A. 10 m/s B. 20 m/s C. 22.4 m/s D. 24.5 m/s E. 30 m/s 7. A piece of iron is suspended from a vertical

spring. The iron (but not the spring) is immersed in a jar of water, and oscillates with period T0. A vertical sinusoidal force of variable period T is now applied to the iron, using an electromagnet. Which one of the following statements is NOT correct?

A. When the electromagnetic is switched off,

the period of the oscillations changes from T to T0.

B. The amplitude of the oscillations increases greatly when T is brought close to T0.

C. For any value of T, the water temperature rises, due to energy transferred from the electromagnet.

D. If the electromagnetic is switched off, the water gains energy as the amplitude of the oscillations decreases.

E. For T not close to T0, the forced oscillations decrease slowly in amplitude due to damping.

8.

Load

P

S 1 S 2X

A trolley attached to two fixed supports S1 and

S2 by identical springs is displaced from the equilibrium position along the direction X and set into oscillation. A load is dropped onto and is retained by the trolley when it passes through its equilibrium position P. Which of the following statements is/are correct?

(1) Linear momentum in the horizontal

direction is conserved just before and after the load lands on the trolley.

(2) The amplitude of oscillation decreases after the landing of the load.

(3) The period increases after the landing of the load.

A. (1), (2) and (3) B. (1) and (2) only C. (2) and (3) only D. (1) only E. (3) only 9. The turntable of a gramophone rotates at a

steady angular speed ω. A record is dropped from rest on the turntable. Initially, the record slips but it undergoes uniform angular acceleration and eventually moves with the same angular speed as the turntable. The turntable turns through an angle θ while the record is slipping on its surface. Find the angular acceleration of the record and the angle which it turns through before it attains the steady speed of the turntable.

Angular acceleration Angle A. ω²/(2θ) θ/2 B. ω²/(2θ) θ C. ω²/(2θ) 2θ D. ω²/θ θ/2 E. ω²/θ θ 10. Which of the following properties of molecules

of an ideal gas is/are the same on the moon as on the earth, if the temperature and volume of the gas are unchanged?

Page 119: AL Physics 1981-2004 MC

91 AL Physics/M.C./P.3

(1) The average momentum change when a

molecule of the gas rebounds from a wall of the container.

(2) The average kinetic energy of a molecule of the gas.

(3) The weight of a molecule of the gas. A. (1), (2) and (3) B. (1) and (2) only C. (2) and (3) only D. (1) only E. (3) only 11. The breaking stress of a steel wire is 5.0 × 108

N/m². If the steel wire is replaced by a similar piece which is twice as long, which of the following statements is/are true?

(1) The extension when the longer wire breaks

is the same as for the shorter wire. (2) The work done in stretching the longer

wire to the breaking point is the same as for the shorter wire.

(3) The stress needed to break the longer wire is 5.0 × 108 N/m².

A. (1), (2) and (3) B. (1) and (2) only C. (2) and (3) only D. (1) only E. (3) only 12.

Strain

Stress

Contracting

Stretching

A suspended fibre was stretched by an

increasing load attached to the bottom end. Then it was allowed to contract by slowly reducing the load. A stress-strain graph was obtained as shown. Which one of the following conclusions may be deduced from the graph?

(1) All the work done in stretching the fibre is converted into potential energy.

(2) More work is done in stretching than is recovered in contracting.

(3) The temperature of the fibre rises after it has been stretched and allowed to contract for a few times.

A. (1), (2) and (3) B. (1) and (2) only C. (2) and (3) only D. (1) only E. (3) only 13. A glass plate X is placed at an angle to a liquid

so that the surface S appears to be a plane at Y as shown.

liquid

S

X

Y

Which of the following statements is/are

correct? (1) The angle of contact between the liquid

and the glass is greater than 90º. (2) The cohesive force between two molecules

of liquid is greater than the adhesive force between a molecule of the liquid and a molecule of the glass.

(3) When spilt on glass, the liquid would spread over the glass.

A. (1), (2) and (3) B. (1) and (2) only C. (2) and (3) only D. (1) only E. (3) only 14. The graph shows how the potential energy of a

pair of ions varies with the distance between them.

Page 120: AL Physics 1981-2004 MC

91 AL Physics/M.C./P.4

BA

C

0

potential energy

+

_

distancebetween ions

Which of the following arguments about the

points A, B and C marked on the graph is/are correct?

(1) From A to B the potential energy falls with

distance, so here the net force between the ions must be pushing them apart.

(2) At C the potential energy curve is a minimum, so a supply of energy is needed either to increase or decrease the distance between ions.

(3) At B, the potential energy is zero, so here any repulsive force between the ions must be zero.

A. (1), (2) and (3) B. (1) and (2) only C. (2) and (3) only D. (1) only E. (3) only 15.

x

x + ∆x∆x

In an idealised atomic model of the material of

a wire, each atom is in equilibrium at a distance x from its nearest neighbours, both in its own layer and in the layer above or below. There are n atoms per unit area within each layer. If the force required to increase the separation between two atoms from x to (x + ∆x) is (k∆x), what is the longitudinal stress in the wire?

A. ∆x/x B. k ∆x

C. nk ∆x/x² D. nk ∆x E. k/x 16.

lx

T

The diagram shows a microscope slide of

length l partially immersed in water. As the level of water is gradually lowered, which of the following graphs represents the variation of the tension T in the thread with x?

A.

l x

T

B.

l x

T

C.

l x

T

D.

l x

T

E.

Page 121: AL Physics 1981-2004 MC

91 AL Physics/M.C./P.5

l x

T

17. A fixed mass of ideal gas at S.T.P. occupies a

volume of 2 m³. The gas is heated and allowed to expand to a final volume of 4 m³ with its pressure doubled. The root mean square speed of the gas molecules is

A. reduced to one quarter of its value. B. halved. C. unchanged. D. doubled. E. increased four times. 18. An ideal gas is taken through the series of

changes shown.

pressureP / 10 5Pa

volume V / m3

1 2 3 40

1

2 A B

CD

Which of the following statements is/are

correct? (1) No net work is done by the gas in

completing one cycle. (2) The work done by the gas during the

change AB is equal to that done on the gas during the change CD.

(3) The gas has the same temperature at B and at C.

A. (1), (2) and (3) B. (1) and (2) only C. (2) and (3) only D. (1) only E. (3) only

19. Which of the following statements about an experiment showing the Brownian motion with smoke particles in air is/are correct?

(1) The motion is caused by collisions

between air molecules and smoke particles.

(2) The experiment makes it possible to see the motion of the air molecules.

(3) The motion is irregular because air is a mixture of gases, and the molecules have different masses.

A. (1), (2) and (3) B. (1) and (2) only C. (2) and (3) only D. (1) only E. (3) only 20. Orange light of wavelength 600 nm (in air) is

incident normally from air onto a liquid film whose refractive index is 1.25. For what minimum value of film thickness will the greatest amount of light be transmitted through the film?

A. 120 nm B. 180 nm C. 240 nm D. 300 nm E. 480 nm 21. In experiment with optical diffraction gratings,

the grating spacing

wavelength of light is of the order of

A. 10-6 B. 10-3 C. 1 D. 103 E. 106 22. In an arrangement for viewing Newton’s rings,

if the lens which rests on a glass plate were moved slowly upwards by one wavelength (of the viewing light), which of the following would be observed?

A. The central spot becomes bright. B. The rings disappear. C. The rings move towards the centre. D. The rings move out from the centre.

Page 122: AL Physics 1981-2004 MC

91 AL Physics/M.C./P.6

E. The rings are no longer concentric. 23.

observerA

P

lightsource

Light from an unpolarised source is allowed to

fall on a piece of polaroid P and then on a second polaroid A. In the position shown in the diagram, the intensity of the light emerging from polaroid A is a maximum. As A is slowly rotated, the intensity of light emerging is reduced to half the maximum value at angle θ1 and to a minimum at angle θ2.

Which of the following gives the correct values

of angles θ1 and θ2? θ1 θ2 A. 30º 90º B. 45º 90º C. 45º 180º D. 60º 90º E. 60º 180º 24. When a diffraction grating is replaced by

another with more lines per mm, which of the following quantities is/are increased?

(1) the angle of diffraction for every spectral

line (2) the angle separation of red and blue lines

in the first order spectrum (3) the number of orders which can be

observed A. (1), (2) and (3) B. (1) and (2) only C. (2) and (3) only D. (1) only E. (3) only 25.

M

N

W1

W2

Two trains of travelling waves W1 and W2, of

the same amplitude, wavelength and speed move in opposite directions. The period of both of the waves is T and their amplitude is A. At time t = t1, the waves W1 and W2 are as shown in the figure above. Which of the following statements is/are correct?

(1) The wavelength of the waves is four times

the distance MN. (2) At time t = t1 + T/4, the displacement at

point M is 2A. (3) Point N is a displacement node. A. (1), (2) and (3) B. (1) and (2) only C. (2) and (3) only D. (1) only E. (3) only 26. Ultrasonic waves are used instead of audible

sound waves to measure the depth of the sea because

(1) ultrasonic waves will not be interfered

with by other sound waves in the water. (2) ultrasonic waves can penetrate a greater

depth of water with less spreading-out of waves.

(3) small-sized objects can also be located. A. (1), (2) and (3) B. (1) and (2) only C. (2) and (3) only D. (1) only E. (3) only 27.

Q P

Page 123: AL Physics 1981-2004 MC

91 AL Physics/M.C./P.7

A stationary sound wave vibrating in its fundamental mode is set up in a tube closed at one end. P and Q are two points at the end and the middle of the tube along its axis. Neglecting end-correction, what is the phase difference between the vibrations of the air molecules at points P and Q?

A. 0 B. ¼ π radian C. ½ π radian D. π radian E. 1 ¼ π radians 28. A simple two-convex lens refracting telescope

has a magnifying power of 12.5 when the telescope is in normal adjustment. The focal length of the objective is 0.75 m. The separation between the objective and the eye piece is

A. 0.06 m B. 0.69 m C. 0.81 m D. 1.35 m E. 16.7 m 29. A transistor radio produces a sound of intensity

level 40 dB at a point 5 m away from it. What is the intensity level in decibels 20 m from the radio, which may be regarded as a point source?

A. 10 dB B. 28 dB C. 30 dB D. 34 dB E. 40 dB 30. The f-number of a certain camera is decreased

without changing any other camera controls. How would the image size, image brightness and depth of field be affected?

Image Image Depth size brightness of field A. decreased decreased increased B. decreased increased decreased C. increased increased decreased D. unaffected increased decreased E. unaffected decreased increased

31. Which of the graphs below best represents the

variation of electrical potential V with distance r from the centre of a charged hollow metal sphere of external radius a?

A.

ra0

V

B.

ra0

V

C.

ra0

V

D.

ra0

V

E.

ra0

V

32.

Page 124: AL Physics 1981-2004 MC

91 AL Physics/M.C./P.8

S

12 V

5 Ω 5 Ω

L 1 L 2 In the circuit shown, filament lamps L1 and L2

are identical and are seen to be marked ‘6V 18 W’. Which of the following statements is/are correct?

(1) When switch S is closed, lamp L1 and

Lamp L2 glow with the same brightness. (2) When switch S is closed, lamp L2 glows

with its normal brightness. (3) When switch S is opened, lamp L1 glows

with its normal brightness. A. (1), (2) and (3) B. (1) and (2) only C. (2) and (3) only D. (1) only E. (3) only 33.

C C CC

C

C

d.c. source d.c. source

Circuit A Circuit B Three identical capacitors are connected to the

same d.c. source in series (circuit A) and then in parallel (circuit B). The magnitude of the ratio of the total energies stored by the capacitors in circuit A to circuit B is

A. 1/9 B. 1/3 C. 1 D. 3 E. 9

34. The magnetic field due to a steady current in

the long air-core solenoid (uniformly wound) shown below is 8 × 10-3 T at X and 1 × 10-3 T at Y.

L L

X Y

If an identical solenoid is connected to the end

of the first so as to extend it to as far as Y, and the same current as before is passed through the complete solenoid, the magnetic field at Y will then be

A. 7 × 10-3 T B. 8 × 10-3 T C. 9 × 10-3 T D. 16 × 10-3 T E. 17 × 10-3 T 35.

Q

R

PZ

uniform magneticfield into paper

In the above figure, a copper disc rotates

uniformly between the pole-pieces of a powerful magnet (not shown in figure) in a clockwise direction. P and Q are metallic brushes making contact with the axle and the edge of the disc respectively. Which of the following statements is correct?

A. No current flows through R, because there

is no flux change through the disc. B. No current flows through R because P and

Q are at the same horizontal level. C. An alternating current flows through R. D. A steady current flows from P to Q

through R. E. A steady current flows from Q to P

through R.

Page 125: AL Physics 1981-2004 MC

91 AL Physics/M.C./P.9

36. An additional mechanical load is applied to a rotating motor. Which of the following is/are true?

(1) The motor will decrease in speed. (2) The e.m.f. induced in the rotating coil will

decrease. (3) More power will be drawn from the power

supply. A. (1), (2) and (3) B. (1) and (2) only C. (2) and (3) only D. (1) only E. (3) only 37. It is desired to re-design a moving coil

galvanometer so as to make it four times as sensitive. Which of the following would alone achieve the desired result?

(1) Increasing the magnetic flux density of the

permanent magnet to twice its value and doubling the cross-sectional area of the coil.

(2) Providing the coil with a shunt so that only a quarter of the input current flows through the coil itself.

(3) Changing the suspension characteristics so that four times as great a couple is needed to cause one radian twist.

A. (1), (2) and (3) B. (1) and (2) only C. (2) and (3) only D. (1) only E. (3) only 38.

time

voltage

Vp

voltage

time

Vp

(a)

(b) A signal generator produces either (a) a

sinusoidal wave or (b) a square wave with the same peak value of voltage as shown in the diagram. For a pure resistive load, the ratio of the mean power for (a) compared with (b) is

A. 1/√2 B. √2 C. 1/2 D. 2 E. 2√2 39.

10 kHzf

impedance Z

10 Ω

signalgenerator

L 1 Fµ

A coil and a capacitor are joined in series to a

signal generator as shown in the diagram. As the frequency f of the signal generator is increased, a graph of the total impedance Z of the circuit is plotted against f. If the capacitance of C is 1 µ F, what is the inductance of the coil?

A. 0.25 mH B. 1.6 mH C. 2.5 H D. 10 H E. 16 H

Page 126: AL Physics 1981-2004 MC

91 AL Physics/M.C./P.10

40. 6 V r.m.s.

R = 2 ΩX L = 1 Ω An inductor of reactance 1 Ω and a resistor of

resistance 2 Ω are joined in series to the terminals of a 6 V (r.m.s.) a.c. source as shown. What is the power dissipated in the circuit?

A. 8 W B. 12 W C. 14.4 W D. 16.1 W E. 18 W 41.

A parallel-plate capacitor is connected to a

battery as shown. What will happen if the separation of the plates is increased?

capacitance voltage charge A. decreases decreases decreases B. decreases unchanged decreases C. decreases decreases increases D. increases unchanged decreases E. increases unchanged increases

42. A.C. voltages of frequencies f1 and f2 were

applied to the x-plates and y-plates of a cathode ray oscilloscope, respectively. The resulting observed trace is:

x

y

If the frequency of f1 is 100 Hz, what is the frequency of f2?

A. 25 Hz B. 50 Hz C. 100 Hz D. 200 Hz E. 400 Hz 43.

0 V

+6 V

VoutVin

15 kΩ

2.2 k Ω

An n-p-n transistor is used in the above circuit

as a pulse shaper or a squarer. When a sinusoidal voltage whose magnitude varies between +2 V and -2 V is applied to the input, what will be the output voltage?

A.

0

6 V

time

Vout

B.

0

-6 V

time

Vout

C.

3 V

time

Vout

-3 V

0

D.

Page 127: AL Physics 1981-2004 MC

91 AL Physics/M.C./P.11

time

Vout

0

2 V

E.

time

Vout

-2 V

0

2 V

44. A beam of monochromatic light falls on a

surface. If the frequency of the light is doubled but the intensity remains unchanged, which of the following statements is/are correct?

(1) The photon energy is doubled. (2) The momentum of a photon is doubled. (3) The number of photons falling on the

surface per second is halved. A. (1), (2) and (3) B. (1) and (2) only C. (2) and (3) only D. (1) only E. (3) only 45. In an experiment on the photoelectric effect, a

beam of monochromatic light is directed onto a metal plate to liberate electrons. Which of the following statements is true?

A. The velocity of the fastest electrons is

directly proportional to the frequency of the incident light.

B. The velocity of the fastest electrons is directly proportional to the intensity of the incident light.

C. The kinetic energy of the fastest electrons is directly proportional to the frequency of the incident light.

D. The velocity of the fastest electrons is independent of the intensity of the incident light.

E. The velocity of the fastest electrons is independent of the type of metal.

46. The minimum wavelength of the X-rays emitted from a hot cathode X-ray tube is controlled by

(1) the cathode temperature. (2) the nature of the target. (3) the anode-cathode voltage. A. (1), (2) and (3) B. (1) and (2) only C. (2) and (3) only D. (1) only E. (3) only 47. Electron transitions occur in an atom resulting

in the emission of the following light wavelengths:

from level C to level A: 600 nm from level B to level A: 500 nm Which of the following statements is/are

correct? (1) Level A has a lower energy than both

levels B and C. (2) Level C has a higher energy than level B. (3) The wavelength of light emitted for the

transition between C and B is 100 nm. A. (1), (2) and (3) B. (1) and (2) only C. (2) and (3) only D. (1) only E. (3) only 48. The graph below shows how the count rate A

of a radioactive source as measured by a Geiger-counter varies with time t:

10 20 30

1

2

3

ln A

time t /s

The relationship between A and t is A. A e t= −2 5 10. B. A e t= 12 10

Page 128: AL Physics 1981-2004 MC

91 AL Physics/M.C./P.12

C. A e t= −2 5 0 1. . D. A e t= −12 0 1. E. A e t= 12 0 1. (Assume ln 12 = 2.5) 49. Which of the following statements concerning

a moderator in a nuclear fission reactor is/are correct?

(1) It is used to slow down the neutrons

released during fissions. (2) It is used to absorb excess neutrons. (3) Boron steel is a suitable material. A. (1), (2) and (3) B. (1) and (2) only C. (2) and (3) only

D. (1) only E. (3) only 50. Which of the following illustrates atomic

resonance? (1) Electron diffraction (2) Photoelectric emission (3) Fraunhofer dark lines in the sun’s

spectrum A. (1), (2) and (3) B. (1) and (2) only C. (2) and (3) only D. (1) only E. (3) only

- End of Paper -

Page 129: AL Physics 1981-2004 MC

91 AL Physics/M.C./P.13

Question No. Key 1. E 2. C 3. D 4. C 5. C 6. C 7. E 8. A 9. D 10. B 11. E 12. C 13. B 14. B 15. D 16. C 17. D 18. E 19. D 20. C 21. C 22. C 23. B 24. B 25. A

Question No. Key 26. A 27. A 28. C 29. B 30. D 31. C 32. B 33. A 34. C 35. E 36. A 37. D 38. C 39. A 40. C 41. B 42. A 43. A 44. A 45. D 46. E 47. D 48. D 49. D 50. E

Page 130: AL Physics 1981-2004 MC

92 AL Physics/M.C./P.1

1992 Hong Kong Advanced Level Examination AL Physics

Multiple Choice Question

1.

F A B C

Three blocks A, B and C of masses m, 2m and

2m respectively are placed on a smooth horizontal ground as shown in the figure above. A constant horizontal force is applied to block A so that the three blocks move with the same acceleration towards the right. What is the resultant force acting on block B?

A. F/5 B. F/3 C. 2F/5 D. 3F/5 E. F 2. A man in a lift measures his weight with a

compression balance and discovers that his weight is reduced by 10%. The lift is probably

(1) moving upwards and accelerating at g/10. (2) moving upwards and decelerating at g/10. (3) moving downwards and accelerating at

g/10. (4) moving downwards and decelerating at

g/10. (5) moving downwards with uniform velocity. A. (1) and (3) only B. (1) and (4) only C. (2) and (3) only D. (2) and (4) only E. (5) only 3.

A simple pendulum is swinging in a vertical

plane. When it is at the position shown, which

of the following diagrams best represents the forces acting on the bob? Neglect air friction.

A.

B.

C.

D.

E.

Page 131: AL Physics 1981-2004 MC

92 AL Physics/M.C./P.2

4. An object is placed on a horizontal platform

vibrating vertically in S.H.M. with a period of 0.2 s. The maximum amplitude of oscillation which will allow the object to remain in contact with the platform throughout the motion is

A. 1 cm B. 5 cm C. 10 cm D. 100 cm E. indeterminate 5. An object of mass m is released from a

spacecraft at a distance 3 R from the centre of the Earth which has radius R and mass M. On reaching the Earth’s surface, the increase in kinetic energy of the object is

A. Gm M/(3R) B. 2 Gm M/(3R) C. Gm M /(2R) D. Gm M/R E. 2 Gm M/R 6.

3 m 2 kg

3 m/s

1 m/s6 kg

2 m

O Two objects are moving with instantaneous

velocities as shown in the above diagram. The total angular momentum about the point O at this instant is

A. 4 kg m² s-1 B. 6 kg m² s-1 C. 10 kg m² s-1 D. 20 kg m² s-1 E. 30 kg m² s-1 7. A gymnast on a trampoline is performing a

somersault and draws his knees to his chest. Which of the following quantities (about a horizontal axis through his centre of mass) will be increased?

(1) His angular momentum.

(2) His moment of inertia. (3) His rotation speed. A. (1), (2) and (3) B. (1) and (2) only C. (2) and (3) only D. (1) only E. (3) only 8.

r

U

OH K

XY

Z

W The diagram above shows the variation of the

potential energy U between neighbouring atoms in a solid with the separation r between them . Which of the following features of the curves best explains why the solid expands on heating?

A. OH > HK B. OH > XZ C. KY > YW D. HK > XY E. YZ > XY 9. The radius of a soap bubble is r. If the

atmospheric pressure is P, and the surface tension of the soap solution is T, then the pressure inside the soap bubble is

A. P - 4T/r B. P - 2T/r C. P D. P + 2T/r E. P + 4T/r 10. A flat plate of area 0.1 m² is placed on a flat

surface, and is separated from it by a film of oil 10-5 m thick. If the coefficient of viscosity of oil is 1.8 N s m-2, the force required to cause the plate to slide on the surface at a constant speed of 1 cm/s is

Page 132: AL Physics 1981-2004 MC

92 AL Physics/M.C./P.3

A. 1.8 N B. 18 N C. 180 N D. 1800 N E. 18000 N 11. A ball-bearing is dropped into viscous oil.

Which of the following correctly describes its motion before the terminal speed is reached?

Velocity Acceleration A. decreases decreases B. decreases increases C. increases constant D. increases decreases E. increases increases 12. A ductile fracture occurs (1) after appreciable plastic deformation, by

slow crack propagation. (2) when dislocations are not free to move. (3) after a brittle fracture occurs. A. (1), (2) and (3) B. (1) and (2) only C. (2) and (3) only D. (1) only E. (3) only 13. Given: Avogadro constant = 6 × 1023 mol-1 Boltzmann constant = 1.38 × 10-23 J/K Molar gas constant = 8.31 J mol-1 K-1 If 1 mole of an ideal gas is heated under

constant pressure from 20 ºC to 70 ºC, the total kinetic energy of the gas molecules is increased by

A. 208 J B. 415 J C. 581 J D. 623 J E. 831 J 14. The relative atomic mass of oxygen is 16.

What is the ratio of the average speed of oxygen molecules to that of hydrogen molecules at room temperature?

A. 1/16 B. 1/4 C. 1 D. 4 E. 16 15. Which of the graphs best represents the

distribution of molecular speeds in a gas at 500 K if the dotted curve represents this distribution for the same gas at 300 K?

A.

speed

0

fraction ofmolecules

B.

speed

0

fraction ofmolecules

C.

speed

0

fraction ofmolecules

D.

speed

0

fraction ofmolecules

E.

speed

0

fraction ofmolecules

Page 133: AL Physics 1981-2004 MC

92 AL Physics/M.C./P.4

16. The image of an object formed on a screen by a convex lens has height a. By moving the lens towards the screen, it is found that there is a second lens position at which another image of height b is formed on the screen. The height of the object is

A. (a + b)/2 B. √(a² + b²) C. √(ab) D. √(a³/b) E. √(b³/a) 17. The sun subtends an angle of 0.5º at the surface

of the Earth. A convex lens of focal length 100 cm is used to form an image of the sun onto a screen. The diameter of the image is about

A. 1 mm B. 3 mm C. 5 mm D. 9 mm E. 50 mm 18. In an astronomical telescope set at normal

adjustment, the focal lengths of the objective and the eyepiece are 50 cm and 10 cm respectively. Which of the following gives the separation of the lenses and the angular magnification of the telescope?

Lens separation Angular magnification A. 30 cm 5 B. 40 cm 0.5 C. 40 cm 5 D. 60 cm 0.5 E. 60 cm 5 19.

A

D

BC

Light is incident on an octagonal mirror and

traverses a path ABCD, of total length L, before being reflected again from another face of the octagonal mirror. The octagonal mirror is rotated and its angular velocity adjusted until

the light emerges in the same direction as when the mirror is stationary. If the speed of light is c, the smallest angular velocity to achieve this condition is

A. πc/(8L) B. πc/(4L) C. πc/(2L) D. πc/L E. 2πc/L 20.

Direction of sound wave

1 2 3 4 5 678 9 10 11

1 2 3 4 5 6 7 8 9 10 11

Figure (a)

Figure (b)

Figure (a) shows the positions of equally

spaced molecules in a solid lattice. A longitudinal sound wave travels from left to right through he solid. At a certain instant, the displaced positions of the molecules are shown in Figure (b). Immediately afterwards, what will be the directions of motion of particle 1 and particle 7?

Particle 1 Particle 7 A. to the right to the right B. to the right to the left C. to the left to the right D. to the left to the left E. at rest at rest 21. A loudspeaker produces a sound intensity level

of 80 dB at a point P. If the electrical power to the loudspeaker is halved, the intensity level at P will be

A. 20 dB B. 30 dB C. 40 dB D. 50 dB E. 77 dB

Page 134: AL Physics 1981-2004 MC

92 AL Physics/M.C./P.5

22.

3.30 m

3.22 m

signalgenerator

Y

X

Two loudspeakers are connected to the same

signal generator. A microphone positioned at X detects maximum intensity. When the microphone is moved upwards, maximum intensity is also detected at Y. Which of the following may give possible values of the wavelength of the sound emitted from the loudspeakers?

(1) 0.04 m (2) 0.08 m (3) 0.16 m A. (1), (2) and (3) B. (1) and (2) only C. (2) and (3) only D. (1) only E. (3) only 23.

vibrator

Stationary wave patterns can be produced on

an elastic string using the experimental set-up shown above by adjusting the frequency f of the vibrator. Which of the following statements concerning the experiment is/are correct?

(1) Stationary wave patterns can be observed

for more than one value of the frequency f. (2) When the frequency f increases, the

number of loops for the stationary wave pattern to be observed also increases.

(3) For a stationary wave pattern to occur, the length of the string must be equal to an integral number of wavelengths.

A. (1), (2) and (3) B. (1) and (2) only C. (2) and (3) only

D. (1) only E. (3) only 24. When unpolarised light travelling in air falls on

the surface of a glass block, it is possible to find an angle of incidence such that

A. none of the light is reflected. B. the reflected light and the transmitted light

are both plane polarised. C. all the light is reflected. D. the transmitted light is plane polarised. E. the reflected light is plane polarised. 25.

SP Q

A light source S is viewed through two parallel

pieces of Polaroid P and Q. Q is gradually rotated until the field of view becomes dark. which of the following conclusions can be drawn?

(1) The experiment shows that light is a

transverse wave. (2) The experiment shows that light from

source S must be unpolarised. (3) When Q is rotated further by 90º, the field

of view becomes dark again. A. (1), (2) and (3) B. (1) and (2) only C. (2) and (3) only D. (1) only E. (3) only 26.

N MA

O Ct

air

soap

air

r

Page 135: AL Physics 1981-2004 MC

92 AL Physics/M.C./P.6

A ray AO of monochromatic light wavelength λ falls on a thin parallel-sided soap film of thickness t and refractive index n. Constructive interference will take place between rays M and N when

A. nt = mλ B. nt = (m + ½)λ C. 2 nt = mλ D. 2 nt cos r = mλ E. 2 nt cos r = (m + ½)λ 27. How do the two physical quantities below

change along the direction indicated by an electric field line from a point positive charge?

(1) electric field intensity E (2) potential V A. Only E will increase B. Only V will increase C. Both E and V will increase D. Both E and V will decrease E. Both E and V will remain constant 28.

RµA

C

A charged capacitor C, of 10 µF capacitance, is

connected in the circuit above. For a period of 40 s, the current is kept constant at 20 µA by continuous adjustment of R. During this period, the p.d. across the capacitor has fallen by

A. 8.0 × 10-4 V B. 1.3 × 10-2 V C. 14 V D. 20 V E. 80 V 29.

G

X8 V 2 V

150 Ω

In the circuit shown above, the galvanometer G

reading is zero. So X has a resistance of A. 15 Ω B. 25 Ω C. 50 Ω D. 100 Ω E. 150 Ω 30.

2

4

6

8

I

I

I

B

B

B

= 40

= 80

= 120

µ

µ

µ

A

A

A

VCE /V

IC /mA

The graphs show the characteristics for a

transistor operating in the common emitter mode. IC is the collector current, IB is the base current and VCE is the potential difference between the collector and emitter. The current gain for this transistor is

A. 20 B. 50 C. 80 D. 100 E. 150 31. The graph shows the transfer characteristic of

an electronic device.

+1

+2

+3

+4

output

10 2 3 4-1-2-3-4input voltage / V

voltage/V

Page 136: AL Physics 1981-2004 MC

92 AL Physics/M.C./P.7

The input is a sinusoidal voltage with a peak value of 1.5 V and a mean value of zero. Which one of the following waveforms best represents the variation of the output voltage with time?

A.

B.

C.

D.

E.

32.

_

+

1 k

50 k

In the circuit above, what is the voltage

amplification? A. 50 000 with inversion B. 1 000 with inversion C. 50 with inversion D. 50 without inversion E. 50 000 without inversion

33. A moving-coil galvanometer with a coil of area A and N turns has a full-scale deflection for a current i. If the coil were of area 3 A and 2 N turns, the current which would give full scale deflection would be

A. i/6 B. 2i/3 C. i D. 3i/2 E. 6i 34.

YXθ

B

I

A square coil if N turns and area A carries a

current I. The coil is free to rotate about the axis XY which is normal to a uniform magnetic field B. When the field makes an angle θ with the plane of the coil, what are the magnitude and direction of the torque acting on the coil as detected by an observed by X?

Magnitude Direction A. BANI cos θ clockwise B. BANI sin θ clockwise C. BANI cos θ anticlockwise D. BANI sin θ anticlockwise E. none of the above 35.

steel strip

S

iron core

50 Hz

When switch S is closed, the steel strip will A. remain stationary B. remain attracted to the iron core C. vibrate at a frequency of 50 Hz

Page 137: AL Physics 1981-2004 MC

92 AL Physics/M.C./P.8

D. vibrate at a frequency of 100 Hz E. vibrate at a frequency of 200 Hz 36. Two parallel straight conductors separated by a

distance r carry currents in the same direction. Which of the following statements is/are correct?

(1) The two wires attract each other. (2) The force acting on each wire is inversely

proportional to r². (3) The wires produce a magnetic field with

maximum flux density midway between them.

A. (1), (2) and (3) B. (1) and (2) only C. (2) and (3) only D. (1) only E. (3) only 37.

12 V

2 H 8 Ω When the switch in the circuit is closed, (1) the current will rise initially at the rate of 6

A/s, (2) the final value of the current is 1.5 A, (3) the final energy stored in the inductor is

2.25 J. A. (1), (2) and (3) B. (1) and (2) only C. (2) and (3) only D. (1) only E. (3) only 38.

P

Q

In the above figure, when the metal rod PQ

moves with constant velocity across a uniform magnetic field, a p.d. is induced across the rod. Which of the following statements is/are correct?

(1) The magnitude of the p.d. depends on the

length of the rod. (2) Q is at a higher potential than P. (3) A force is acting on the rod to oppose its

motion. A. (1), (2) and (3) B. (1) and (2) only C. (2) and (3) only D. (1) only E. (3) only 39.

RD D1 2

E = A ωtsin I

V/V

/mA

00.6

In the circuit shown above, D1, D2 are two

diodes used to rectify a sinusoidal a.c. supply. Each of the diodes has the I-V characteristic as shown. For a current to flow through R, the minimum value of A is

A. 0.6 V B. 1.2 V C. 0.3 √2 V D. 0.6 √2 V E. 1.2 √2 V 40.

Page 138: AL Physics 1981-2004 MC

92 AL Physics/M.C./P.9

A metal ring is made to float above a coil carrying alternating current. Which of the following will affect the height of the ring?

(1) the resistivity of the ring (2) the density of the ring (3) the frequency of the alternating current A. (1), (2) and (3) B. (1) and (2) only C. (2) and (3) only D. (1) only E. (3) only 41.

0.40 µ F

0.20 µ F

In the above circuit, the a.c. source has a r.m.s.

voltage of 5.0 V alternating at 50 Hz. What is the r.m.s. current?

A. 0.033 µA B. 0.12 µA C. 0.21 mA D. 120 mA E. 210 mA 42.

50 Ω

In the above circuit, if the sinusoidal a.c.

source has a peak-to peak voltage of 20 V, the r.m.s. current through the 50 Ω resistor is

A. 0.07 A B. 0.10 A C. 0.14 A D. 0.20 A E. 0.28 A 43.

A.C. voltages of frequencies f1 and f2 are

applied to the x-plates and y-plates of a CRO respectively. The trace observed is shown above. f1 : f2 is equal to

A. 1 : 3 B. 2 : 3 C. 1 : 1 D. 3 : 2 E. 3 : 1 44. The main reason why a chain reaction can

occur in a nuclear reactor using uranium is that A. a large quantity of energy is evolved in

each fission. B. the products of nuclear fission are highly

radioactive. C. plutonium is produced and it undergoes

further fission. D. neutrons are produced when a nucleus

undergoes fission. E. uranium is a highly radioactive element. 45. A radioactive source consists of a mixture of

two radioisotopes P and Q. P has a half-life of 1 hour and Q has a half-life of 2 hours. Both P and Q have stable daughter nuclei. The initial activity recorded by a counter is 600/min. After 4 hours the counter registers an activity of 60 counts per min. What was the contribution of P to the initial count rate (in counts per min.)?

A. 120 B. 200 C. 360 D. 400 E. 480 46. The work function of a metal is the least

energy required to A. release one mole of electrons from the

surface of the metal. B. bring one mole of electrons from the

interior of the metal to the surface. C. releases one electron from the surface of

the metal. D. bring one electron from the interior of the

metal to the surface.

Page 139: AL Physics 1981-2004 MC

92 AL Physics/M.C./P.10

E. bring one electron from the interior of the metal to the surface and release it.

47. Which of the following conclusions could

NOT be deduced from Rutherford’s scattering experiment?

(1) Alpha particles are helium nuclei. (2) There are discrete energy levels in an

atom. (3) The positive charge in an atom is confined

to a very small region. A. (1), (2) and (3) B. (1) and (2) only C. (2) and (3) only D. (1) only E. (3) only 48. N7

14 + alpha particle → proton + X In the above nuclear reaction, X is A. O8

17

B. F917

C. N817

D. C611

E. F918

49. The emf of a thermocouple E is measured at a

number of Celsius temperatures θ. E and θ are related by

E a= θ θ + b 2, where a and b are constants The above relationship can most easily be

verified by plotting a graph of A. E against θ B. E/θ against θ C. E/θ² against θ D. E against θ² E. log E against log θ 50. To determine the area of cross-section of a

metal wire, a student measures its diameter and obtains a value of 0.20 mm, subject to an error of ± 0.02 mm. Which of the following is the most appropriate way of expressing the result?

A. 0.03 ± 0.01 mm² B. 0.031 ± 0.003 mm² C. 0.031 ± 0.006 mm² D. 0.0314 ± 0.0031 mm² E. 0.0314 ± 0.0063 mm²

- END OF PAPER -

Page 140: AL Physics 1981-2004 MC

92 AL Physics/M.C./P.11

Question No. Key 1. C 2. C 3. A 4. A 5. B 6. B 7. E 8. E 9. E 10. C 11. D 12. D 13. D 14. B 15. A 16. C 17. D 18. E 19. B 20. C 21. E 22. B 23. B 24. E 25. D

Question No. Key 26. E 27. D 28. E 29. C 30. B 31. E 32. C 33. A 34. A 35. D 36. D 37. A 38. D 39. B 40. A 41. C 42. B 43. E 44. D 45. E 46. C 47. B 48. A 49. B 50. C

Page 141: AL Physics 1981-2004 MC

93 AL Physics/M.C./P.1

1993 Hong Kong Advanced Level Examination AL Physics

Multiple Choice Question

1.

F

F1

2 The resultant of two forces F1 and F2 acting at

a point can have a minimum value of 1 N and a maximum value of 7 N. When the two forces act at right angles to each other, the magnitude of their resultant is

A. 3 N B. 5 N C. 6 N D. √50 N E. 8 N 2.

sphere

30o

A sphere of mass 1 kg is released from rest on

an inclined plane of inclination 30º to the horizontal (as shown). If the sphere rolls without slipping, find the gain in kinetic energy and the work done against friction by the sphere after travelling a distance of 5 m along the plane.

gain in

kinetic energy/J

work done against friction/J

A. 25 0 B. 25 25 C. 50 cannot be determined D. 50 0 E. 50 25

3. A box moves at a uniform velocity of 2 m/s on

a frictionless horizontal surface. Sand falls into the box with negligible speed at a rate of 90 kg per minute. What horizontal force is required to keep the box moving uniformly at 2 m/s?

A. 0 N B. 3 N C. 6 N D. 90 N

E. 180 N 4.

P

Q

Two identical coins P and Q are placed at the

edge of a table. At the same instant, P is pushed slightly and falls vertically to the ground while Q is projected horizontally and reaches the ground through a parabolic path (as shown). Which of the following statements is/are correct? (Neglect air resistance)

(1) P and Q reach the ground at the same

time. (2) P and Q have the same acceleration. (3) P and Q have the same vertical speed on

reaching the ground. A. (1) only B. (3) only C. (1) and (2) only D. (2) and (3) only E. (1), (2) and (3) 5.

O OO

The above diagram shows a uniform hollow

metal sphere with a small opening on top. O is the position of the centre of mass of the hollow sphere. What will happen to the position of the centre of mass of the system as the sphere is being slowly filled with oil from the opening?

A. It will fall gradually, and its final position

will be below O. B. It will fall gradually at first and then rise to

its original position.

Page 142: AL Physics 1981-2004 MC

93 AL Physics/M.C./P.2

C. It will rise gradually and its final position will be above O.

D. It will rise gradually at first and then fall to its original position.

E. It will remain unchanged throughout the process.

6. The period of a simple pendulum undergoing

simple harmonic motion may be increased by A. using a heavier pendulum bob. B. increasing the amplitude of oscillation. C. placing the pendulum at the top of a

mountain. D. placing the pendulum at the North Pole. E. shortening the string attached to the bob. 7. A particle oscillates with simple harmonic

motion along a straight line with amplitude A. When the displacement of the particle from its equilibrium position is ½A, its speed is u. The speed of the particle when passing the equilibrium position is

A. 2u/√3 B. √2 u C. √3 u D. 2u E. 4u 8.

P

A

The figure shows a small heavy bob P attached

to a fixed point A on the ceiling by a light inextensible string. The bob is pulled aside with the string taut and then released from rest. Which of the following descriptions is/are true?

(1) When moving towards the lowest point of

its path, the angular speed of the bob is increasing.

(2) The centripetal acceleration of the bob is constant.

(3) When the bob is at the lowest point, the tension in the string equals the centripetal force.

A. (1) only B. (3) only C. (1) and (2) only D. (2) and (3) only E. (1), (2) and (3) 9. A ring of radius a is made from thin wire. The

moment of inertia of the ring about an axis through its centre and perpendicular to its plane is I. What would be the moment of inertia of a ring, made from the same type of wire but with radius 2a, about a similar axis?

A. I B. 2I C. 4I D. 8I E. 16I 10. The velocity of escape from the earth is V0.

For a planet with radius twice that of the earth and with density three times that of the earth, the velocity of escape from the planet would be

A. √3 V0/2 B. 2V0 C. √6 V0 D. 2√3 V0 E. 2√6 V0 11. For planets or satellites in circular orbits

around a celestial body such as the sun or the earth, the period T is related to the radius of orbit r by Kepler's 3rd law T² = kr³ where k is a constant.

Which of the following statements concerning

the constant k is correct? A. It is a dimensionless constant whose value

is not affected by the choice of units. B. It is a universal constant whose value is

not affected by the choice of units. C. It is a universal constant whose value

depends on the choice of units. D. It would have a certain value for the earth

moving around the sun, but a different

Page 143: AL Physics 1981-2004 MC

93 AL Physics/M.C./P.3

value for another planet moving around the sun.

E. It would have a certain value for all planets moving around the sun, but a different value for all satellites moving around the earth.

12. A communication satellite appears stationary

vertically above an observer at the equator. The height of the satellite above the observer is 3.6 × 107 m. Calculate the mass of the earth.

Given: Radius of the earth = 6.4 × 106 m Gravitational constant = 6.7 × 10-11 Nm2kg-2 A. 4.5 × 1024 kg B. 5.0 × 1024 kg C. 5.5 × 1024 kg D. 6.0 × 1024 kg E. 6.5 × 1024 kg 13. Which of the following phenomena is/are due

to capillary action? (1) The drying action of blotting paper. (2) Mercury in a narrow tube is depressed

below the mercury level outside. (3) Molten solder penetrates cracks in

soldering. A. (1) only B. (3) only C. (1) and (2) only D. (2) and (3) only E. (1), (2) and (3) 14. A metal sphere released in a liquid falls with

terminal velocity V1. If a polystyrene sphere of the same radius is released in the same liquid, it then rises with terminal velocity V2. Find the ratio V1/V2.

Given: d = density of the liquid ρ1 = density of the metal sphere ρ2 = density of the polystyrene

sphere A. ρ1/ρ2 B. (ρ2 - d)/(d - ρ1)

C. ρ2/ρ1 D. (ρ1 - d)/(d - ρ2) E. (ρ1 + d)/(ρ2 + d) 15.

fluid flowspeed v

h

The above figure shows a Pitot-static tube

situated in a moving fluid. Which of the following graphs best shows the relation between the speed v of the fluid and the difference in manometer levels h?

A.

h

v

B.

h

v

C.

h

v

D.

Page 144: AL Physics 1981-2004 MC

93 AL Physics/M.C./P.4

h

v

E.

h

v

16. An ideal gas is contained in a cylinder fitted

with a light piston as shown below:

pistonIDEALGAS

cylinder

smooth Initially, the piston is held fixed and the gas is

cooled. Afterwards, the piston is pushed inwards slowly. Which of the following graphs represents the variation of gas pressure P with gas volume V? (i represents initial state; f represents final state)

A.

if

V

P

B.

f

V

P

i

C.

i

f

V

P

D.

f

V

P i

E.

fV

Pi

17. An inexpansible vessel contains air at 50 ºC.

What percentage of air remains in the vessel if it is heated to 100 ºC under constant pressure? (You may take the ice point to be 273 K)

A. 87% B. 85% C. 73% D. 63% E. 50% 18. Identical containers A and B contain oxygen

(O2) and hydrogen (H2) respectively.

Container Container

O 2 H 2

A B Both gases are at room temperature and

atmospheric pressure. Which of the following statements is/are true?

In both containers, (1) the number of gas molecules is the same;

Page 145: AL Physics 1981-2004 MC

93 AL Physics/M.C./P.5

(2) the r.m.s. speed of gas molecules is the same;

(3) the frequency of collision of gas molecules with the walls of container is the same.

A. (1) only B. (3) only C. (1) and (2) only D. (2) and (3) only E. (1), (2) and (3) 19. The calibration curve of a resistance

thermometer against a constant volume gas thermometer is given below:

0 20 40 60 80 1001.1

1.2

1.3

1.4

1.5

1.6

Temperature / oC

Resistance / Ω

When the resistance thermometer is left in a

room, its resistance if found to be 1.28 Ω . Find the values of room temperature on both the resistance thermometer scale and the gas thermometer scale.

Room temperature on

the resistance thermometer scale/ºC

Room temperature on the gas thermometer

scale/ºC A. 36 36 B. cannot be

determined 36

C. 20 36 D. 20 cannot be determined E. 36 20

20. If the diameter of an air molecule is d, which of

the following gives correct estimates of the average separation of air molecules and their mean free path under atmospheric pressure?

Average separation

of air molecules Mean free path

A. d 10 d B. 10 d 10 d C. 10 d 100 d D. 100 d 10 d E. 100 d 100 d

21. The f-numbers on the aperture ring of a camera

are as follows: 2 2.8 4 5.6 8 11 16 22 Which of the following statements is/are true? (1) The camera’s largest aperture has an area

of πf²/16 where f is the focal length of the camera lens.

(2) The depth of field increases with increasing f-number.

(3) A photograph taken at f-number 5.6 and exposure time 1/60 s receives the same exposure as another taken at f-number 8 and exposure time 1/120 s.

A. (1) only B. (3) only C. (1) and (2) only D. (2) and (3) only E. (1), (2) and (3) 22. Which of the following equations correctly

gives the relation between the polarising angle i and the refractive index n of a material?

A. sin i = n B. n sin i = 1 C. cos i = n D. n tan i = 1 E. tan i = n 23. An object placed in front of a magnifying glass

forms an image at infinity with magnifying power 3. What is the focal length of the magnifying glass? (You may assume the least distance of distinct vision to be 25 cm)

A. 6.3 cm B. 8.3 cm C. 12.5 cm D. 25 cm E. 75 cm 24.

Page 146: AL Physics 1981-2004 MC

93 AL Physics/M.C./P.6

displacement

time in seconds

0 0.20 0.45 0.70

A displacement-time graph of a particle in a

plane progressive wave is shown above. What is the frequency of this wave?

A. 1.43 Hz B. 2 Hz C. 2.22 Hz D. 4 Hz E. 5 Hz 25. A radio produces a sound intensity level of 50

dB at a point 2 m away from it. If the power output of the radio is doubled, what is the sound intensity level at a point 6 m from the radio? (You may regard the radio as a point source)

A. 30.9 dB B. 43.5 dB C. 46.5 dB D. 48.2 dB E. 49.7 dB 26. A hydrogen source in a laboratory emits a line

spectrum with one of the lines having wavelength 656.3 nm. For a star receding from the earth at a speed of 200 km/s, what would be the wavelength of the corresponding line from the star observed on the earth? (Velocity of light = 3 × 108 m/s)

A. 655.5 nm B. 655.9 nm C. 656.7 nm D. 657.1 nm E. 657.5 nm 27.

M

Ssoft soft

sound sound

L

A loudspeaker L produces sound waves with frequency 1 000 Hz. The sound waves are reflected from a wall S. When a microphone M is moved between L and S, the loudness of the sound detected varies. (Speed of sound in air = 340 m/s)

Which of the following statements is/are true? (1) The variation in the loudness of the sound

is due to diffraction. (2) The separation between consecutive

positions of soft sound is 0.34 m. (3) Increasing the sound frequency will make

the positions of soft sound closer. A. (1) only B. (3) only C. (1) and (2) only D. (2) and (3) only E. (1), (2) and (3) 28. An unchanged metal sphere is placed in a

uniform electric field. Which of the following best represents the electric field pattern around the metal sphere?

A.

B.

C.

D.

Page 147: AL Physics 1981-2004 MC

93 AL Physics/M.C./P.7

E.

29.

AB

a

b

A positively-charged metal sphere A of radius

a is joined by a conducting wire to an uncharged metal sphere B of radius b placed far away from the first sphere. The ratio of the surface charge density on sphere A to that on sphere B is

A. b/a B. b²/a² C. a/b D. a²/b² E. √(b/a) 30.

RC

E

An initially uncharged capacitor of capacitance

C is connected in series with a resistor R and a battery of e.m.f. E. What will be (1) the total work done by the battery in fully charging up C and (2) the energy finally stored in C?

Work done by battery

Energy stored in C

A. ½CE² 0 B. CE² 0 C. ½CE² ½CE² D. CE² ½CE² E. CE² CE²

31.

G

Y

X 12 V, 3 Ω

6 Ω 6 Ω

3 Ω Two cells X and Y, each of internal resistance

3 Ω, are connected with two 6 Ω resistors as shown in the above circuit.

If cell X has e.m.f. 12 V and the galvanometer G shows null deflection, what is the e.m.f. of cell Y?

A. 3.0 V B. 4.0 V C. 4.8 V D. 6.0 V E. 7.2 V 32. Which of the following devices is/are used to

stored energy? (1) an inductor (2) a capacitor (3) a photocell A. (1) only B. (3) only C. (1) and (2) only D. (2) and (3) only E. (1), (2) and (3) 33. Which of the following physical quantities

take(s) the unit ohm (Ω)? (1) resistance (2) reactance (3) impedance A. (1) only

Page 148: AL Physics 1981-2004 MC

93 AL Physics/M.C./P.8

B. (3) only C. (1) and (2) only D. (2) and (3) only E. (1), (2) and (3) 34.

P QX

E R

RE

A B

G galvanometer22

1 1

The figure above shows a potentiometer circuit

used for measuring the e.m.f.s of cells. The potential gradient along the potentiometer wire PQ is assumed to be constant. Which of the following statements is/are true?

(1) If the balance point X is close to P, the

resistance of R1 should be decreased. (2) At balance, it is better to have point X near

the center of PQ. (3) At balance, there is no current flowing

through the path PABX. A. (1) only B. (3) only C. (1) and (2) only D. (2) and (3) only E. (1), (2) and (3) 35. Two insulated parallel metal plates are

connected to the terminals of an EHT. When a charged aluminium foil strip is placed between the plates, deflection of the foil is observed as shown.

EHT_ +

metalplate

polystyrenerod

metalplate

aluminiumfoil strip

Which of the following statements is/are true?

(1) The charge on the foil is negative. (2) Deflection of the foil increases if the

separation between the plates decreases. (3) When moving the foil towards the positive

plate, the deflection of the foil increases. A. (1) only B. (3) only C. (1) and (2) only D. (2) and (3) only E. (1), (2) and (3) 36.

0.2 m

1 A

1 A

X

The above figure shows two long parallel

straight wires separated by a distance of 0.2 m, carrying currents of 1 A in opposite directions. The magnetic field at a point X mid-way between the wires is (Given: permeability constant µ0 = 4π × 10-7 TmA-1)

A. 0 T B. 2 × 10-6 T into paper C. 2 × 10-6 T out of paper D. 4 × 10-6 T into paper E. 4 × 10-6 T out of paper 37.

10 VA

M

A 10 V battery of negligible internal resistance

is applied to a d.c. motor M with armature resistance 5 Ω. If the ammeter A of negligible internal resistance shows a reading of 0.4 A, the maximum useful output power delivered by the motor is

A. 0.8 W. B. 1.6 W. C. 2.4 W. D. 3.2 W. E. 4.0 W.

Page 149: AL Physics 1981-2004 MC

93 AL Physics/M.C./P.9

38.

0

V1

time

0

V2

time

Potential differences V1 and V2 are applied to

the X-plates and Y-plates of a C.R.O. respectively. The trace on the screen is

A.

B.

C.

D.

E.

39.

primary coil secondary coil

soft iron

A Bcurrentsource

I p

bar

Two coils are linked by a soft iron bar as shown. A current source is connected to the primary coil. The primary current Ip varies with time as shown by the graph below:

0t t time

Ip

1 2 Which of the following sketches represents the

variation of the voltage across the secondary coil VAB with time?

A.

0

t t time

V AB

1 2 B.

0

t t time

V AB

1 2 C.

0

t t time

V AB

1 2 D.

0

t t time

V AB

1 2 E.

Page 150: AL Physics 1981-2004 MC

93 AL Physics/M.C./P.10

0

t t time

V AB

1 2 40.

0 V

+ 6 V

VoutVin

15 k Ω

2 k Ω

0 1 2

2

4

6

Vout / V

Vin / V

The above diagrams show an NPN transistor

circuit and its input/output voltage characteristic. What is the current amplification factor of the transistor?

A. 10 B. 30 C. 60 D. 75 E. 150 41.

V

0 1 2 3 4

101 /V

time/ms

time/ms0 1 2 3 4

5V2 /V

_+

V1V2

-15 V

+15 V

Vout

Two electrical signals V1 and V2 are fed into an

operational amplifier. The variations of V1 and V2 with time are shown above. Which of the following graphs represents the variation of the output Vout with time?

A.

0

Vout

time

B.

0

Vout

time

C.

0

Vout

time

D.

0

Vout

time

E.

0

Vout

time

42.

CL B

a.c. supply

The above figure shows an a.c. circuit with a

filament lamp B connected in series with a variable inductor L and a capacitor C. The frequency of the a.c. supply can be varied.

Page 151: AL Physics 1981-2004 MC

93 AL Physics/M.C./P.11

Initially the applied voltage leads the current in the circuit. Which of the following methods will make the filament lamp B brighter?

(1) Connect a capacitor in parallel with C (2) Increase the inductance of L (3) Decreases the frequency of the a.c. supply A. (1) only B. (3) only C. (1) and (2) only D. (2) and (3) only E. (1), (2) and (3) 43. Which of the following materials satisfies the

description: ductile, strong and stiff? A. steel B. diamond C. glass D. concrete E. wood 44. The ionisation potential of a hydrogen atom is

13.6 V. What is the minimum excitation potential of a ground state hydrogen atom?

A. 1.9 V B. 3.4 V C. 6.8 V D. 10.2 V E. 12.8 V 45. The spectrum of sunlight has dark lines in it.

Which of the following statements concerning these dark lines is/are correct?

(1) They are due to the absorption of certain

wavelengths of light in the sun’s atmosphere.

(2) Light absorbed in the sun’s atmosphere is re-emitted but in all directions.

(3) They are due to the absorption of certain wavelengths of light in the earth’s atmosphere.

A. (1) only B. (3) only C. (1) and (2) only D. (2) and (3) only E. (1), (2) and (3)

46. An X-ray tube emits X-rays with a minimum wavelength of 3.55 × 10-11 m. Estimate the potential difference between the cathode and the anode (target) in the X-ray tube.

Given: Planck constant = 6.63 × 10-34 Js Electronic charge = 1.6 × 10-19 C Velocity of light in air = 3 × 108 m/s A. 20 000 V B. 25 000 V C. 30 000 V D. 35 000 V E. 40 000 V 47.

P

atomicnucleus

r0-particleα

The diagram shows an alpha-particle

‘colliding’ head-on with an atomic nucleus. At P, the alpha-particle is at the closest distance r0 from the nucleus. Which of the following statements is/are correct?

(1) At P, the electric potential energy of the

system is maximum. (2) r0 is of the order 10-14 m. (3) r0 gives an upper limit for the sum of the

radii of the alpha-particle and the nucleus. A. (1) only B. (3) only C. (1) and (2) only D. (2) and (3) only E. (1), (2) and (3) 48. The number of radioactive nuclides in two

different samples P and Q are initially 4N and N respectively. If the half-life of P is t and that of Q is 2t, the number of radioactive nuclides in P will be the same as the number of radioactive nuclides in Q after a time of

A. t/2 B. t C. 2t D. 4t E. 8t 49.

Page 152: AL Physics 1981-2004 MC

93 AL Physics/M.C./P.12

P

Q

ion source

Two particles P and Q of same quantity of

charge and mass but moving with different speeds vP and vQ respectively enter a region of uniform magnetic field directed into the plane of the paper. The subsequent circular paths are as shown. Which of the following statements is/are correct?

(1) Both P and Q are positively charged. (2) vP is small than vQ. (3) The period of circular motion of P is

shorter than that of Q. A. (1) only B. (3) only

C. (1) and (2) only D. (2) and (3) only E. (1), (2) and (3) 50. A parallel-plate capacitor is formed by two

square metal plates. To determine the capacitance C, a student measures the length of side l and separation d of the plates.

If the maximum percentage error of l = 5% the maximum percentage error of d = 3% then the maximum percentage error for C will

be A. 7% B. 8% C. 13% D. 22% E. 28%

- End of Paper -

Page 153: AL Physics 1981-2004 MC

93 AL Physics/M.C./P.13

Question No. Key 1. B 2. A 3. B 4. E 5. B 6. C 7. A 8. A 9. D 10. D 11. E 12. D 13. E 14. D 15. A 16. A 17. A 18. A 19. C 20. C 21. C 22. E 23. B 24. B 25. B

Question No. Key 26. C 27. B 28. A 29. A 30. D 31. C 32. C 33. E 34. B 35. C 36. D 37. D 38. E 39. B 40. D 41. E 42. B 43. A 44. D 45. C 46. D 47. E 48. D 49. C 50. C

Page 154: AL Physics 1981-2004 MC

94 AL Physics/M.C./P.1

1994 Hong Kong Advanced Level Examination AL Physics

Multiple Choice Question

1.

P

rod

motor

propeller The above figure shows a propeller-motor

system connected by a light, rigid rod to a fixed point P on the ceiling. The system remains stationary when the motor is on. Which of the following diagrams correctly represents the forces acting on the propeller-motor system?

A.

B.

C.

D.

E.

2. A man of weight W stands on a compression

balance placed inside a lift. The velocity-time graph of the lift is shown below (The upward direction is taken to be positive):

0 t ttime

velocity

1 2 Which of the following graphs best shows the

variation of the reading on the balance, R, with time, t?

A.

0 t t

t

R

1 2

W

B.

0 t t

t

R

1 2

W

C.

0 t t

t

R

1 2

W

D.

Page 155: AL Physics 1981-2004 MC

94 AL Physics/M.C./P.2

0 t t

t

R

1 2

W

E.

0 t t

t

R

1 2

W

3.

R

Q

P

X

A uniform metre rule of mass 0.15 kg is hinged

to a wall at P and the other end R is connected by a wire attached to the wall at Q, vertically above P. A block X of mass 0.1 kg is hung from the rule 30 cm from R. The metre rule is horizontal. Find the moment about P produced by the tension in the wire.

A. 1.45 Nm B. 1.05 Nm C. 0.75 Nm D. 0.70 Nm E. 0.25 Nm 4. A softball of mass 0.5 kg flies horizontally

with a speed of 20 m/s towards a player. After being hit by the bat, it flies away at 30 m/s perpendicular to its original direction. Find the magnitude of the impulse acting on the softball.

A. 5 kg m/s B. 18 kg m/s C. 20 kg m/s D. 25 kg m/s E. 36 kg m/s 5. A student whirls a small bucket of water in a

vertical circle of radius 0.6 m. For no spilling, what is the minimum speed of the bucket at the highest point of its path?

A. 2.45 m/s B. 3.46 m/s C. 4.08 m/s D. 4.90 m/s E. 5.77 m/s 6. Three bombs are released from a bomber

flying horizontally with constant velocity to the right. They are released from rest (relative to the bomber) one by one at one-second intervals. Neglecting air resistance, which of the following diagrams correctly shows the positions of the bomber and the three bombs at a certain instant?

A.

B.

C.

D.

E.

Page 156: AL Physics 1981-2004 MC

94 AL Physics/M.C./P.3

7. When an object falls freely, its total energy A. increases during the fall. B. decreases during the fall. C. remains constant during the fall. D. is zero at the beginning of the fall. E. is at a maximum at the end of the fall. 8. A small mass is hung vertically from a light

spring fixed at its upper end. When the mass is pulled down 1 cm from its equilibrium position and released from rest, it takes 0.3 s to rise back to its equilibrium position. If the mass is pulled down 2 cm from its equilibrium position and released from rest, how long does it take for the mass to rise 1 cm? (Assume that the spring obeys Hooke’s law.)

A. 0.30 s B. 0.25 s C. 0.20 s D. 0.15 s E. 0.10 s 9. Which of the following physical quantities will

decrease with time in damped harmonic motion?

(1) Period (2) Amplitude (3) Mechanical energy

A. (1) only B. (3) only C. (1) and (2) only D. (2) and (3) only E. (1), (2) and (3) 10.

O O'

The above figure shows a uniform ring and a

uniform disc, with equal mass and radius,

smoothly hinged at points O and O’ on their respective circumferences. The are set into small oscillation of equal amplitude about axes through O and O’ perpendicular to the plane of the paper. Which of the following statements is/are correct?

(1) The moment of inertia of the ring about

the axis through O is greater than that of the disc about the axis through O’.

(2) The period of oscillation of the ring is longer.

(3) Both the ring and the disc have the same total kinetic energy when their centres are vertically below their respective axes.

A. (1) only B. (3) only C. (1) and (2) only D. (2) and (3) only E. (1), (2) and (3) 11. Two satellites A and B of the same mass are

moving in circular orbits round the earth. The radius A’s orbit is r and that of B’s orbit is 2r. Their total mechanical energies are EA and EB respectively. Which of the following descriptions about EA and EB is correct? (Gravitational potential energy is taken to be zero at infinity)

A. EA > 0 and EB = 2EA B. EA > 0 and EB = ½EA C. EA > 0 and EB = -2EA D. EA < 0 and EB = 2EA E. EA < 0 and EB = ½EA 12.

A

B0.4 m

The figure above shows part of a pipe having

circular cross-sections. The area of the cross-section at B is double that at A and the centre of the cross-section at B is 0.4 m higher than that at A. If an ideal liquid flows steadily through the pipe with speed 4 m/s at A, what is the difference in static pressure between A and B? (Given: density of the liquid is 1200 kg/m³)

A. 2 400 N/m² B. 4 200 N/m² C. 4 800 N/m²

Page 157: AL Physics 1981-2004 MC

94 AL Physics/M.C./P.4

D. 7 200 N/m² E. 12 000 N/m² 13. One mole of an ideal gas is contained in a

cylinder fitted with a light, frictionless piston. The gas is heated under constant pressure P so that its volume increases from V to 4V and the final temperature is T. If R is the universal gas constant, the work done by the gas is

A. 3RT/4 B. RT/3 C. RT D. 3RT E. 4RT 14.

00.3 0.6 0.9

displacement / m

position/m

Figure (a)

00.5 1.5 2.5

displacement / m

time/ms

Figure (b) Figure (a) represents the displacement-position

graph of a travelling wave at a certain instant and Figure (b) represents the displacement-time graph of a particle in the wave. The speed of the wave is

A. 300 m/s B. 150 m/s C. 1.2 m/s D. 0.6 m/s E. 0.3 m/s 15. The speed of light in a certain material is 1.6 ×

108 m/s. Find the critical angle for that material and air. (Speed of light in air = 3 × 108 m/s)

A. 28.1º

B. 32.2º C. 41.8º D. 48.0º E. 57.8º 16. Three tuning forks X, Y and Z are of slightly

different frequencies. If X and Y are sounded together, 3 beats per second are heard while X and Z sounded together give 1 beat per second. Which of the following is/are correct deduction(s)?

(1) 4 beats per second are heard when Y and Z

are sounded together. (2) The frequency of Z is higher than that of

Y. (3) If X has the highest frequency, then the

frequency of Y must be the lowest. A. (1) only B. (3) only C. (1) and (2) only D. (2) and (3) only E. (1), (2) and (3) 17. Which of the following waves can be

polarised? (1) microwaves (2) X-rays (3) ultrasonic waves A. (1) only B. (3) only C. (1) and (2) only D. (2) and (3) only E. (1), (2) and (3) 18.

/2

P Q R

l/2l

A stationary sound wave vibrating in its

fundamental mode is set up in a pipe open at both ends. If an air particle at P oscillates with amplitude a, what are the amplitudes of oscillation of the air particles at Q and R? (Neglect end corrections)

Amplitude at Q Amplitude at R A. 0 a/2 B. a/2 0 C. 0 a D. a 0

Page 158: AL Physics 1981-2004 MC

94 AL Physics/M.C./P.5

E a a 19. An ambulance, sounding its siren to produce a

note of 800 Hz, approaches a stationary pedestrian P at a steady speed of 40 m/s. Calculate the frequency of the sound heard by P. (Speed of sound in air = 340 m/s)

A. 706 Hz B. 716 Hz C. 800 Hz D. 894 Hz E. 907 Hz 20.

standvibrator

In the above experimental set-up, different

stationary wave patterns are produced on an elastic string by adjusting the frequency f of the vibrator. Which of the following statements is/are correct?

(1) When f increases, the number of antinodes

increases. (2) When f increases, the speed of the waves

on the string increases. (3) The waves produced in air by the string

has the same speed as the waves on the string.

A. (1) only B. (3) only C. (1) and (2) only D. (2) and (3) only E. (1), (2) and (3) 21. Which of the following ray diagrams is/are

correct? (F is the focus of the corresponding optical instrument.)

F F

F

F F

(1)

(2)

(3)

A. (1) only B. (3) only C. (1) and (2) only D. (2) and (3) only E. (1), (2) and (3) 22. A uniform wire of force constant k and Young

modulus E is cut into two shorter wires of equal length. If they are arranged side by side and treated as a single wire combination, what are the force constant and the Young modulus for this combination?

Force Constant Young modulus A. k E B. 2k E C. 2k 2E D. 4k E E. 4k 2E 23. A 10 W light bulb gives a certain illumination

on a surface 1.5 m away. At what distance would a 40 W light bulb give the same illumination? (You may regard a light bulb as a point source)

A. 1.5 m B. 3.0 m C. 6.0 m D. 12.0 m E. 24.0 m

Page 159: AL Physics 1981-2004 MC

94 AL Physics/M.C./P.6

24. X, Y are two different points in an electric field. A small charged object is released from rest at X. Which of the following conditions would ensure that the charged object will NOT pass through Y?

A. The electric field at Y is zero. B. The electric field at Y is stronger than that

at X. C. The electric field between X and Y is not

zero. D. The electric potentials at X and Y are

equal. E. The electric potential difference between

X and Y is not zero. 25. Which of the following pairs of physical

quantities is/are both scalars? (1) work done and electric field intensity (2) pressure and force (3) charge and electric potential A. (1) only B. (3) only C. (1) and (2) only D. (2) and (3) only E. (1), (2) and (3) 26. Two sinusoidal a.c. signals having a phase

difference of 90º are applied to the x-plates and y-plates of a CRO. Which of the following traces may appear on the screen of the CRO?

(1) (2) (3)

A. (1) only B. (3) only C. (1) and (2) only D. (2) and (3) only E. (1), (2) and (3) 27. The electric potential energy of a system of

charges at infinitely large distances from one another is taken to be zero. What is the electric potential energy stored in a system of four charges, each of +1 C, placed at the vertices of a square with length of side 1 m? (ε0 = permittivity of vacuum)

A. 3

0πε

B. 1

2120πε

( )+

C. 1

0πε

D. 5

4 0πε

E. 1

44 2

0πε( )+

28. A parallel-plate capacitor of capacitance C0 is

formed by two rectangular metal plates having separation d. Now each of the plates is cut into two smaller, identical ones to form two capacitors, each with plate separation 2d. What is the capacitance when they are connected in parallel?

A. ½C0 B. ¼C0 C. C0 D. 2C0 E. 4C0 29.

X

G

80 cm60 cm10 cm

Ω40

uniform

carrying steadycurrent

resistance wire

In the above circuit, the galvanometer G reads

zero. The resistance of resistor X is A. 40 × (5/2) Ω B. 40 × (4/3) Ω C. 40 × (2/5) Ω D. 40 × (3/4) Ω E. 40 × (7/5) Ω 30.

A B

C C

C

2 3

1

S

In the above circuit, a charged capacitor C1 is

connected across AB so that it is parallel with

Page 160: AL Physics 1981-2004 MC

94 AL Physics/M.C./P.7

two initially uncharged capacitors C2 and C3. Which of the following statements is/are correct when switch S is closed?

(1) The potential difference across AB will

drop. (2) The combined capacitance across AB will

increase. (3) The quantity of charge on C2 is the same

as that on C3. A. (1) only B. (3) only C. (1) and (2) only D. (2) and (3) only E. (1), (2) and (3) 31. For two long, straight parallel conducting wires

carrying the same current, the magnitude of the force acting on a section of the wires depends on

(1) the distance between the wires (2) the length of that section of the wires (3) the directions of current flow in the wires A. (1) only B. (3) only C. (1) and (2) only D. (2) and (3) only E. (1), (2) and (3) 32. The current in a coil changes steadily from 3 A

to 6 A in 75 ms so that a back e.m.f. of 4 V is induced in the coil. The self-inductance of the coil is

A. 0.025 H B. 0.10 H C. 40 H D. 100 H E. 160 H 33.

0 t

y

V1 V2

Si

When the switch S in the above circuit is

closed, the variation of quantity y with time t is plotted as shown. The quantity y could be

(1) the current i in the circuit. (2) the voltage V1 across the resistor. (3) the voltage V2 across the inductor. A. (1) only B. (3) only C. (1) and (2) only D. (2) and (3) only E. (1), (2) and (3) 34.

V V V

R C L

R C L

Vr.m.s.

In the LCR series circuit shown above, the

frequency of the a.c. source is varied while its r.m.s. output voltage Vr.m.s. is kept constant. The voltmeters connected across R, C and L give r.m.s. readings VR, VC and VL respectively. Which of the following statements is/are correct?

(1) At any instant, the current through R, C

and L is the same. (2) At resonance VL = VC. (3) At any instant, Vr.m.s. = VR + VC + VL. A. (1) only B. (3) only C. (1) and (2) only D. (2) and (3) only E. (1), (2) and (3) 35. Which of the following statements about an

operational amplifier is/are correct? (1) It amplifies the difference between the

voltages at its two inputs. (2) For d.c., the open loop voltage gain is of

the order 105. (3) For a.c., the open loop voltage gain

decreases with increasing frequency. A. (1) only B. (3) only C. (1) and (2) only D. (2) and (3) only E. (1), (2) and (3)

Page 161: AL Physics 1981-2004 MC

94 AL Physics/M.C./P.8

36.

X

0 V

+ 0.5 V10 k

100 kΩ

Ω

+

_

The above figure shows an operational

amplifier circuit which uses a ±15 V supply (not shown). If the input potential is +0.5 V, what is the potential at point X?

A. -5 V B. -15 V C. +0.5 V D. +5 V E. +15 V 37.

f0

V

A student measures the p.d. V to stop

photoelectrons emitted in a photocell illuminated by monochromatic light of various frequencies f. The resulting points when plotted on a V-f graph (as shown) do not lie on the solid line drawn from standard results obtained with a similar photocell. The reason could be

A. the standard results are obtained with light

of higher intensity. B. he has used a voltmeter which has a fixed

zero error. C. he has read the wrong scale on his

voltmeter so that his readings always double the actual readings.

D. he has connected the variable d.c. supply with the wrong polarities to the photocell.

E. he has plotted the wavelength of light in place of the frequency on the horizontal axis.

38. In a hydrogen atom, electron transitions from

the first excited state to the ground state give photons of frequency f. If an electron falls

from the second excited state to the first one, the frequency of the photon emitted would be

A. 0.19 f B. 0.44 f C. 0.84 f D. 1.19 f E. 2.25 f 39.

a.c.source to CRO

100 k Ω

In the above circuit, the waveform across the

diode shown on the screen of the CRO should be

A.

B.

C.

D.

E.

40. In Thomson's experiment, an electron is

accelerated from rest through a p.d. V (not shown) and then passes without any deflection through a region with mutually perpendicular electric and magnetic fields.

e-

+

_

Y

Y

1

2

Page 162: AL Physics 1981-2004 MC

94 AL Physics/M.C./P.9

The electric field is provided by the deflecting plates Y1, Y2 with p.d. V and separation d. The applied uniform magnetic field is B. The charge to mass ratio (e/m) of an electron is given by

A. 2 2 2B d

V

B. V

B d2 2 2

C. B d

V

2 2

2

D. 22 2V

B d

E. B d

V

2 2

32

41. When a radioactive atom emits γ-rays, which

of the following statements about the atom is/are correct?

(1) Its mass number remains unchanged. (2) The energy of the atomic nucleus

decreases. (3) An electron falls from a higher energy

level to a lower one. A. (1) only B. (3) only C. (1) and (2) only D. (2) and (3) only E. (1), (2) and (3) 42. Given: mass of proton = 1.0073 u mass of neutron = 1.0087 u mass of 82

206 Pb = 205.969 u 1 u corresponds to 931 MeV Find the binding energy per nucleon for a

82206 Pb nucleus.

A. 7.46 MeV B. 7.72 MeV C. 12.39 MeV D. 12.83 MeV E. 19.40 MeV 43.

0 x

y

A plot of two physical quantities x and y yields

the above graph. Which of the following could be those two physical quantities?

Quantity x Quantity y A. radius of a

satellite’s orbit period of the satellite

B. distance from a point charge

electric potential due to the point charge

C. extension of a spring

energy stored in the spring

D. time charge on a charging capacitor

E. frequency of an a.c. supply

inductive reactance of an inductor connected to the a.c. supply

44. A detector is used for monitoring an α-source

and a reading of 120 units is observed. After a time equal to the half-life of the α-source, the reading has fallen to 64 units. If a 5 mm thick lead sheet is inserted between the α-source and the detector, the reading would probably be

A. 0 unit B. 4 units C. 8 units D. 16 units E. 32 units 45. A stationary uranium-238 nucleus undergoes α

-decay. What is the ratio of the kinetic energy of the daughter nucleus to that of the α -particle?

A. 238 : 4 B. 4 : 238 C. 234 : 4 D. 4 : 234 E. 1 : 1

- End of Paper -

Page 163: AL Physics 1981-2004 MC

94 AL Physics/M.C./P.10

Question No. Key 1. D 2. D 3. A 4. B 5. A 6. D 7. C 8. C 9. D 10. E 11. E 12. A 13. A 14. A 15. B 16. B 17. C 18. C 19. E 20. A 21. B 22. D 23. B 24. D 25. B

Question No. Key 26. D 27. E 28. A 29. C 30. E 31. C 32. B 33. C 34. C 35. E 36. A 37. B 38. A 39. E 40. B 41. C 42. B 43. E 44. C 45. D

Page 164: AL Physics 1981-2004 MC

95 AL Physics/M.C./P.1

1995 Hong Kong Advanced Level Examination AL Physics

Multiple Choice Question

1. When given a slight push, a toy car moves freely with constant velocity down a plane inclined at 20º to the horizontal. If the mass of the car is 0.5 kg, find the force parallel to the inclined plane for pulling the car up the plane with constant velocity.

A. 1.7 N B. 3.4 N C. 4.7 N D. 6.7 N E. 9.4 N 2.

A

Bh

h4

Two small spheres A and B of masses 1 kg

and 2 kg respectively are released from rest at heights 4h and h above the ground as shown. Which of the following statements is/are correct? (Assume air resistance is negligible)

(1) The acceleration of sphere A doubles

that of sphere B. (2) The time taken for sphere A to reach

the ground is double that of sphere B. (3) The kinetic energy of sphere A when

reaching the ground is double that of sphere B.

A. (1) only B. (3) only C. (1) and (2) only D. (2) and (3) only E. (1), (2) and (3) 3.

A trolley travels with constant velocity to

the right on horizontal ground and collides with a light helical spring attached to a wall fixed to the ground (earth) as shown. At the instant that the trolley comes momentarily to rest during collision, what has happened to the initial momentum of the trolley?

A. It has been transferred to the earth. B. It has been stored in the spring. C. It has been dissipated as sound and

heat. D. It has been destroyed by the friction

due to the ground. E. It has been lost because the collision

was not perfectly elastic. 4.

30o

x

For safety reasons, a vehicle should be so

designed that no sideways toppling occurs before reaching an angle of inclination of 30º. If the centre of gravity of that vehicle is 1.8 m above the ground, what is the minimum separation x between its wheels?

A. 1.0 m B. 1.5 m C. 1.8 m D. 2.1 m E. 2.4 m 5. A small object of mass 0.05 kg is released

from rest at the rim of a heavy, smooth semi-spherical bowl of radius 10 cm. Find

Page 165: AL Physics 1981-2004 MC

95 AL Physics/M.C./P.2

the force acting on the object by the bowl when it passes the bottom of the bowl.

A. 0.5 N B. 1.0 N C. 1.5 N D. 2.0 N E. 2.5 N 6. An aircraft flies along a horizontal circle

of radius 10 km with a constant speed of 155 m/s. Calculate the angle between its wings and the horizontal.

A. 11.5º B. 12.0º C. 12.5º D. 13.0º E. 13.5º 7.

mg

T

θ

A simple pendulum is displaced an angle θ

and is released from rest. If T is the tension in the string and m is the mass of the bob, which of the following statements is/are correct?

(1) At the moment when the bob is

released, T cos θ = mg. (2) The restoring force of the harmonic

motion is T sin θ. (3) The period of oscillation is

independent of θ when θ is small. A. (1) only B. (3) only C. (1) and (2) only D. (2) and (3) only E. (1), (2) and (3) 8.

wall

W

beam AB

C

wire

The figure shows a uniform rigid beam

AB, pivoted at A, held in horizontal position by a wire attached to a wall at point C, vertically above A. The beam carries a load W. If W is shifted gradually from A towards B, which of the following quantities will increase?

(1) The tension in the wire. (2) The horizontal compression force in

the beam. (3) The vertical component of the

reaction at A. A. (1) only B. (3) only C. (1) and (2) only D. (2) and (3) only E. (1), (2) and (3) 9.

2 kg

4 kg

A

CB

1 m

2 m

Point masses of 2 kg and 4 kg are attached

to the ends of an L-shaped light frame ABC, with AB vertical and BC horizontal. The frame is pivoted at and free to rotate about point B in a vertical plane. What is the initial angular acceleration of the system when released form rest?

A. 4.4 rad s-2 B. 5.0 rad s-2 C. 5.6 rad s-2 D. 8.0 rad s-2 E. 10.0 rad s-2 10.

A BP

S

Page 166: AL Physics 1981-2004 MC

95 AL Physics/M.C./P.3

The figure shows a satellite S moving round a planet P in an elliptical orbit. Which of the following statements is/are correct?

(1) The speed of the satellite at A is faster

than when it is at B. (2) The angular momentum of the

satellite about P is the same when it is at positions A and B.

(3) The total mechanical energy of the satellite is the same at positions A and B.

A. (1) only B. (3) only C. (1) and (2) only D. (2) and (3) only E. (1), (2) and (3) 11. On a certain planet, an object is thrown

vertically upwards with an initial velocity of v1 and it returns to the ground after time t. If the velocity of escape from the planet is v2, find the radius of the planet.

A. 2 12

2

v tv

B. 4 12

2

v tv

C. 2 22

1

v tv

D. v tv22

14

E. v tv22

12

12.

direction of propagation of wave

A B The figure shows a sound wave travelling

to the right in air. Air particles A and B are at the centre of a compression and a rarefaction respectively. Which of the following gives correctly the directions of motion of A and B at the moment shown?

Particle A Particle B

A. to the right to the left B. to the right at rest C. to the right to the right D. at rest to the right E. to the left to the right 13.

θ

Y

X

θ

intensity

0 30 60 90o o o o When a beam of light travels from a

medium X to another medium Y, the variation in intensity of the refracted beam when angle θ varies from 0º to 90º is as shown. What is the ratio speed of light inspeed of light in

XY

?

A. 1 : 2 B. 1 : 3 C. 2 : 1 D. 2 : 3 E. 3 : 1 14. In Young’s double-slit experiment, which

of the following combinations of monochromatic light, the slit-separation and the slit-to-screen distance would produce the widest fringe separation on the screen?

Monochromatic

light Slit-

separation Slit-to-screen

distance A. red light 1 mm 1 m B. red light 1 mm 2 m C. red light 2 mm 1 m D. green light 1 mm 2 m E. green light 2 mm 1 m

15. A coating material of refractive index 1.25

is used for the blooming of a lens having a larger refractive index. For normal incidence, if green light is to be transmitted in its greatest amount through the lens, which of the following thicknesses of the coating would do?

(Given : wavelength of green light in air is 550 nm)

Page 167: AL Physics 1981-2004 MC

95 AL Physics/M.C./P.4

(1) 137.5 nm (2) 220 nm (3) 330 nm A. (1) only B. (3) only C. (1) and (2) only D. (2) and (3) only E. (1), (2) and (3) 16. An astronomical refracting telescope

consists of two converging lenses of focal lengths 100 cm and 5 cm. Under normal adjustment, it is used to observe a distant object which subtends an angle of 0.2º when viewed directly. Which of the following statements is/are correct?

(1) The lens with focal length 5 cm

should be the objective. (2) The height of the first image formed

by the telescope is 3.5 mm. (3) The angle subtended by the final

image seen by the observer is 4º. A. (1) only B. (3) only C. (1) and (2) only D. (2) and (3) only E. (1), (2) and (3) 17. When an object placed far away from a

convex lens is gradually moved towards the lens, the separation between the object and its real image will

A. decrease. B. decrease and then increase. C. increase. D. increase and then decrease. E. remain unchanged. 18. An aircraft flies near the earth’s surface

over a stationary observer on a windless day. The frequency of the notes from the engine received by the observer is 300 Hz when approaching, and becomes 150 Hz when leaving. Assume the speed of sound in air to be 336 m/s. The speed of the aircraft is

A. 56 m/s B. 84 m/s C. 112 m/s D. 168 m/s E. 224 m/s

19. Two loudspeakers are connected to the

same signal source of negligible impedance. At a point equidistant from the two loudspeakers, a maximum intensity of intensity level 60 dB is detected. If one loudspeaker is disconnected, the intensity level at that point will be

A. 57 dB B. 54 dB C. 46 dB D. 30 dB E. 15 dB 20. The pressure of an ideal gas in a container

is P. If the number of gas molecules is halved, the volume of the container is doubled and the temperature is kept constant, the pressure will be

A. P/4 B. P/2 C. P D. 2P E. 4P 21. The r.m.s. speed of the molecules of a

certain gas X is 341 m/s at 298 K. Find the molar mass of the gas X.

(Given: Universal gas constant R = 8.31 J mol-1 K-1)

A. 5.4 g B. 21.8 g C. 33.8 g D. 42.6 g E. 63.9 g 22. A uniform wire is clamped at both ends

which are 0.4 m apart. The tension in the wire is 65 N and the area of its cross-section is 4.5 × 10-7 m2. When the wire is plucked to produce transverse vibrations, the frequency of the fundamental note obtained is 160 Hz. The density of the wire is

A. 5 600 kg/m3 B. 8 800 kg/m3 C. 12 600 kg/m3 D. 14 000 kg/m3

E. 22 000 kg/m3

Page 168: AL Physics 1981-2004 MC

95 AL Physics/M.C./P.5

23.

3 V

1 V2

4

Ω

Ω

Y

X Two cells of negligible internal resistance

are connected with two resistors as shown. What is the potential difference between X and Y?

A. 1.33 V B. 1.67 V C. 2.00 V D. 2.33 V E. 2.67 V 24. In which of the following graphs does the

area of the shaded part represent energy? (1)

0

σ

ε

ε = strain in a wire

σ = stress applied to the wire

(2)

0

Φ

Ι

Ι = current passing through a coilΦ = magnetic flux throguh the coil

(3)

0 Q

= charge on a capacitor

= p.d. between the plates

V Q

V

A. (1) only B. (3) only C. (1) and (2) only D. (2) and (3) only E. (1) and (3) only

25. A 1 µF capacitor, which is initially

charged, is discharged through a 50 Ω resistor. The maximum current during discharge is 100 A. What is the initial charge on the capacitor?

A. 5 × 10-3 C B. 1 × 10-4 C C. 5 × 10-4 C D. 1 × 10-3 C E. 2 × 10-6 C 26. Two uncharged metal spheres, A and B,

supported by insulating stands are placed side by side but not touching each other. A student places a positively-charged rod near sphere A and he touches sphere B with his finger momentarily. When the rod is removed afterwards, what are the signs of the charges induced on the spheres?

Sphere A Sphere B A. positive neutral B. negative negative C. negative neutral D. neutral positive E. neutral negative 27.

A B C D A, B, C, D are four points on a straight line

as shown. A point charge +Q is fixed at A. When another point charge -Q is moved from B to C, which of the following quantities will increase?

(1) The electric potential energy of the

system of charges. (2) The magnitude of the electric field

strength at the point D. (3) The electric potential at the point D. A. (1) only B. (3) only C. (1) and (2) only D. (2) and (3) only E. (1), (2) and (3) 28. The mechanical power output of a d.c.

motor is always less than the electrical

Page 169: AL Physics 1981-2004 MC

95 AL Physics/M.C./P.6

power input. Which of the following can be an explanation of this?

(1) heat loss in the coil of the motor (2) work done against friction (3) work done against the back e.m.f.

generated by the rotating coil A. (1) only B. (3) only C. (1) and (2) only D. (2) and (3) only E. (1), (2) and (3) 29. A ‘black box’ containing two unknown

components is connected to a cell, a resistor and an ammeter as shown.

black box

A

A current flows steadily no matter which

way the box’s terminals are connected to the cell and the same ammeter reading is obtained. The two components in the ‘black box’ could be

A. two diodes in series. B. two diodes in parallel. C. two capacitors in parallel. D. a diode and a resistor in parallel. E. a diode and a capacitor in parallel. 30.

generatorfactorymachines

The machines in a factory consume 10 kW

of electrical power at a voltage of 500 V. If the generator is connected to the factory through cables of total resistance 0.2 Ω, the e.m.f. produced by the generator should be

A. 500 V B. 501 V C. 502 V D. 504 V E. 508 V

31.

+_

12 V

JQP

L1 2L In the above circuit, PQ is a uniform

resistance wire connected to a 12 V constant voltage source. L1 and L2 are two identical light bulbs, each of rating 12 V, 8 W. What is the power dissipated by L1 when the sliding contact J is at the mid-point of PQ?

A. 1 W B. 2 W C. 4 W D. 6 W E. 8 W 32. In a Hall probe, the slice of semiconductor

inside has 1025 charge-carriers per cubic metre. When a steady current of 0.4 A passes through the slice and a uniform magnetic field of 0.1 T applies perpendicularly to it, a Hall voltage of 20 µV is set up. Find the thickness of the slice.

(Given: electronic charge = 1.6 × 10-19 C) A. 0.9 × 10-3 m B. 1.1 × 10-3 m C. 1.3 × 10-3 m D. 1.5 × 10-3 m E. 1.7 × 10-3 m 33. A uniform wire is stretched under tension.

The strain in the wire depends on (1) the Young modulus of the wire. (2) the cross-sectional area of the wire. (3) the unstretched length of the wire. A. (1) only B. (3) only C. (1) and (2) only D. (2) and (3) only E. (1), (2) and (3) 34.

Page 170: AL Physics 1981-2004 MC

95 AL Physics/M.C./P.7

uniform magnetic

field (out of paper)

BY

AX

Particles A and B moving at the same

speed enter a square region of uniform magnetic field as shown. Particle A leaves at X while particle B leaves at Y. If the charge to mass ratio of particle A is k, what is that of particle B?

A. k/2 B. k/4 C. k D. 2k E. 4k 35. A magnet is moved along a perpendicular

direction towards a sheet of copper. Which of the following statements is/are correct?

(1) Eddy current flows in the sheet. (2) Temperature of the sheet increases. (3) Repulsive force is experienced by the

magnet. A. (1) only B. (3) only C. (1) and (2) only D. (2) and (3) only E. (1), (2) and (3) 36. When light of frequency f1 is shone on to a

metal surface, the maximum energy of the electrons emitted is E1. If the same surface is illuminated with light of frequency f2, the maximum energy of the electrons emitted is E2. The Planck constant is given by

A. f E f E

f f2 1 1 2

1 2

+

B. f E f E

f f2 1 1 2

1 2

C. E Ef f1 2

1 2

++

D. E Ef f1 2

1 2

−+

E. E Ef f1 2

1 2

−−

37. In an LCR series circuit, the total

impedance across the three components is the same at both frequencies 25 Hz and 225 Hz. What is the resonant frequency of the circuit?

A. 50 Hz B. 75 Hz C. 100 Hz D. 125 Hz E. 150 Hz 38. When an electron in an atom undergoes a

transition from a higher energy level to a lower one, the time taken is about 10-9 s. Which of the following statements about the transition is/are correct?

(1) Electromagnetic radiation is emitted

during the transition. (2) The length of the wave train from

such a transition is about 0.3 m. (3) The energy of the photon emitted

depends on the energy difference between the two levels.

A. (1) only B. (3) only C. (1) and (2) only D. (2) and (3) only E. (1), (2) and (3) 39.

N2 turnsarea A 2

n 1 turns per metrearea A 1

I I

A solenoid of n1 turns per metre and cross-

sectional area A1 carries a current I. It is inserted into the core of another larger solenoid of N2 turns and cross-sectional area A2. If the current in the smaller solenoid drops uniformly to zero in time t, what is the e.m.f. induced in the larger solenoid?

A. 2µ0n1IA1N2/t

Page 171: AL Physics 1981-2004 MC

95 AL Physics/M.C./P.8

B. 2µ0n1IA2N2/t C. µ0n1IA1A2/t D. µ0n1IA2N2/t E. µ0n1IA1N2/t 40.

to CROLC

A B

S

A capacitor C and an inductor L with some

resistance are connected to a battery as shown in figure. When switch S is moved from A to B, what is the trace observed on the screen of the CRO with the time base on?

A.

B.

C.

D.

E.

41.

reactivecomponent

R

An a.c. power source of negligible

impedance is connected in series with a reactive component and a resistor of resistance R. At the source frequency, the source voltage has r.m.s. value V, the component’s reactance is X and the circuit’s total impedance is Z. The power consumption in the circuit is

A. VR

2

B. VZ

2

C. V RZ

2

2

D. V RX

2

2

E. V XZ

2

2

42. The diagram below shows the trace on the

screen of a CRO when a sinusoidal signal of frequency 75 Hz was applied to the Y-plates.

What time base, in ms per division, was

the CRO set at? A. 1 B. 2 C. 5 D. 10 E. 20 43. An operational amplifier is connected as

shown below with input voltage Vi = +2V.

Page 172: AL Physics 1981-2004 MC

95 AL Physics/M.C./P.9

_

+

+15 V

-15 V

60 k Ω

10 k Ω

20 k Ω

V iV o

0 V

What is the output voltage Vo? A. -6 V B. -4 V C. +4 V D. +6 V E. +12 V 44. 88

226 Ra decays to 86222 Rn with a half-life of

1 600 years. Which of the following statements is/are correct?

(1) α particle is produced in the decay. (2) All 88

226 Ra has decayed after 3 200 years.

(3) The half-life of 88226 Ra can be

shortened by heating. A. (1) only B. (3) only C. (1) and (2) only D. (2) and (3) only E. (1), (2) and (3) 45. In an experiment to measure the density of

steel, a steel sphere was used. The following measurements were obtained:

Mass of the sphere = 530 mg ± 1 mg Diameter of the sphere = 0.51 cm ± 0.01 cm Estimate the percentage error in the

calculated value of the density of steel. A. 1% B. 2% C. 4% D. 6% E. 8%

- End of Paper -

Page 173: AL Physics 1981-2004 MC

96 AL Physics/M.C./P.1

1996 Hong Kong Advanced Level Examination AL Physics

Multiple Choice Question

1. Which of the following pairs of quantities of a moving object must be in the same direction?

(1) acceleration and change in

momentum (2) displacement and velocity (3) velocity and acceleration A. (1) only B. (3) only C. (1) and (2) only D. (2) and (3) only E. (1), (2) and (3) 2. In which of the following cases is the

resultant force on the object zero? (1) a satellite moving round the earth (2) a feather falling freely in a vacuum

cylinder in a laboratory (3) a gas bubble rising with terminal

velocity in water A. (1) only B. (3) only C. (1) and (2) only D. (2) and (3) only E. (1), (2) and (3) 3. Two satellites of the same mass travel

around the earth in circular orbits of different radii. The satellite in the orbit with smaller radius has

A. a greater speed. B. a longer period. C. a smaller acceleration towards the

earth’s centre. D. a greater angular momentum about

the earth’s centre. E. a greater sum of gravitational

potential energy and kinetic energy. 4. An object is thrown vertically upward and

experiences an air resistance opposing its motion with magnitude proportional to its speed. Which of the following graphs

best represents the variation of the acceleration, a, of the object with time, t, starting from the moment when the object leaves one’s hand up to the time when it returns to the ground? (Take downward as positive.)

A.

a

t

g

- g

0

B.

a

t

g

- g

0

C.

a

t

g

- g

0

D.

a

t

g

- g

0

E.

a

t

g

- g

0

Page 174: AL Physics 1981-2004 MC

96 AL Physics/M.C./P.2

5.

B

A

The figure shows two blocks A and B, each of mass m, connected by two light springs to a fixed support. Each spring has a force constant k. What is the total extension of the system when it is at static equilibrium?

A. mg/(2k) B. mg/k C. 3mg/(2k) D. 2mg/k E. 3mg/k 6. X, Y and Z are three spheres of the same

size but with masses m, m and 2m respectively, lying on a smooth horizontal track with Y and Z in contact as shown. X is moving to the right with velocity u and makes a head-on collision with Y. If all collisions are perfectly elastic, which of the following gives the possible velocities of the three spheres after all collision? (Take to the right as positive.)

X Y Z A. 0 0 u/2 B. 0 u/3 u/3 C. -u/3 0 2u/3 D. 0 -u/3 2u/3 E. u/4 u/4 u/4 7.

A small object P of mass 0.3 kg is

attached to one end of a light, rigid rod of length 0.5 m, which is free to rotate about the other end O as shown. The object is swung to rotate in a vertical circle so that it attains a speed of 2 ms-1 at its topmost position. What is the force exerted on one end of the rod at this instant?

A. a compressive force of 0.6 N B. a tensional force of 0.6 N C. a tensional force of 2.4 N D. a tensional force of 5.4 N E. a compressive force of 5.4 N 8. Three uniform wires, of same length and

mass but with different shapes, are hinged to an axis passing through O and perpendicular to the plane of the paper as shown. Arrange their moments of inertia about O in ascending order of magnitude.

(1) (2) (3) A. (1), (2), (3) B. (2), (1), (3) C. (1), (3), (2) D. (2), (3), (1) E. (3), (2), (1) 9. A system A oscillating at its natural

frequency fA is coupled to a system B of natural frequency fB, and causes system B to oscillate. When the steady state is reached, which of the following statements is/are correct?

(1) System B oscillates at frequency fA. (2) The amplitude of system B depends

on the difference of fA and fB. (3) The rate of transfer of energy from

system A to B is high when fA is close to fB.

A. (1) only B. (3) only C. (1) and (2) only D. (2) and (3) only E. (1), (2) and (3)

O O O

m m 2m

X Y Z u

0.5 m

O

P

Page 175: AL Physics 1981-2004 MC

96 AL Physics/M.C./P.3

10. A metre rule is clamped horizontally to the edge of a bench so that most of its length overhangs and it is free to vibrate with vertical simple harmonic motion. The tip of the rule vibrates with an amplitude of 3.5 cm and a maximum speed of 1.0 ms-1. What is the frequency of vibration of the rule?

A. 3.0 Hz B. 3.5 Hz C. 4.0 Hz D. 4.5 Hz E. 5.0 Hz 11. The least separation between two points

with a phase difference of π6

on a

progressive wave is 0.05 m. If the velocity of the wave is 150 ms-1, the frequency of the wave is

A. 125 Hz. B. 250 Hz. C. 375 Hz. D. 500 Hz. E. 1 000 Hz. 12. When monochromatic light travels from

glass to air, the emergent light, relative to the incident light, shows an increase in

(1) frequency. (2) wavelength. (3) velocity. A. (1) only B. (3) only C. (1) and (2) only D. (2) and (3) only E. (1), (2) and (3) 13. When a beam of unpolarized light

travelling in air falls on a water surface, it is possible to find an angle of incidence such that

(1) all the light is reflected. (2) the light passing into water is plane

polarized. (3) the reflected light is plane polarized. A. (1) only B. (3) only

C. (1) and (2) only D. (2) and (3) only E. (1), (2) and (3) 14.

θ

Monochromatic light of wavelength 589

nm is shone normally on a wedge-shaped thin air film to form interference fringes. If the distance between the sixth and ninety-sixth dark fringes is 15.8 mm, calculate the angle of the wedge, θ.

A. 0.024° B. 0.048° C. 0.072° D. 0.096° E. 0.190° 15.

second-orderspectrum

first-orderspectrum

zeroth orderwhite

light

grating

SR

QP

α

β

20°

A beam of white light is shone normally

on a diffraction grating. The diagram shows the spectra of the first two orders, which may not be drawn to scale. The first-order spectrum starts at an angle of 20° from the zeroth order. The respective angular separations between the two ends (red and violet) of a spectrum are α and β for the first- and second- order spectra. Which of the following statements is/are correct?

(1) In the first-order spectrum, P is the

violet end. (2) β is greater than α. (3) There is no third-order spectrum. A. (1) only B. (3) only C. (1) and (2) only D. (2) and (3) only E. (1), (2) and (3)

Page 176: AL Physics 1981-2004 MC

96 AL Physics/M.C./P.4

16. The straight lines in the diagram represent electric field lines. Which of the following statements about this electric field is/are correct?

(1) A stationary negative charge placed at

Q tends to move to P. (2) The electric field strength at P is

stronger than that at Q. (3) Work has to be done in moving a

negative charge from R to P. A. (1) only B. (3) only C. (1) and (2) only D. (2) and (3) only E. (1), (2) and (3) 17. Which of the following statements is/are

correct? (1) Radio waves can be polarized. (2) Ultrasonic waves are electromagnetic

in nature. (3) All standing waves are transverse

waves. A. (1) only B. (3) only C. (1) and (2) only D. (2) and (3) only E. (1), (2) and (3) 18.

LM

O

80 cm

A small object O is placed at the focus of

a convex lens L as shown. A concave mirror M is placed 80 cm behind the lens. If the focal length of L is 50 cm and the final image formed by this system is at infinity, the focal length of M is

A. 20 cm

B. 30 cm C. 40 cm D. 50 cm E. 60 cm 19. The image of the moon is focused on a

screen by a converging lens of focal length 20 cm. If the moon subtends an angle of 9.5 × 10-3 radian at the centre of the lens, calculate the diameter of the image.

A. 4.8 × 10-2 cm B. 9.5 × 10-2 cm C. 1.9 × 10-1 cm D. 3.8 × 10-1 cm E. 7.6 × 10-1 cm 20. 3-cm microwaves are used in a radar

speed trap. When the emitted and reflected signals are superposed, beats are produced. Find the minimum beat frequency above which a driver would be prosecuted for exceeding a speed limit of 100 kmh-1.

A. 1 000 Hz B. 1 900 Hz C. 2 400 Hz D. 3 400 Hz E. 4 700 Hz 21. A parallel-plate capacitor is charged by a

constant voltage source. With the plates remaining connected to the source, the plate separation, d, is then gradually increased from an initial value d0. Which of the following graphs best shows the relationship between the energy, U, stored in the capacitor and plate separation, d?

A.

U

dd

0

0

P

Q

R

Page 177: AL Physics 1981-2004 MC

96 AL Physics/M.C./P.5

B.

U

dd

0

0

C.

U

dd

0

0

D.

U

dd

0

0

E.

U

dd

0

0

22. A charged capacitor is connected across a

resistor. Its voltage drops to one-third of the original value in 10 s. Find the time constant of the circuit.

A. 4.1 s B. 9.1 s C. 11 s D. 25 s E. 30 s 23. Five identical point charges, each of

charge Q, are fixed evenly on a circle of radius r. How much work has to be done to bring another point charge Q from infinity to the centre of the circle? (ε0 = permittivity of free space)

A. zero

B. 54 0

Qrπε

C. 54

2

0

Qrπε

D. 54

2

02

Qrπε

E. 54 0

2Q

rπε

24. In the above circuit, the galvanometer

reading is zero. If both cells have negligible internal resistance, the e.m.f. of the cell E is

A. 6 V. B. 7 V. C. 8 V. D. 9 V. E. 10 V. 25. Six wires, each of resistance 1 Ω, are

joined to form a regular tetrahedron ABCD. A current of 2 A flows across the tetrahedron at A and B.

2 A

2 A

A

B

C

D Which of the following statements is/are

correct? (1) Points C and D are of the same

potential. (2) The current in wire AC is 0.5 A. (3) The potential drop across AB is 1 V. A. (1) only B. (2) only C. (1) and (2) only

1 kΩ

2 kΩ 500 Ω

E 3 V

G

Page 178: AL Physics 1981-2004 MC

96 AL Physics/M.C./P.6

D. (2) and (3) only E. (1), (2) and (3) 26. Four infinitely long straight parallel wires

P, Q, R, S carrying equal currents are situated at the corners of a square as shown. The currents in P, Q are into paper and those in R, S are out of paper. What is the direction of the resultant magnetic induction at the centre of the square?

A. I B. II C. III D. IV E. V 27.

soft iron core

S

load

magneticflux

The diagram shows the circuit of an ideal

transformer with a load connected to the secondary. Which of the following is/are correct at the moment when switch S is closed?

(1) The magnetic flux in the soft iron

core will be reduced. (2) The back e.m.f. in the primary will

drop. (3) The primary current will increase. A. (1) only B. (3) only C. (1) and (2) only D. (2) and (3) only E. (1), (2) and (3)

28. In the above circuit, both the cell E and

the inductor L have negligible resistance. The e.m.f. of E is 10 V, and the resistors R1 and R2 are of resistances 2 Ω and 3 Ω respectively. What will the current drawn from the cell be

(I) at the moment when switch S is

closed? (II) when the steady state is reached after

closing switch S? (I) (II) A. 2 A 0 A B. 2 A 2 A C. 2 A 5 A D. 5 A 0 A E. 0 A 5 A 29.

time

V

0t

1t

2

V

C

A sinusoidal voltage V is applied across a

capacitor C. Which of the following statements is/are correct?

(1) At time t1, the charge in the capacitor

is at a maximum. (2) At time t2, the current in the circuit is

at a maximum. (3) The current in the circuit leads the

voltage V. A. (1) only B. (3) only C. (1) and (2) only D. (2) and (3) only E. (1), (2) and (3)

E S

R1

R2 L Q P

R S

II III

IV

I

V

Page 179: AL Physics 1981-2004 MC

96 AL Physics/M.C./P.7

30.

A

B output

Which of the following pairs of inputs

would give a HIGH output from the above combination of NAND gates?

Input A Input B (1) LOW HIGH (2) LOW LOW (3) HIGH HIGH A. (1) only B. (3) only C. (1) and (2) only D. (2) and (3) only E. (1), (2) and (3) 31. In the above transistor circuit, the voltage

across the base and the emitter is 0.5 V when the transistor works. The current amplification factor of the transistor is 80. What input voltage (Vin) would give an output voltage (Vout) of 4 V?

A. 0.25 V B. 0.50 V C. 0.75 V D. 1.00 V E. 1.25 V 32.

An input voltage (Vin) of 2.0 V is applied

to an ideal operational amplifier connected as shown. The current flowing through the 8 kΩ resistor is

A. 0.25 mA from X to Y. B. 0.80 mA from X to Y. C. 0.80 mA from Y to X. D. 1.0 mA from X to Y. E. 1.0 mA from Y to X. 33.

y

A

OB

magnetic fieldout of paper

magnetic fieldinto paper

Two uniform magnetic fields of equal

magnitude but with one into paper and the other out of paper, are separated on the two sides of the y-axis as shown. A wire is bent into a closed loop OABO, which is shaped as a quarter of a circle. It is rotated uniformly in a clockwise direction about O on the plane of the paper. Which of the following graphs best shows the variation of current, I, in the loop with time, t, in one revolution?

A.

t

I

0

B.

t

I

0

Vin Vout

+6 V

1 kΩ

10 kΩ

0 V

2 kΩ

-15 V

+15 V

8 kΩ

Y X

Vin

Page 180: AL Physics 1981-2004 MC

96 AL Physics/M.C./P.8

C.

t

I

0

D.

t

I

0

E.

t

I

0

34. The reactance of a solenoid depends on (1) the frequency of the applied a.c.

voltage. (2) the inductance of the solenoid. (3) the resistance of the solenoid. A. (1) only B. (3) only C. (1) and (2) only D. (2) and (3) only E. (1), (2) and (3) 35.

When a CRO is connected to a circuit, the

trace obtained is as shown above. To which of the following circuits has the CRO been connected?

A.

to CRO

B.

to CRO

C.

to CRO

D.

to CRO

E.

to CRO

36.

r

U

O

QP

The potential energy, U, of a pair of

atoms as a function of their separation, r, is shown for two crystalline solids P and Q. From these curves alone, one may conclude that

(1) the equilibrium separation of the

atoms in P is greater than that in Q. (2) P is stiffer than Q. (3) the energy required to separate two

atoms of P in equilibrium is more than that for Q.

A. (1) only B. (3) only C. (1) and (2) only D. (2) and (3) only E. (1), (2) and (3)

Page 181: AL Physics 1981-2004 MC

96 AL Physics/M.C./P.9

37. Which of the following statements is/are correct?

(1) The boiling point of pure water

increases with atmospheric pressure. (2) The water molecules in 100 °C steam

have a higher r.m.s. speed than those in pure water boiling at 100 °C.

(3) When pure water is boiling, the supplied energy becomes the intermolecular potential energy and the kinetic energy of the water molecules.

A. (1) only B. (3) only C. (1) and (2) only D. (2) and (3) only E. (1), (2) and (3) 38. Of the three common materials, copper,

glass and rubber, which two best illustrate the properties described in the following statements when each is stretched under room temperature?

(1) It obeys Hooke’s law almost up to its

breaking point. (2) It tolerates a large strain when

exhibiting elastic behaviour. (1) (2) A. copper glass B. copper rubber C. glass copper D. glass rubber E. rubber glass 39. A hypothetical atom has only four energy

levels as represented above. It can change from any one level to any other. Which of the following statements about this hypothetical atom is INCORRECT?

A. The most stable state of the atom corresponds to the level -12 eV.

B. If the atom is at the level -4 eV, it can absorb a photon of energy 4 eV.

C. If the atom is at the level -4 eV, it can emit a photon of energy 8 eV.

D. If the atom is at the level -12 eV and collides with an electron of kinetic energy 10 eV, it can change to the level -4 eV.

E. If the atom is at the level -12 eV and collides with two photons, each of energy 4 eV, it can change to the level -4 eV.

40.

θ

ball

air stream

BN

A

A steady stream of air is ejected from a nozzle N and keeps a small polythene ball staying in mid air as shown. The nozzle and the direction of air stream make an angle θ with the horizontal. Which of the following statements is/are correct?

(1) The ball spins in an anti-clockwise

direction. (2) Air flows faster on side A than side B

of the ball. (3) When angle θ increases, the ball

spins more slowly. A. (1) only B. (3) only C. (1) and (2) only D. (2) and (3) only E. (1), (2) and (3) 41. A radioactive source is tested as follows:

Absorber placed between source and GM

counter

Effect on count rate

(1) Thin aluminium foil (0.2 mm)

Falls appreciably.

(2) Thin lead sheet (2 mm)

No significant difference with (1).

(3) Thick lead sheet (20 mm)

Falls below that in (1).

-12 eV

-4 eV

-1 eV

0 eV energy

Page 182: AL Physics 1981-2004 MC

96 AL Physics/M.C./P.10

What type(s) of radiation does the source

emit? A. α only B. β only C. γ only D. α and γ only E. α and β only 42. A metal surface is illuminated with

monochromatic light so that it emits photoelectrons. The maximum kinetic energy of the emitted photoelectrons depends on

(1) the distance of the metal surface

from the light source. (2) the work function of the metal

surface. (3) the wavelength of the incident

monochromatic light. A. (1) only B. (3) only C. (1) and (2) only D. (2) and (3) only E. (1), (2) and (3) 43. Given: mass of photon = 1.0073 u mass of neutron = 1.0087 u binding energy of an alpha particle

= 28.396 MeV 1 u is equivalent to 931 MeV The mass of an alpha particle is A. 3.9100 u. B. 4.0015 u. C. 4.0320 u.

D. 4.0625 u. E. 4.1540 u. 44. A counter is placed near a very weak

radioactive source which has a half-life of 1 hour. The counter registers 100 counts/min at noon and 80 counts/min at 1 p.m. The expected count rate, in counts/min, at 3 p.m. on the same day is

A. 40 B. 50 C. 55 D. 60 E. 65 45. The period of oscillation, T, of a simple

pendulum is related to its length, l, by the

formula Tlg

= 2π . To find experimen-

tally the acceleration of free fall by using a simple pendulum, a student takes the following measurements:

time for 15 oscillations: 14.4 ± 0.2 s length of the pendulum: 0.229 ± 0.001 m Which of the following is the most

appropriate way of expressing the result? A. 9.8 ± 0.2 ms-2 B. 9.8 ± 0.3 ms-2 C. 9.81 ± 0.32 ms-2 D. 9.810 ± 0.179 ms -2 E. 9.810 ± 0.315 ms -2

- End of Paper -

Page 183: AL Physics 1981-2004 MC

96 AL Physics/M.C./P.11

Question No. Key 1. A 2. B 3. A 4. C 5. E 6. C 7. A 8. B 9. E 10. D 11. B 12. D 13. B 14. D 15. E 16. C 17. A 18. B 19. C 20. B 21. A 22. B 23. C 24. D 25. E

Question No. Key 26. A 27. E 28. C 29. E 30. A 31. C 32. E 33. D 34. C 35. E 36. E 37. A 38. D 39. E 40. D 41. D 42. D 43. B 44. E 45. B

Page 184: AL Physics 1981-2004 MC

97 AL Physics/M.C./P.1

1997 Hong Kong Advanced Level Examination AL Physics

Multiple Choice Question

1. Two wooden blocks A and B are

connected by a string which passes over a smooth, fixed pulley as shown. The maximum friction between any two surfaces is 2 N. If a horizontal force F is applied to block B, find its minimum value for moving B.

A. 2 N B. 4 N C. 6 N D. 8 N E. 10 N 2. A ball of mass m is projected vertically

downwards with speed v from a certain height and rebounds from the ground back to the same height. Which of the following statements is/are correct? (Neglect air resistance.)

(1) The collision between the ball and the

ground is not perfectly elastic. (2) The loss in energy of the ball in the

collision is ½mv2. (3) If the ball is projected vertically

upwards from the same height with the same speed, it would rebound to a greater height.

A. (1) only B. (3) only C. (1) and (2) only D. (2) and (3) only E. (1), (2) and (3) 3. A particle is projected horizontally from

a table with an initial speed u and attains a speed v just before hitting the ground. What is the time of flight of the particle? (Neglect air resistance.)

A. vg

B. v u

g−

C. v u

g

2 2

2−

D. v u

g

2 2−

E. It cannot be found as the vertical distance fallen is not known.

4.

O

A8 m

centre

20º

of mass

The figure shows a car moving round a

corner with a radius of 8 m on a banked road of inclination 20º. At what speed would there be no fiction acting on the car along OA?

A. 5.0 m/s B. 5.2 m/s C. 5.4 m/s D. 5.6 m/s E. 5.8 m/s 5. A ball bearing is released from rest at a

height h on a smooth track and completes the circular loop of the track. If R is the reaction acting on the ball bearing at the highest point A of the loop, which of the

h

A

A B F

Page 185: AL Physics 1981-2004 MC

97 AL Physics/M.C./P.2

following graphs correctly shows the variation of R with h?

A. B. C. D. E. 6. A nucleus originally at rest splits into two

fragments of unequal mass. The fragment with smaller mass has a larger

(1) momentum. (2) speed. (3) kinetic energy. A. (1) only B. (3) only C. (1) and (2) only D. (2) and (3) only E. (1), (2) and (3) 7. A constant external torque X is applied to

a flywheel which is initially at rest. The angular speed of the flywheel increases to

a certain value after 21 s. If the external torque is now doubled, the flywheel will acquire the same angular speed after 9 s. Find the average frictional torque exerted at the bearings of the flywheel.

A. 12

X

B. 37

X

C. 7

10X

D. 47

X

E. 14

X

8. A 50-g mass suspended from a light

helical spring oscillates with vertical simple harmonic motion of amplitude 2.5 cm. If the maximum kinetic energy of the mass is 3.0 × 10-3 J, its frequency of oscillation is

A. 0.4 Hz B. 0.8 Hz C. 1.1 Hz D. 1.9 Hz E. 2.2 Hz 9. When a body performs a simple harmonic

motion, which of the following is/are correct?

(1) Displacement from the equilibrium

position is π/2 out of phase with the velocity.

(2) Acceleration is π/2 out of phase with the velocity.

(3) Displacement from the equilibrium position is in phase with the acceleration.

A. (1) only B. (3) only C. (1) and (2) only D. (2) and (3) only E. (1), (2) and (3) 10. A low-altitude satellite near the earth’s

surface has a speed of 7.90 km/s. The radius of the earth is about 4 times that of the moon and the ratio of the average density of the earth to that of the moon is

0 h

R

h 0

R

0 h

R

0 h

R

0 h

R

Page 186: AL Physics 1981-2004 MC

97 AL Physics/M.C./P.3

about 5 : 4. The speed of a low-altitude satellite near the moon’s surface would be

A. 1.77 km/s B. 2.21 km/s C. 2.47 km/s D. 3.57 km/s E. 4.42 km/s 11.

P

Q R

S T

The above figure shows a transverse wave

propagating to the right along a string. At the moment shown, which of the labelled particles has its velocity and acceleration in opposite directions?

A. P B. Q C. R D. S E. T 12.

incident pulse

Three identical long spiral springs are

connected respectively to a wall, a light thread and a heavy spring. If a crest-shaped pulse is sent along each spiral spring towards the respective boundary, which of the following diagrams correctly show(s) the reflected pulse? (Transmitted pulse is not shown.)

(1)

wall

(2)

light thread

(3)

heavy spring

A. (1) only B. (3) only C. (1) and (2) only D. (2) and (3) only E. (1), (2) and (3) 13. A beam of monochromatic light passes

through three media of refractive indexes n1, n2 and n3 respectively as shown. The boundaries between the media are parallel.

Which of the following relations between

n1, n2 and n3 is correct? A. n1 > n3 > n2 B. n3 > n1 > n2 C. n1 > n2 > n3 D. n2 > n1 > n3 E. n3 > n2 > n1 14. A moving ship sounds a foghorn and an

echo is received from a cliff behind it. Which of the following wave characteristics of the original sound is/are different from that of the echo?

(1) frequency (2) amplitude (3) speed A. (1) only B. (3) only C. (1) and (2) only D. (2) and (3) only E. (1), (2) and (3) 15. In a Young’s double-slit experiment, light

of wavelength 400 nm is used. If the path

X

Y light source

P

screen

n1

n2

n3

Page 187: AL Physics 1981-2004 MC

97 AL Physics/M.C./P.4

difference between the light from the two slits X and Y to a point P on the screen is 3000 nm, which of the following is/are correct?

(1) P is the 7th dark fringe. (2) The fringe separation on the screen

increases if the light source is moved closer to the slits.

(3) P becomes a bright fringe if light of wavelength 500 nm is used.

A. (1) only B. (3) only C. (1) and (2) only D. (2) and (3) only E. (1), (2) and (3) 16. Two pipes A and B, with pipe A closed at

one end and pipe B open at both ends, are sounded to give fundamental notes of the same frequency. The length ratio of pipe A to pipe B is

A. 1 : 4 B. 1 : 2 C. 1 : 1 D. 2 : 1 E. 4 : 1 17. In an arrangement for viewing Newton’s

rings, which of the following would be observed if water is introduced to the space between the lens and the glass plate?

A. The rings disappear. B. The central spot becomes bright. C. The separation between the rings

increases. D. The rings shift towards the centre. E. There is no observable change. 18.

The above diagram shows two light rays

from a point source O passing in turn through a convex lens L1 and a concave lens L2. Which of the following is true of the focal lengths of the lenses?

Focal length of L1 Focal length of L2 A. 5 cm less than 10 cm B. 10 cm less than 10 cm C. 10 cm 10 cm D. 5 cm greater than 10 cm E. 10 cm greater than 10 cm 19. A plane transmission grating is placed at

the centre of a circular 0º - 360º protractor. A beam of monochromatic light is incident normally on the grating. The zeroth-order maximum occurs at a scale reading of 90º and a first-order maximum occurs at a scale reading of 65º. At what scale reading would a second-order maximum be observed?

A. 148º B. 140º C. 130º D. 58º E. 40º 20. The diagram below shows the stress-

strain graphs up to the breaking point for two cylindrical rods made from materials P and Q respectively, which have the same diameter and original length.

tensile

0

P

Q

tensile strain

stress

Which of the two materials (1) is the more ductile (2) exhibits greater stiffness (3) has the greater strength? (1) (2) (3) A. Q P Q B. P Q P C. Q P P D. P P Q E. Q Q P

10 cm 10 cm 10 cm 10 cm

O

L1 L2

Page 188: AL Physics 1981-2004 MC

97 AL Physics/M.C./P.5

21. Which of the following quantities can be increased by increasing the voltage across a metal wire?

(1) the average drift velocity of the

conduction electrons (2) the number of conduction electrons

per unit volume (3) the speed of the electrical signal A. (1) only B. (3) only C. (1) and (2) only D. (2) and (3) only E. (1), (2) and (3) 22. A metal sphere is charged to a potential

of 100 V. If the charge density on its surface is 6.0 × 10-9 Cm-2, find the radius of the sphere.

(Given: permittivi ty of free space = 8.85 × 10-12 Fm-1)

A. 0.04 m B. 0.15 m C. 0.21 m D. 0.35 m E. 0.60 m 23.

N S

coil

to light beam galvanometer The above figure shows a small coil,

connected to a light beam galvanometer, placed in a region of uniform magnetic field between the poles of a magnet. The plane of the coil is parallel to the pole faces. Which of the following actions would produce a deflection of the galvanometer?

(1) Moving the coil to and fro between

the poles. (2) Moving the coil away from the region

between the pole faces. (3) Rotating the coil about a diameter

through an angle of 180º. A. (1) only B. (3) only C. (1) and (2) only D. (2) and (3) only E. (1), (2) and (3)

24. A variable capacitor can be used (1) as the volume control of an amplifier. (2) to tune a radio to receive a certain

channel. (3) to change the power factor of an a.c.

circuit. A. (1) only B. (3) only C. (1) and (2) only D. (2) and (3) only E. (1), (2) and (3) 25. Two cylindrical metal rods, X and Y, are

made from the same material and have the same mass. The length of X is three times that of Y. If currents of 1 A and 2 A pass through X and Y respectively, the ratio of the power dissipation in X to that in Y is

A. 9 : 1 B. 1 : 4 C. 9 : 2 D. 3 : 4 E. 9 : 4 26. A d.c. motor is connected to a source of

constant voltage V. When rotating at a steady speed, the motor draws a current I and the back e.m.f. developed in its armature coil is ε. Which of the following statements is/are correct?

(1) The resistance of the armature coil is

V/I. (2) The mechanical power developed is

Iε. (3) The back e.m.f. ε increases with the

speed of rotation. A. (1) only B. (3) only C. (1) and (2) only D. (2) and (3) only E. (1), (2) and (3) 27. Which of the following combinations of

physical quantities take(s) the unit of time?

(1) capacitance times resistance (2) inductance divided by resistance (3) capacitance times inductance

Page 189: AL Physics 1981-2004 MC

97 AL Physics/M.C./P.6

A. (1) only B. (3) only C. (1) and (2) only D. (2) and (3) only E. (1), (2) and (3) 28. A charged particle is accelerated across

the gap between two parallel metal plates maintained at a certain potential difference in a vacuum. Assuming there is no gravitational force, the energy acquired by the charged particle in crossing the gap depends on

(1) the mass of the charged particles. (2) the width of the gap. (3) the potential difference between the

plates. A. (1) only B. (3) only C. (1) and (2) only D. (2) and (3) only E. (1), (2) and (3) 29. A cell of e.m.f. 1.5 V is connected in

series with a variable resistor of resistance R and an ammeter A of resistance 0.5 Ω. By varying R, a series of ammeter readings, I, are taken and a

graph of 1I

against R is plotted. The

value of the y-intercept is found to be 0.88 A-1. What is the internal resistance of the cell?

A. 0.59 Ω B. 0.82 Ω C. 1.14 Ω D. 1.20 Ω E. 1.32 Ω

30.

6 V

A

B

CR1 R2

R3 R4

In the above circuit, the cell has an e.m.f. 6 V and negligible internal resistance. The resistances of the four resistors R1, R2, R3 and R4 are 6 Ω, 3 Ω, 1 Ω and 2 Ω respectively. The capacitor has capacitance 20 µF. At steady state, the charge on the capacitor plate connecting to A is

A. +40 µC B. +80 µC C. 0 µC D. -80 µC E. -40 µC 31. A light rectangular wire frame ABCD

moves with a uniform speed to the left across a region of uniform magnetic field acting into paper. Which of the following is/are true at the instant shown in the figure?

(1) A current is flowing in the clockwise

direction. (2) The electric potential at B is higher

than that at C. (3) The side AD experiences a magnetic

force acting to the right. A. (1) only B. (3) only C. (1) and (2) only D. (2) and (3) only E. (1), (2) and (3)

A R

A

D

B

C

Page 190: AL Physics 1981-2004 MC

97 AL Physics/M.C./P.7

32. A circuit is required for doubling the amplitude of a sinusoidal alternating voltage. Which of the following circuits might be suitable?

(1) (2) (3) A. (1) only B. (3) only C. (1) and (2) only D. (2) and (3) only E. (1), (2) and (3) 33. Only a small fraction of the electrical

energy supplied to a light bulb is converted into light energy. Which of the following explanations is/are correct?

(1) Energy is absorbed by the glass wall

of the bulb. (2) The resistance of the filament of the

bulb is very high. (3) Most of the energy is emitted as

infra-red radiation. A. (1) only B. (3) only C. (1) and (2) only D. (2) and (3) only E. (1), (2) and (3)

34. In a horizontal pipeline of cross-sectional area 2.5 × 10-3 m2, the static pressure and total pressure at a certain point are 0.6 × 104 Pa and 1.0 × 104 Pa respectively. The fluid flowing through it is incompressible, non-viscous and has a density of 1.2 × 103 kgm-3. The volume flow rate in the pipeline is

A. 3.2 × 10-3 m3s-1

B. 6.5 × 10-3 m3s-1

C. 7.9 × 10-3 m3s-1

D. 1.0 × 10-2 m3s-1

E. 1.7 × 10-2 m3s-1

35. Two different ideal gases, A and B, are

contained in two identical vessels. If the ratio of their absolute temperature and the ratio of the root-mean-square speed of the molecules are respectively 2 : 1 and 3 : 1, the ratio of their molecular mass is

A. 2 : 3 B. 2 : 9 C. 1 : 6 D. 9 : 2 E. 3 : 2 36. A fixed mass of an ideal gas undergoes a

cycle ABCD in which its pressure p and volume V change as shown in the p-V diagram below.

A

B

D C

p

V0

Which of the following statements is/are

correct? (1) From A to B, work is done by the gas. (2) From B to C, heat is extracted from

the gas. (3) From C to D, the internal energy of

the gas increases. A. (1) only B. (3) only C. (1) and (2) only

input output

500 turns

1000 turns

output

input

5 kΩ

5 kΩ

output input 10 kΩ

10 kΩ

Page 191: AL Physics 1981-2004 MC

97 AL Physics/M.C./P.8

D. (2) and (3) only E. (1), (2) and (3) 37. When a gas is below its critical

temperature, which of the following is/are correct?

(1) Intermolecular forces are significant. (2) The gas can be liquefied by applying

pressure alone. (3) Collisions between molecules are

not perfectly elastic. A. (1) only B. (3) only C. (1) and (2) only D. (2) and (3) only E. (1), (2) and (3) 38. The instantaneous value of an alternating

current I (in A) at time t (in s) is given by the equation

I t= 2 2 50sin π Which of the following descriptions

is/are correct? (1) The a.c. frequency is 50 Hz. (2) The value of the current varies

between − 2 2 A and + 2 2 A. (3) The r.m.s. value of the current is 2 A. A. (1) only B. (3) only C. (1) and (2) only D. (2) and (3) only E. (1), (2) and (3) 39.

incidentmonochromatic

radiation

d.c. source

A

C

A d.c. source is applied to a photocell as

shown. Monochromatic radiation is incident on cathode C so that photoelectrons are emitted from the cathode surface. The maximum kinetic

energy of the photoelectrons reaching anode A can be increased by using

(1) a d.c. source of higher voltage. (2) monochromatic radiation of longer

wavelength. (3) the same monochromatic radiation

but higher of intensity. A. (1) only B. (3) only C. (1) and (2) only D. (2) and (3) only E. (1), (2) and (3) 40. In the above circuit, a signal generator of

fixed output voltage is connected in series with a resistor R and a capacitor C. When the frequency of the signal generator is gradually increased, which of the following is the correct variation of the ammeter and voltmeter readings?

Ammeter reading Voltmeter reading A. increases increases throughout throughout B. increases decreases throughout throughout C. increases increases and throughout then decreases D. increases and decreases then decreases throughout E. increases and decreases and then decreases then increases

signal generator

R

C

A V

Page 192: AL Physics 1981-2004 MC

97 AL Physics/M.C./P.9

41. The transition of electrons between three energy levels in a particular atom gives rise to three spectral lines. The shortest and longest wavelengths of those spectral lines are λ1 and λ2 respectively. The wavelength of the other spectral line is

A. λ λ1 2

2+

B. λ λ2 1−

C. λ λ

λ λ1 2

1 2+

D. ( )1 1

1 2

1

λ λ− −

E. λ λ1 2

42. In the above operational amplifier circuit,

the output voltage Vout is A. -5 V B. -2.5 V C. 0 V D. +2.5 V E. +5 V 43. 88

226 Ra is one of the nuclides in the uranium decay series. If the stable end-product of this series is 82

206Pb , the number of β-particles emitted between the 88

226 Ra stage and the end of the series is

A. 4 B. 6 C. 10 D. 14 E. 20

44. Which of the following statements about a typical nuclear reactor in a power plant is/are correct?

(1) The percentage of 235U isotope in

fuel rods is higher than that in natural uranium.

(2) Boron-coated steel rods are used to control the rate of neutron production.

(3) Pressurized water is used as coolant inside the reactor core.

A. (1) only B. (3) only C. (1) and (2) only D. (2) and (3) only E. (1), (2) and (3) 45. In an experiment to determine the period

of oscillation, T, of a simple pendulum, the time, t, for a number of complete oscillations is taken. It is found that the time for 30 complete oscillations is 28.7 ± 0.3 s. Which of the following statements is/are correct?

(1) The reading error in t can be reduced

by counting 50 oscillations. (2) The percentage error in T is the same

as that in t. (3) The period T determined should be

quoted as 0.96 ± 0.01 s. A. (1) only B. (3) only C. (1) and (2) only D. (2) and (3) only E. (1), (2) and (3)

-15 V

+15 V

1 kΩ

1 kΩ

1 kΩ

10 kΩ 20 kΩ

20 kΩ Vout

Page 193: AL Physics 1981-2004 MC

97 AL Physics/M.C./P.10

Question No. Key 1. C 2. C 3. D 4. C 5. E 6. D 7. E 8. E 9. C 10. A 11. E 12. D 13. B 14. C 15. B 16. B 17. D 18. D 19. A 20. A 21. A 22. B 23. D 24. D 25. E

Question No. Key 26. D 27. C 28. B 29. B 30. E 31. A 32. C 33. B 34. B 35. B 36. C 37. C 38. D 39. A 40. B 41. D 42. A 43. A 44. E 45. D

Page 194: AL Physics 1981-2004 MC

98 AL Physics/M.C./P.1

1998 Hong Kong Advanced Level Examination AL Physics

Multiple Choice Question

1.

1.0 kg

0.5 kg Two identical light springs are connected

with two masses of 1.0 kg and 0.5 kg as shown. Which of the following statements is/are correct?

(1) The tension in the upper spring is

double that in the lower spring. (2) The force acting on the ceiling by the

whole system is 15 N. (3) The tension in the upper spring will

remain unchanged if the two masses are exchanged in position.

A. (1) only B. (3) only C. (1) and (2) only D. (2) and (3) only E. (1), (2) and (3) 2. The velocities of the longitudinal and

transverse waves produced by an earthquake are 5.0 kms-1 and 9.0 kms-1 respectively. A seismograph records the arrival of the transverse waves 64 s before that of the longitudinal waves. How far away was the earthquake?

A. 256 km B. 320 km C. 576 km D. 659 km E. 720 km

3. Two blocks A and B of mass ratio 1:2 are

placed on a horizontal frictionless surface as shown above. P and Q are horizontal forces acting on A and B respectively (with P > Q) so that the blocks move to the right with constant acceleration. Find the force acting on B by A.

A. P Q−3

B. P Q+3

C. 23

( )P Q−

D. 23

P Q+

E. P Q+ 23

4. The initial momentum of an object is 2

Ns in the direction of S 45° E and its final momentum is 1 Ns due east. If this change takes place in 0.5 s, the average force acting on the object during the change is

A. 1 N due north B. 1 N due south C. 0.5 N due north D. 2 N due south E. 2 N due north

P Q A B

final momentum

initial momentum

45°

North

Page 195: AL Physics 1981-2004 MC

98 AL Physics/M.C./P.2

5.

F/N

4

05 t/s

The graph shows the time variation of the

net force, F, acting on an object of mass 1 kg. The object is initially at rest and its subsequent motion is along a straight line. Which of the following statements is/are correct?

(1) The object is accelerating in the first

5 seconds. (2) The maximum speed of the object is

less than 10 ms-1. (3) The final speed of the object is zero. A. (1) only B. (3) only C. (1) and (2) only D. (2) and (3) only E. (1), (2) and (3) 6. A small object is thrown horizontally

towards a vertical wall 1.2 m away. It hits the wall 0.8 m below its initial horizontal level. At what speed does the object hit the wall? (Neglect air resistance.)

wall

0.8 m

1.2 m

A. 2 ms-1 B. 3 ms-1 C. 4 ms-1 D. 5 ms-1 E. 7 ms-1 7. Two small identical coins A and B are

placed on a horizontal turntable which is kept rotating at a constant angular speed about a vertical axis through its centre. The distance of B from the axis of rotation is twice that of A from the axis.

Which of the following statements is/are correct?

(1) The kinetic energy of B is four times

that of A. (2) The frictional force acting on B is

double that acting on A. (3) If the angular speed of the turntable

gradually increases, B will slip before A.

A. (1) only B. (3) only C. (1) and (2) only D. (2) and (3) only E. (1), (2) and (3) 8. A body hanging on a light spring

oscillates vertically between levels X and Z as shown below. Its static equilibrium position is at level Y.

X

Y

Z

Which of the following statements is/are correct?

(1) The accelerating of the body is zero

when it is at level X. (2) The strain energy of the spring is

zero when the body is at level Y. (3) The net downward force acting on the

body is at its maximum when it is at level Z.

A. (1) only B. (3) only C. (1) and (2) only D. (2) and (3) only E. (1), (2) and (3) 9. A mass is suspended by a light spring

fixed to the ceiling. When the mass-spring system undergoes vertical oscillation freely, its frequency is f0. If the system is removed from the ceiling

Page 196: AL Physics 1981-2004 MC

98 AL Physics/M.C./P.3

and forced to oscillate vertically by a periodic driving force by variable frequencies, which of the following graphs best represents the relationship between its frequency of oscillation, f, and the applied frequency, fa?

A.

f

faf0

0

B.

f

faf0

0

C.

f

faf0

0

D.

f

faf0

0

E.

f

faf0

0

10. Which of the following travel(s) with the

speed of light in vacuum? (1) cathode rays (2) γ-rays (3) ultra violet radiation A. (1) only B. (3) only C. (1) and (2) only D. (2) and (3) only E. (1), (2) and (3) 11. The displacement-time graphs of two

oscillating particles P and Q are shown below.

Displacement

Time0

P

Q

Which of the following phase relations between P and Q is correct?

A. P leads Q by π4

.

B. P leads Q by π2

.

C. P leads Q by π.

D. Q leads P by π4

.

E. Q leads P by π2

.

Page 197: AL Physics 1981-2004 MC

98 AL Physics/M.C./P.4

12. A beam of red (R) light and violet (V) light strikes normally on a diffraction grating. Several orders of diffraction of the light beam are produced on each side of the central axis. The wavelength of the red light is 700 nm and that of the violet light is 400 nm. How will the second- and third- order diffracted beams of both colours appear? (The central axis is marked with a dot and the diffracted beams of other orders are not shown.)

Central axis A. B. C. D. E. 13.

A

B

C

D

The above figure shows a transverse wave

propagating along a string. At the instant shown, the particle D on the string is moving downward. Which of the following deductions is/are correct?

(1) The wave is propagating to the left. (2) Particle B takes longer than particle

A to return for the first time to the respective equilibrium positions along the dotted line.

(3) Particles C and D are moving in opposite directions at the instant shown.

A. (1) only B. (3) only C. (1) and (2) only D. (2) and (3) only E. (1), (2) and (3) 14. A wave is incident on a boundary between

two media X and Y at an angle of incidence i. AB represents the incident wavefront and CD the refracted wavefront. Which of the following statements is/are correct?

(1) The frequency of the wave changes

when passing from X to Y. (2) The end B of the wavefront travels to

D at the same time as the end A travels to C.

(3) When angle i increases, it is possible for the wavefront to be totally reflected at the boundary.

A. (1) only B. (3) only C. (1) and (2) only D. (2) and (3) only E. (1), (2) and (3) 15. Sunlight is scattered by air molecules in

the atmosphere. Which of the following statements concerning this process is/are correct?

(1) Light is absorbed by air molecules

and is then re-emitted. (2) The scattered light observed on the

earth’s surface is partially plane-polarized.

(3) Red light is scattered more than blue light is.

V R V R

V V R R

V R R V

R R V V

R V R V

medium X medium Y A

B D

C

i

Page 198: AL Physics 1981-2004 MC

98 AL Physics/M.C./P.5

A. (1) only B. (3) only C. (1) and (2) only D. (2) and (3) only E. (1), (2) and (3) 16.

water

tap

The above figure shows a long vertical

glass tube containing water. The water level is lowered until the first resonance occurs when a vibrating tuning fork is placed at the opening of the tube. Which of the following statements is/are correct? (Neglect end corrections.)

(1) The sound produced by the air

column above the water has the same frequency as the tuning fork.

(2) Resonance will occur again if the length of the air column above the water is increased to three times its original value.

(3) Resonance will occur again if another tuning fork of twice the frequency is used.

A. (1) only B. (3) only C. (1) and (2) only D. (2) and (3) only E. (1), (2) and (3) 17. The mass ratio of the earth and the moon

is 81:1 and the earth-moon separation is 3.8 × 108 m. At which position between the earth and the moon is the gravitational potential at a maximum? (Ignore all other planets.)

A. On the surface of the earth B. 3.4 × 108 m from the earth C. 1.9 × 108 m from the earth D. 0.4 × 108 m from the earth E. On the surface of the moon 18. In the above circuit, light bulbs L1, L2 and

L3 consume power at the rate 1:1:3. If the resistance of L3 is R, the resistance of L1 is

A. 3R/16 B. R/4 C. 3R/4 D. 4R/9 E. 3R 19.

current/A

voltage/V0 1.0 2.0

1.0

5.0

The variation of current with the voltage

applied across a device is as shown. What is the change in resistance of the device when the voltage increases from 1.0 V to 2.0 V?

A. It decreases by 0.25 Ω B. It increases by 0.25 Ω C. It decreases by 1.50 Ω D. It increases by 0.60 Ω E. It decreases by 0.60 Ω

L1

L2

L3

Page 199: AL Physics 1981-2004 MC

98 AL Physics/M.C./P.6

20. A coil of inductance 0.8 H is connected

in series with a switch S and a cell of e.m.f. 2.0 V. The total resistance of the circuit is 4.0 Ω. What is the initial rate of growth of the current when the switch is closed?

A. 0 As -1 B. 0.4 As -1 C. 0.5 As -1 D. 1.6 As -1 E. 2.5 As -1 21. In the above circuit each cell has

negligible internal resistance. The voltmeter has a resistance of 5 kΩ. The reading on the voltmeter is

A. 0 V B. 1.0 V C. 1.2 V D. 1.5 V E. 2.0 V 22. Which of the following graphs shows

correctly the relationship between the energy E stored in an isolated conducting sphere and the charge Q on it?

A.

0

E

Q B.

0

E

Q C.

0

E

Q D.

0

E

Q E.

0

E

Q

4.0 Ω

2.0 V

0.8 H S

10 kΩ

5 kΩ

V 2 V

6 V

voltmeter, 5 kΩ

Page 200: AL Physics 1981-2004 MC

98 AL Physics/M.C./P.7

23. A changing voltage V is applied across AB

in the circuit shown. The potential difference VAC across the capacitor is found to vary with time t as shown. Which of the following graphs shows the variation of potential difference VR across the resistor R with time t?

A. B. C. D.

E. 24. A charged capacitor C1, is connected to

an uncharged capacitor C2 of larger capacitance through a resistor R as shown above. When switch S is closed, which of the following statement is INCORRECT?

A. The total charges on the two

capacitors remain unchanged. B. The final charge on C2 is greater than

that on C1. C. The potential difference across C1

falls. D. The potential difference across the

two capacitors finally becomes equal. E. The total energy stored in the two

capacitors remains the same. 25. An alternating current flowing through a

resistor varies with time as shown above. What is the value of the direct current that gives the same heating effect as the alternating current?

A. 3.9 A B. 4.0 A C. 4.1 A D. 4.2 A E. 4.3 A

B

A

V R

C

D

VR

VAC

0 t

VR

0 t

VR

0 t

VR

0 t

VR

0 t

VR

0 t

current /A

5.0

0

-1 2 3 4 5

time/ms

C1

C2

R S

Page 201: AL Physics 1981-2004 MC

98 AL Physics/M.C./P.8

26. The figure shows a charged particle

moving in a circle with constant speed v on a plane perpendicular to a uniform magnetic field. Which of the following graphs represents the relation between the time T for the particle to complete a circle and its speed v?

A.

0

T

v B.

0

T

v C.

0

T

v D.

0

T

v

E.

0

T

v 27. Two long, straight parallel conducting

wires, each carrying a current I, are separated by a distance r as shown. What is the magnetic field at a point P at the same distance r from both wires? (µ0 = permeability of free space)

A. µπ0

2Ir

to the left

B. 32

0µπ

Ir

to the left

C. µπ

0 Ir

to the left

D. 32

0µπ

Ir

to the right

E. µπ0

2Ir

to the right

28. The root-mean-square voltages across the

resistor R, capacitor C and inductor L of an RCL series circuit are respectively 4 V, 1 V and 4 V. What is the phase difference between the applied voltage and the current flowing in the circuit?

C R L

r r

r

P

current I into paper

Page 202: AL Physics 1981-2004 MC

98 AL Physics/M.C./P.9

A. 30° B. 37° C. 45° D. 53° E. 60° 29. A capacitor has a capacitance of 1 farad.

Which of the following deductions must be correct?

(1) It stores 1 coulomb of charge at a

potential difference of 1 volt. (2) It gains 1 joule of electrical energy

when it has 1 coulomb of charge. (3) It will be fully charged in 1 second by

a constant current of 1 ampere. A. (1) only B. (3) only C. (1) and (2) only D. (2) and (3) only E. (1), (2) and (3) 30. The above diagram shows a rectangular

current-carrying coil ABCD in a uniform magnetic field between two pole pieces. The magnetic field is perpendicular to the plane of the coil. Which of the following statements is/are correct?

(1) There is a magnetic force acting on

the side BC of the coil. (2) The magnetic forces acting on the

coil tend to reduce its area. (3) The coil will return quickly to its

original vertical position when it is disturbed slightly.

A. (1) only B. (3) only C. (1) and (2) only D. (2) and (3) only E. (1), (2) and (3)

31. The above diagram shows two α-particles

moving directly towards each other with the same speed. Which of the following statement is/are correct?

(1) The two particles attract one another. (2) The electric potential energy of the

two particles is decreasing at the instant shown.

(3) The total momentum of the two particles remains constant in the subsequent motion.

A. (1) only B. (3) only C. (1) and (2) only D. (2) and (3) only E. (1), (2) and (3) 32. Which of the following is/are random

process(es)? (1) The background radiation in

radioactivity. (2) The Brownian motion of smoke

particles in a smoke cell. (3) The scattering of α-particles by the

nuclei in a piece of gold foil. A. (1) only B. (3) only C. (1) and (2) only D. (2) and (3) only E. (1), (2) and (3) 33. The internal energy of an ideal gas always

changes when (1) its volume increases. (2) it undergoes an isothermal change. (3) it is compressed in an insulating

container. A. (1) only B. (3) only C. (1) and (2) only D. (2) and (3) only E. (1), (2) and (3)

α-particle α-particle

N S

A

B

C

D

Page 203: AL Physics 1981-2004 MC

98 AL Physics/M.C./P.10

34. In which of the following cases will heat always flow away from a body X?

(1) X is changed from its liquid state to a

solid at freezing point. (2) X is in thermal contact with another

system at a lower temperature. (3) X is in thermal contact with another

system which has a smaller amount of internal energy.

A. (1) only B. (3) only C. (1) and (2) only D. (2) and (3) only E. (1), (2) and (3) 35. The root-mean-square speed of a sample

of helium gas molecules, each of mass m, is c. Which of the following deductions is correct?

A. The percentage of molecules

travelling at speed c is greater than at other speeds.

B. Half of the molecules travel at a speed higher than c.

C. All molecules travel randomly with speed c.

D. The average speed of the molecules is c.

E. The average kinetic energy of the molecules is ½mc2.

36. A uniform vertical steel wire is stretched

by hanging a weight from its lower end. Which of the following would put a greater strain on the wire? (The Young modulus of steel is greater than that of brass.)

(1) Using a steel wire of greater

unstretched length. (2) Using a steel wire of greater cross-

sectional area. (3) Replacing the steel wire with a brass

wire of the same unstretched length and cross-sectional area.

A. (1) only B. (3) only C. (1) and (2) only D. (2) and (3) only E. (1), (2) and (3)

37.

0 P

Q

Force

Separation

The figure shows the force-separation

graph for a pair of atoms in a material. Which of the following features is most closely related to the stiffness and the tensile strength of the material?

(1) the value of the separation at P (2) the value of the separation at Q (3) the value of the slope of the graph at

P (4) the value of the force at Q Stiffness Tensile strength A. (1) (2) B. (1) (4) C. (3) (2) D. (3) (4) E. (4) (3) 38.

filament

anode

electron gun

screenV

P

Q

d

L

deflection

In a cathode ray tube, the beam of

electrons from the electron gun is deflected vertically when a p.d. is applied between the deflection plates, P and Q, of length L and separation d. Which of the following changes will increase the deflection of the beam for a given p.d. across the deflection plates?

(1) Reducing the p.d. V between the

filament and the anode. (2) Reducing the separation d between

the deflection plates.

Page 204: AL Physics 1981-2004 MC

98 AL Physics/M.C./P.11

(3) Reducing the length L of the deflection plates.

A. (1) only B. (3) only C. (1) and (2) only D. (2) and (3) only E. (1), (2) and (3) 39. In the above circuit, the resistors are

identical, the battery has negligible internal resistance and the voltmeter draws negligible current. What are the possible voltmeter readings before and after closing switch S?

S open S closed A. 2 V 4 V B. 2 V 2 V C. 0 V 4 V D. 4 V 0 V E. 4 V 2 V 40. In the above circuit, the output voltage

from the operational amplifier is positive. Which of the following changes could cause the output voltage to change to negative?

(1) Increasing the value of R1. (2) Decreasing the value of R3. (3) Decreasing the light intensity on the

LDR.

A. (1) only B. (3) only C. (1) and (2) only D. (2) and (3) only E. (1), (2) and (3) 41. The activity of a sample of radioactive

isotopes decreases to 1/3 of its initial value in 12 s. How much more time would be required for the activity to decrease to 1/9 of its initial value?

A. 4 s B. 8 s C. 12 s D. 16 s E. 24 s 42. The diagram shows some energy levels

(drawn to scale) of a certain atom. Transition X results in the emission of a photon of wavelength 600 nm. Which transition (P to T) would result in the emission of a photon of wavelength 300 nm?

A. P B. Q C. R D. S E. T

Energy

X

T S

R Q

P

_

+

Vout

0 V

R1

R2 R3

LDR

V-

V+

V S

Page 205: AL Physics 1981-2004 MC

98 AL Physics/M.C./P.12

43. A radioactive sample, initially consists of only nuclide X, decays by the emission of an alpha particle to form a stable daughter nuclide Y. Which of the following quantities will decrease with time?

(1) The rate of decay of nuclide X. (2) The rate of growth of nuclide Y. (3) The rate of emission of alpha

particles. A. (1) only B. (3) only C. (1) and (2) only D. (2) and (3) only E. (1), (2) and (3) 44. When several neutrons and protons come

together to form a stable nucleus, which of the following statements is/are correct?

(1) Energy is released. (2) The mass of the nucleus is smaller

than the sum of the mass of the individual nucleons.

(3) In the nucleus, the electrostatic repulsion between two protons is overcome by the nuclear force between them.

A. (1) only B. (3) only C. (1) and (2) only D. (2) and (3) only E. (1), (2) and (3)

45. In a nuclear reactor, the 92235U nucleus is

fissioned by a neutron into two nuclei and a few neutrons are produced. Which of the following statements about this process is/are correct?

(1) High energy neutrons are more likely

to produce nuclear fission than low energy neutrons.

(2) Graphite can be used to absorb excess neutrons.

(3) The two nuclei produced have smaller neutron-to-proton ratios than that of the uranium nucleus.

A. (1) only B. (3) only C. (1) and (2) only D. (2) and (3) only E. (1), (2) and (3)

Page 206: AL Physics 1981-2004 MC

98 AL Physics/M.C./P.13

Question No. Key 1. D 2. E 3. D 4. E 5. C 6. D 7. E 8. B 9. D 10. D 11. A 12. B 13. A 14. D 15. C 16. C 17. B 18. A 19. E 20. E 21. A 22. A 23. A 24. E 25. C

Question No. Key 26. B 27. D 28. B 29. A 30. C 31. B 32. C 33. B 34. C 35. E 36. B 37. D 38. C 39. A 40. D 41. C 42. B 43. E 44. E 45. B

Page 207: AL Physics 1981-2004 MC

99 AL Physics/M.C./P.1

1999 Hong Kong Advanced Level Examination AL Physics

Multiple Choice Question

1.

A B F

Two objects A and B of equal mass m are

connected by two identical light springs and are placed on a horizontal smooth surface. A horizontal force F is applied to B so that the system is in equilibrium. If the applied force F is suddenly removed, what are the magnitudes of the acceleration of each object at the instant when force F is removed?

Acceleration of A Acceleration of B

A. zero mF

B. zero zero

C. mF2

mF

D. mF

zero

E. mF

mF

2. A block of mass 5 kg is placed on the

inclined surface of the wedge shown above. All contact surfaces are assumed to be smooth. What is the magnitude of the horizontal force F exerted on the wedge so that it remains stationary while the block is sliding down the inclined surface with acceleration?

A. 0 N B. 18 N C. 24 N D. 30 N E. 32 N

3. The above figure shows an open wagon

moving with negligible resistance in vertically falling rain. An appreciable amount of rain falls into the wagon and accumulates there. What are the effects of the accumulating rain on the speed, momentum and kinetic energy of the wagon? (Ignore the effects of the raindrops hitting the front of the wagon.)

speed momentum kinetic energy A. decreased unchanged decreased B. decreased unchanged unchanged C. decreased decreased decreased D. unchanged unchanged unchanged E. unchanged increased increased 4. The velocity of an object changes with

time but the magnitude of its acceleration remains unchanged. The motion of the object can be

(1) simple harmonic motion. (2) parabolic motion of a projectile. (3) uniform circular motion. A. (1) only B. (3) only C. (1) and (2) only D. (2) and (3) only E. (1), (2) and (3)

horizontal surface

F

40 cm

30 cm

Page 208: AL Physics 1981-2004 MC

99 AL Physics/M.C./P.2

5. The above graph shows the variation of

kinetic energy E with the square of velocity v of a moving mass m. What is the momentum of the mass when it is moving at a speed of 2 ms-1?

A. 1 Ns B. 2 Ns C. 4 Ns D. 8 Ns E. 16 Ns 6.

2m

m

P

A light, rigid rod of length 1 m is hinged smoothly at P at one end. Two small masses m and 2m are fixed to the mid-point and the other end of the rod respectively. The rod is held vertically above P and is released from rest. What is the total rotational energy of the system when the rod rotates to the position vertically below P? (Neglect air resistance)

A. 2.5 mg B. 4.5 mg C. 5 mg D. 7.5 mg E. 9 mg

7. A turntable of moment of inertia 1.0 × 10-

3 kg m2 is under the action of a torque. The variation of the torque τ acting about the axis of rotation with time t is as shown. If the turntable is at rest initially, what is its angular momentum at t = 20 s?

A. 40 kg m2 s-1 B. 100 kg m2 s-1 C. 200 kg m2 s-1 D. 1 × 105 kg m2 s-1 E. 2 × 105 kg m2 s-1 8. A small ball-bearing is projected with

velocity v from the lowest position P of a vertical circular track which is not smooth. The ball-bearing starts to leave the track at Q. Which of the following diagrams best represents all the forces acting on the ball-bearing at Q?

A . B. C.

E/J

v2/m2s-2 1

1

0

Q

P

Q

P

Q

P

t/s 14 20

τ/Nm

10

0

Q

v P

Page 209: AL Physics 1981-2004 MC

99 AL Physics/M.C./P.3

D. E. 9. The above graph shows the variation of

the acceleration a of a particle with its displacement x from a fixed point. Which of the following graphs shows the variation of its potential energy U with x?

A. B.

C. D. E. 10.

A converging light beam is directed

towards a convex mirror M. The reflected beam then passes through a convex lens L and forms an image at I. Which of the following about the focal lengths of M and L is correct?

focal length of M focal length of L A. more than 4 cm 2 cm B. less than 4 cm 2 cm C. 4 cm 2 cm D. less than 4 cm 4 cm E. more than 4 cm 4 cm

Q

P

Q

P

a

0 x

U

0 x

U

0 x

U

0 x

U

0 x

U

0 x

Page 210: AL Physics 1981-2004 MC

99 AL Physics/M.C./P.4

11. An astronomical refracting telescope is adjusted to view a distant object under normal adjustment. Which of the following statement is/are correct?

(1) The first image formed by the

objective is real and is smaller than the object.

(2) The final image formed by the eyepiece is at the least distance of distinct vision from the eye.

(3) If part of the objective is blocked by an opaque obstacle, part of the final image could not be viewed.

A. (1) only B. (3) only C. (1) and (2) only D. (2) and (3) only E. (1), (2) and (3) 12. In a Young’s double slit experiment, a

monochromatic light source of wavelength 700 nm is used and the separation of the slits is 0.1 mm. If 15 bright fringes are observed, what is the angle subtended by those fringes at the centre of the double slit?

A. 4.8° B. 5.2° C. 5.6° D. 6.0° E. 6.4°

13. Two dippers S1 and S2 1.2 cm apart are attached to a vibrator. They vibrate in phase in a ripple tank and generate waves of wavelength 0.8 cm.

Which of the following statements is/are

correct? (1) S1 and S2 are positions of antinodes. (2) There is only one point between S1

and S2 which is always stationary. (3) The separation between two nearest

antinodes between S1 and S2 is 0.4 cm.

A. (1) only B. (3) only C. (1) and (2) only D. (2) and (3) only E. (1), (2) and (3) 14. On a straight road, a car and an ambulance

move with uniform speeds u1 and u2 respectively (with u2 > u1) in the same direction. Initially the ambulance is behind the car and it overtakes the car after some time. The siren on the ambulance continuously emits a sound of a certain frequency. What is the ratio of the wavelength of sound received by an observer in the car before the ambulance overtakes to that after the ambulance overtakes?

(Given: velocity of sound in air = v)

A. v uv u

−+

2

2

B. v uv u

+−

2

2

C. u uu u

2 1

2 1

−+

D. v u uv u u

− ++ −

2 1

2 1

E. v u uv u u

+ −− +

2 1

2 1

1.2 cm

S1 S2

15 bright fringes

angle subtended at the centre of the double slit

Page 211: AL Physics 1981-2004 MC

99 AL Physics/M.C./P.5

15. Students A and B are standing 5 m apart. A shouts to B and the intensity level in db recorded by B is x. A then walks away from B so that they are 10 m apart. If A shouts again with the same power, what is the intensity level in db recorded by B?

A. 14

x

B. 12

x

C. log10 4x

D. x - 3 E. x - 6 16. The above circuit can be used to find the

resistance of the resistor R. Which of the following statements is/are correct?

(1) The current passing through the

ammeter is in fact larger than that passing through R.

(2) The ratio of voltmeter reading to ammeter reading is in fact smaller than the resistance of R.

(3) The circuit is suitable for measuring high resistance.

A. (1) only B. (3) only C. (1) and (2) only D. (2) and (3) only E. (1), (2) and (3) 17. Three resistors of resistance R1, R2 and R3

are connected in parallel. It is known that R1 > R2 > R3. The equivalent resistance of this combination is R. Which of the following statements is/are correct?

(1) Energy dissipated in moving 1 C of

charge through the resistor of resistance R1 is greater than that through R3.

(2) R is smaller than R2.

(3) If the resistor with resistance R1 is removed, the resulting equivalent resistance is increased.

A. (1) only B. (3) only C. (1) and (2) only D. (2) and (3) only E. (1), (2) and (3) 18. Which of the following statements about

a communication satellite in parking orbit above the earth’s surface is incorrect?

A. It is accelerating towards the centre

of the earth at all times. B. It must be in a circular orbit above

the earth’s equator. C. It is always vertically above the same

place on the earth’s surface. D. It must be rotating in the same sense

and with the same angular speed as the earth.

E. It is at a height where its gravitational potential energy is numerically equal to its kinetic energy.

19. In the above circuit, the battery has

constant e.m.f. and negligible internal resistance. If switch S is closed, what would happen to the electric potential at P and at Q?

Potential at P Potential at Q A. increased increased B. increased decreased C. unchanged unchanged D. decreased increased E. decreased decreased

V

R

A

R1

R2 R3

S

P Q

Page 212: AL Physics 1981-2004 MC

99 AL Physics/M.C./P.6

20. Each of the following three students states a conclusion after learning the formulae P = IV, P = V2/R and P = I2R in electricity. Which of these conclusion is/are correct?

(1) The power dissipated by any electric

appliance can be calculated from P = IV.

(2) The power dissipated by a kettle is inversely proportional to its resistance as P = V2/R.

(3) The power consumed by a running motor cannot be calculated from P = I2R.

A. (1) only B. (3) only C. (1) and (2) only D. (2) and (3) only E. (1), (2) and (3) 21. Which of the following statements about

electric field lines is incorrect? A. They are closest where the field is

strongest. B. They are always perpendicular to

equipotential lines. C. They always point from high electric

potential to low electrical potential. D. Work has to be done in moving an

electron along the direction of a field line.

E. They tend to attract one another. 22. A charged capacitor of capacitance 47 µF

is connected to a voltmeter. The voltmeter reading drops from 8 V to 2 V in 7.8 s. Find the resistance of the voltmeter.

A. 60 kΩ B. 80 kΩ C. 120 kΩ D. 166 kΩ E. 240 kΩ 23. A resistance wire is connected across the

terminals of a battery. Which of the following statements is incorrect?

A. Before connection to the battery, the conduction electrons in the wire move randomly.

B. After connection to the battery, an electric field is set up along the length of the wire.

C. The conduction electrons in the wire are accelerated momentarily in the opposite direction to that of the electric field.

D. The conduction electrons collide with one another, giving out heat energy.

E. The current in the wire is proportional to the average drift velocity of the electrons.

24. Three capacitors are connected to a 45 V

supply as shown in the above circuit. The voltage at the output is

A. 5 V B. 15 V C. 30 V D. 36 V E. 40 V 25. A capacitor is charged by a constant

current. Which of the following graphs shows the time variation of the energy stored in the capacitor during the charging process?

A.

energy

time

45 V

0 V

2 µF

8 µF

2 µF Output

0 V

Page 213: AL Physics 1981-2004 MC

99 AL Physics/M.C./P.7

B.

energy

time C.

energy

time D.

energy

time E.

energy

time 26. In the above circuit, a capacitor C is

charged from a battery E and then completely discharged through a resistor R using a vibrating reed switch S. The

waveform displayed on the screen of the CRO with suitable time base should be

A.

B.

C.

D.

E.

27. Which of the following graphs correctly

shows the variation of the magnetic flux density B along the axis of a long solenoid carrying a steady current with its length l?

A.

B

0l

B.

B

0l

R

C To CRO

S

E

Page 214: AL Physics 1981-2004 MC

99 AL Physics/M.C./P.8

C.

B

0l

D.

B

0l

E.

B

0l

28. Which of the following products is/are

measured in joules? (1) Boltzmann constant k times absolute

temperature T (2) Planck constant h times frequency ν (3) Electronic charge e times potential

difference V A. (1) only B. (3) only C. (1) and (2) only D. (2) and (3) only E. (1), (2) and (3) 29. In the above circuit, a coil L of large

inductance, a 2 V light bulb B, a switch S and a 2 V battery are connected in series. A neon lamp is connected across L. Which of the following statements is/are correct upon the opening of switch S?

(1) A large e.m.f. is induced across coil L.

(2) The neon lamp lights up momentarily. (3) Light bulb B continues to light up

until the neon lamp dies out. A. (1) only B. (3) only C. (1) and (2) only D. (2) and (3) only E. (1), (2) and (3) 30. The figure shows a long straight wire

carrying an a.c. current which lies on the plane of the paper. When a small search coil is placed at B such that the plane of the coil is on the paper, the length of the trace on the CRO is 2 cm (with the time base of the CRO switched off). If the search coil is moved to A, the length of the trace would be

A. 0.5 cm. B. 1 cm. C. 2 cm. D. 4 cm. E. 8 cm. 31. For which of the following does the force

between two objects vary inversely as the square of the distance between their centres?

(1) Two equal masses joined by a rubber

band under tension. (2) Two long straight parallel conducting

wires carrying steady electric currents.

(3) The earth and a satellite in its orbit. A. (1) only B. (3) only C. (1) and (2) only D. (2) and (3) only E. (1), (2) and (3)

2 V S

B

neon lamp

L

6 cm 6 cm search coil

To CRO A B

long straight

wire carrying an a.c. current

Page 215: AL Physics 1981-2004 MC

99 AL Physics/M.C./P.9

32. The above figure shows an a.c. circuit in

which a variable resistor R is connected in series with an air-cored inductor L of negligible resistance. The a.c. supply has constant r.m.s. voltage and its frequency can be varied. Which of the following methods, on its own, can increase the r.m.s. voltage across the variable resistor R?

(1) Inserting an iron core in the inductor

L. (2) Increasing the frequency of the a.c.

supply. (3) Increasing the resistance of R. A. (1) only B. (3) only C. (1) and (2) only D. (2) and (3) only E. (1), (2) and (3) 33. The following are energy values for

hydrogen.

X mean kinetic energy of hydrogen gas molecules at room temperature

Y ionization energy of a hydrogen atom

Z photon energy of a particular line in the visible emission spectrum of hydrogen

Which of the following shows the correct

ascending order (the least first) of these energy values?

A. X < Z < Y B. X < Y < Z C. Y < Z < X D. Z < Y < X E. Z < X < Y 34. When an electron in an atom falls from an

excited state to the ground state, which of the following forms of radiation is not emitted?

A. infra-red B. gamma-ray C. ultra-violet D. visible light E. X-rays 35. Two long, straight, parallel conducting

wires P and Q are positioned as shown. The same current flows through both wires and is directed into the plane of the paper. Points A, B and C on the plane of the paper are equidistant from both wires where C is the mid-point between the wires. Which of the following statements is/are correct?

(1) The magnetic field strength at C is

greater than that at A. (2) The directions of the magnetic field

at A and at B are the same. (3) The magnetic field strength at B will

increase if the current flowing in the wires increases.

A. (1) only B. (3) only C. (1) and (2) only D. (2) and (3) only E. (1), (2) and (3)

L R

P Q

A

B

C

Page 216: AL Physics 1981-2004 MC

99 AL Physics/M.C./P.10

36. A coil of metal wire is placed on a plane perpendicular to a uniform magnetic field. The coil is rotated through 180º about a diameter as shown.

The total charge circulated in the coil due

to the induced current is independent of A. the area of the coil. B. the flux density of the magnetic field. C. the number of turns of the coil. D. the resistance of the coil. E. the time taken for the flux change. 37. Two metallic containers X and Y of

volume V and 4V respectively are connected by a narrow tube as shown. Initially the tap S is closed and an ideal gas is contained in X at a pressure of 400 kPa while container Y is evacuated. The tap S is then opened and when equilibrium is finally reached

A. the gas pressure in X is 100 kPa. B. there are still gas molecules moving

through the tap S. C. the product of pressure and volume

of the gas in X is equal to that in Y. D. the density of gas molecules in X is

greater than that in Y. E. the gas molecules in Y on average

move faster than those in X. 38. Which of the following descriptions

about ideal gas is/are correct?

(1) An ideal gas obeys Boyle’s law only under high temperature and low pressure.

(2) The molecules of an ideal gas have no size.

(3) The internal energy of an ideal gas consists of kinetic energy only.

A. (1) only B. (3) only C. (1) and (2) only D. (2) and (3) only E. (1), (2) and (3) 39. A fixed mass of an ideal gas undergoes

the process ABC as shown in the pressure-temperature diagram. Which of the following deductions from the diagram is incorrect?

A. The volume of the gas at C is greater

than that at B. B. No work is done by the gas when it

changes from A to B. C. Work is done by the gas on the

surroundings when it changes from B to C.

D. Heat energy is transferred to the gas when it changes from A to B.

E. Heat energy is extracted from the gas when it changes from B to C.

40. The above graph shows the variation in

maximum kinetic energy K of photoelectrons with the frequency f of the incident radiation on a metallic surface. If radiation of twice the intensity is used, which of the following graphs (dotted line) shows the variation of K with f?

K

f 0

180°

V 4V

X Y

S

p/kPa

T/K 0

A

B

C

Page 217: AL Physics 1981-2004 MC

99 AL Physics/M.C./P.11

A. B. C. D. E. 41. Three sinusoidal signals V1, V2 and V3 are

applied in pairs to the X- and Y- inputs of a CRO and the resulting Lissajous figures are as shown.

X-input Y-input Lissajous figures

V1

V2

V2

V3

If V1 and V3 are applied to the X- and Y-

inputs respectively, the Lissajous figure should look like

A.

B.

C.

D.

E.

K

f 0

K

f 0

K

f 0

K

f 0

K

f 0

Page 218: AL Physics 1981-2004 MC

99 AL Physics/M.C./P.12

42.

0 100

100

N

Z

For stable nuclides, the number of

neutrons N when plotted against the number of protons Z would give points lying within the above shaded region. Which of the following statements is/are correct?

(1) The heavy stable nuclides have more

neutrons than protons. (2) Unstable nuclides decay to produce

new nuclides closer to the shaded region.

(3) Unstable nuclides above the shaded region usually decay through α-emission.

A. (1) only B. (3) only C. (1) and (2) only D. (2) and (3) only E. (1), (2) and (3) 43. Carbon-14 is radioactive and undergoes

β-decay with a half-life of about 5 600 years. A GM-tube together with a scaler are used in an experiment to determine the concentration of carbon-14 in a piece of excavated wood. To find an accurate value for the activity of the excavated wood, the experimenter should

(1) prepare a sample of pure carbon-14

from the piece of excavated wood. (2) take a count over as long a period of

times as possible. (3) take a second count without the

presence of the excavated wood. A. (1) only B. (3) only C. (1) and (2) only D. (2) and (3) only E. (1), (2) and (3)

44. Which of the following are reasonable estimates of the density and volume of an adult human?

Density/kgm-3 Volume/m3 A. 100 1 B. 100 0.1 C. 1 000 0.1 D. 1 000 0.01 E. 10 000 0.01 45. Which of the following will give rise to a

systematic error in experimental measurement?

(1) using a slow-running stop watch in

keeping time (2) ignoring the background counts in

radioactive decay measurements (3) uncertainty in locating the sharpest

image position in an optics experiment

A. (1) only B. (3) only C. (1) and (2) only D. (2) and (3) only E. (1), (2) and (3)

Page 219: AL Physics 1981-2004 MC

99 AL Physics/M.C./P.13

Question No. Key 1. A 2. C 3. A 4. D 5. C 6. C 7. B 8. A 9. E 10. A 11. A 12. C 13. B 14. A 15. E 16. C 17. D 18. E 19. E 20. E 21. E 22. C 23. D 24. B 25. B

Question No. Key 26. A 27. B 28. E 29. C 30. D 31. B 32. B 33. A 34. B 35. D 36. This item was deleted 37. B 38. D 39. E 40. A 41. D 42. C 43. D 44. C 45. C

Page 220: AL Physics 1981-2004 MC

00 AL Physics/M.C./P.1

2000 Hong Kong Advanced Level Examination AL Physics

Multiple Choice Question

1. Two books A and B are placed on a

horizontal table surface as shown. A horizontal force F is applied to A but the system remains stationary. Which of the following statements is/are correct?

(1) The frictional force acting on B by

the table surface is greater than F. (2) The frictional force acting on A by B

is towards the left. (3) The system would remain stationary

if F is applied to B instead. A. (1) only B. (3) only C. (1) and (2) only D. (2) and (3) only E. (1), (2) and (3) 2. In which of the following situations is the

magnitude of the normal reaction of the supporting surface always equal to the weight of the body?

(1) A ball bouncing vertically on a

horizontal ground is in contact with the ground.

(2) An astronaut in a spacecraft which performs circular motion around the earth.

(3) A boy standing in a lift which is moving vertically upward with a uniform velocity.

A. (1) only B. (3) only C. (1) and (2) only D. (2) and (3) only E. (1), (2) and (3) 3. A light rigid rod PQ is hinged smoothly to

the wall at one end while the other end is

connected by an inextensible string to a point R directly above P. A weight W is suspended from a point on the rod. If the rod remains horizontal, which of the following change(s) would increase the tension in the string?

(1) Shifting the weight towards Q (2) Replacing the string with a shorter

one and connecting it to the mid-points of PQ and PR

(3) Replacing the string with a longer one and connecting it to a point higher than R

A. (1) only B. (3) only C. (1) and (2) only D. (2) and (3) only E. (1), (2) and (3) 4. A student holds one end of a string to

which a block of mass 8 kg is tied at the other end. He raises the block with an acceleration by pulling the string in an upward direction. If the maximum tension that the string can withstand is 120 N, find the maximum acceleration of the block before the string breaks.

A. 5.0 ms-2 B. 7.5 ms-2 C. 10.0 ms-2 D. 12.5 ms-2 E. 15.0 ms-2 5. A trolley of mass 0.5 kg moves with a

certain acceleration down a runway which is inclined to the horizontal at 15°. If the angle of inclination is increased to 20°, the acceleration of the trolley would be doubled. Find the average frictional force, assuming the same in both cases, acting on the trolley.

A. 0.72 N B. 0.80 N C. 0.88 N D. 0.96 N E. 1.04 N

B A

F

Q

W

R

P

Page 221: AL Physics 1981-2004 MC

00 AL Physics/M.C./P.2

6. Two objects A and B of masses m and 2m respectively are initially at rest on a smooth, horizontal surface. If each of them is acted upon by the same force for the same period of time, the ratio of the gain in kinetic energy of A to that of B is

A. 2:1 B. 1:2 C. 1:1 D. 1:4 E. 4:1 7. A light spring is fixed to the bottom of a

vertical tube. A ball is released from rest at a height h above the upper end of the spring as shown. After rebounding several times the ball eventually comes to rest and stays on top of the spring. Assume all contact surfaces are smooth and the spring obeys Hooke’s law throughout. Which of the following statements is/are correct?

(1) The compression of the spring is

proportional to the mass of the ball. (2) The compression of the spring is

independent of the height h. (3) The gravitational potential energy

lost by the ball is equal to the strain energy stored in the spring.

A. (1) only B. (3) only C. (1) and (2) only D. (2) and (3) only E. (1), (2) and (3) 8. A car of mass m is moving with speed v

on a banked road along a circular path of horizontal radius r. The angle of

inclination of the road is θ. If the centripetal force arises entirely form a component of the normal reaction N from the road, which of the following relations is correct?

A. N cos θ = mg B. N = mg cos θ

C. v2 = θsin

gr

D. v2 = θtan

gr

E. θtan

N =

rmv2

9. X and Y are two planets. Each of them has

a low-altitude satellite revolving in a circular orbit close to the planet. If the two satellites are observed to have the same period, then X and Y must have nearly the same

A. mass. B. average density. C. radius. D. acceleration due to gravity at the

planet’s surface. E. gravitational potential at the planet’s

surface. 10. A small block of mass 0.1 kg is

suspended from the ceiling by a light spring of force constant 12 Nm-1. If the block is projected vertically downwards with a speed of 0.5 ms-1 from its equilibrium position, what is the maximum acceleration of the block in its subsequent motion?

A. 5.0 ms-2 B. 5.5 ms-2 C. 6.0 ms-2 D. 6.5 ms-2 E. 7.0 ms-2 11. A student performing a centripetal force

experiment whirls a rubber bung attached to one end of a string which passes through a glass tube with smooth openings, and has weight W hanging at its

r

N

mg

θ

glass tube

rubber bung

L ω

θ

W

h

Page 222: AL Physics 1981-2004 MC

00 AL Physics/M.C./P.3

other end. The weight of the rubber bung is much smaller that W. The rubber bung is set into a horizontal uniform circular motion with angular speed ω while the length off the string beyond the upper opening of the glass tube is L and this portion of the string makes an angle θ with the vertical as shown. Which of the following statements is/are correct?

(1) If L is kept constant, θ will decrease

with ω. (2) If θ is kept constant, L will increase

with ω. (3) When W increases, θ will increase. A. (1) only B. (2) only C. (3) only D. (1) and (2) only E. (2) and (3) only 12. A student used a simple pendulum to

measure acceleration due to gravity at the earth’s surface. the experimental value was found higher than the standard value. Which of the following is a possible reason for this?

A. The effect due to air resistance is not

negligible. B. The stop watch used for the

experiment runs too slowly. C. The length of the string has been

taken as the effective length of the pendulum.

D. The experiment has been performed at a place above sea-level.

E. The experiment has been performed at a place below sea-level.

13. On a smooth horizontal surface, a block

connected to the wall with a light spring performs simple harmonic motion of amplitude A as shown. If the amplitude is reduced to A/2, which of the following quantities would be halved?

(1) the maximum velocity of the block (2) the maximum elastic potential energy

stored in the spring (3) the period of oscillation of the block A. (1) only B. (3) only

C. (1) and (2) only D. (2) and (3) only E. (1), (2) and (3) 14. With his arms and legs outstretched, a

diver leaves a high-diving board with some initial angular velocity. Before he enters the water, which of the following statements is INCORRECT? (Neglect air resistance.)

A. His angular momentum about the

centre of gravity remains constant. B. No external force acts on him. C. His loss in potential energy becomes

his kinetic energy. D. He can decrease his moment of

inertia by pulling his arms and legs to his chest.

E. He can make more turns before reaching the water surface by coiling up his body.

15. The figure shows a variation of Barton’s

pendulum. A, B and C are three paper cone pendulums of different lengths suspended from the string. The heavy bob is pulled well aside and released so that it oscillates on a plane perpendicular to the paper. The paper cone pendulums are forced into oscillations. When the motion settles down after a short time, which of the following statements is/are correct?

(1) B has the largest amplitude among the

three paper cone pendulums. (2) All the paper cone pendulums are

oscillating with the same frequency. (3) A and C are approximately in

antiphase. A. (1) only B. (3) only C. (1) and (2) only D. (2) and (3) only E. (1), (2) and (3)

C

paper cone pendulums

(loaded with metal rings)

A

B heavy bob

string

A

Page 223: AL Physics 1981-2004 MC

00 AL Physics/M.C./P.4

16. Two transverse pulses travel towards each

other along a stretched string as shown. They are identical and symmetric in shape but one is inverted. At the moment the pulses pass each other the string becomes straight. At this moment the wave energy

A. becomes wholly elastic potential in

the string. B. becomes wholly kinetic due to

transverse motion of the string. C. becomes partly elastic potential and

partly kinetic in the string. D. converts into internal energy and is

dissipated as heat energy in the string.

E. becomes momentarily zero. 17. The above figure shows an instantaneous

wave profile representing a sound wave travelling to the right in air. Which of the following about the part of the wave at P at this instant is/are correct? (Take the displacement towards right as positive.)

(1) P is a centre of compression. (2) The air particle at P has the greatest

kinetic energy. (3) The air particle at P is moving

towards right. A. (1) only B. (3) only C. (1) and (2) only D. (2) and (3) only E. (1), (2) and (3) 18.

A dot vibrator is moving across the

surface of water in a ripple tank with a steady speed as shown. The figure represents the water surface showing the pattern of the water waves against a background of centimetre squares. The speed of the water waves is 20 cms-1. What is the speed of the dot vibrator?

A. 10 cms -1

B. 8 cms-1

C. 5 cms-1

D. 4 cms-1

E. 2 cms-1 19. A glass lens is made non-reflecting for

yellow light by coating it with a thin film of transparent material of a refractive index less than that of glass. Which of the following statements is/are correct?

(1) The surface of the glass lens appears

yellow in white light. (2) The minimum thickness of the film is

¼ of the wavelength of yellow light in the film.

(3) As a result of the destructive interference between the light rays reflected from the two interfaces, energy which would have been wasted as reflected light increases the amount of transmitted light.

A. (1) only B. (3) only C. (1) and (2) only D. (2) and (3) only E. (1), (2) and (3) 20. In the above figure, XY is the principal

axis of a lens L. PQ and OS are two refracted rays from L which originate from a point object placed on the left side of L. Which of the following deductions is/are correct?

position

+ _

P

Displacement of air particles

L

P Q

O

S

Y X

glass

air

thin film

1 cm

Page 224: AL Physics 1981-2004 MC

00 AL Physics/M.C./P.5

(1) The lens L must be a concave lens. (2) The point object must lie along the

line OS. (3) The image of the point object must

be virtual. A. (1) only B. (3) only C. (1) and (2) only D. (2) and (3) only E. (1), (2) and (3) 21. In the above arrangement, two small test

charges +q and +2q are brought from infinity to the positions shown. The two charges are collinear with another charge +Q and their mutual separation is d. Which of the following statements is/are correct?

(1) Charge +q is at a higher potential

than charge +2q. (2) The work done in bringing the

charges +q and +2q from infinity to their respective positions is the same.

(3) The potential energy of the system would increase if d decreases.

A. (1) only B. (3) only C. (1) and (2) only D. (2) and (3) only E. (1), (2) and (3) 22. The figure shows a pattern of electric

field lines in which P, Q and R are points marked on one of the field lines with PQ = QR. If the potential at P is 0 V, which of the following can give the possible potentials at Q and at R?

Potential at Q Potential at R A. -200 V -450 V B. -200 V -400 V C. -200 V -350 V D. +200 V +350 V E. +200 V +450 V

23. The figure shows some of the resistors in

a network of resistors. The magnitudes and directions of some of the currents are marked as shown. Find the magnitude and direction of the current passing through the resistor R.

A. 0.2 A from right to left B. 0.2 A from left to right C. 0.4 A from right to left D. 0.4 A from left to right E. It cannot be determined as the value

of R is not given. 24. In the above circuit, the internal

resistance of the battery is 4 Ω. R is a variable resistor. Which of the following statements about the circuit is/are correct?

(1) If the resistance of R is very small,

the terminal potential difference of the battery is very small but the power dissipated in the battery is large.

(2) If the resistance of R is very large, the current in the circuit is very small but most of the power supplied by the battery is dissipated by R.

(3) If the resistance of R is 4 Ω, the power supplied to R by the battery is at a maximum.

A. (1) only B. (3) only C. (1) and (2) only D. (2) and (3) only E. (1), (2) and (3)

0.3 A

5 Ω

3 Ω

1 Ω R

0.2 A

internal resistance 4 Ω

R

P Q R

+Q

d

+q

d

+2q

Page 225: AL Physics 1981-2004 MC

00 AL Physics/M.C./P.6

25. Four parallel long straight conductors

carrying currents of equal magnitude pass vertically through the four corners of a square PQRS. The current is directed into paper in one conductor and is directed out of paper in the other three conductors. Which of the following arrangement can produce a resultant magnetic induction at the centre O of the square in the direction shown?

Current into paper Current out of paper A. P Q, R, S B. Q P, R, S C. R P, Q, S D. S P, Q, R E. It is impossible to produce a resultant

magnetic induction at O in the direction shown.

26. Two parallel plates are connected to an

E.H.T. of 4.5 kV. Electric breakdown occurs when the separation of the plates is reduced to 1.5 mm. Estimate the maximum acceleration of an electron between the plates.

(Given: charge of an electron = 1.6 × 10-19 C; mass of an electron = 9.1 × 10-31 kg) A. 4.0 × 107 ms-2 B. 1.0 × 109 ms-2 C. 1.2 × 1012 ms-2 D. 1.6 × 1015 ms-2 E. 5.3 × 1017 ms-2 27. A beam of charged particles passes

undeflected through a region of crossed uniform electric and magnetic fields. Which of the following must be common to the particles making up this beam?

A. charge to mass ratio B. velocity C. mass D. sign of charge E. magnitude of charge 28. When a simple motor rotating at a steady

speed is suddenly jammed and comes to a

stop, which of the following statements is/are correct?

(1) The rate of change of magnetic flux

through the coil of the motor will be very large.

(2) A large e.m.f. will be induced in the coil of the motor.

(3) A large current will flow through the coil of the motor.

A. (1) only B. (3) only C. (1) and (2) only D. (2) and (3) only E. (1), (2) and (3) 29. The figure shows a bridge rectifier circuit

in which all the diodes are assumed ideal. The source is a sinusoidal a.c. supply. Which of the following traces (I, II or III) would be displayed on a CRO connected across the load resistor R if

(1) the diode D were reversed in the

circuit, (2) the diode D were removed leaving a

break in the circuit? (1) (2) A. I II B. II II C. II III D. III II E. III III 30.

S R

P Q

O

I II

III

D

R

X Y

Z

6.0 V

Page 226: AL Physics 1981-2004 MC

00 AL Physics/M.C./P.7

In the above circuit, the battery has an e.m.f. of 6.0 V and all the capacitors are identical. Initially none of the capacitors are charged. The switch is first connected to X and then to Y. What is the final potential difference between Z and Y?

A. 1.0 V B. 1.5 V C. 2.0 V D. 2.4 V E. 3.6 V 31. A constant p.d. is applied to a solenoid.

The variation of current I with time t is as shown. Which of the following can be deduced from the shape of the graph?

A. The solenoid has resistance as well as

inductance. B. The inductance of the solenoid

decreases as time increases. C. The inductance of the solenoid

increases as time increases. D. The e.m.f. induced in the solenoid is

proportional to the current. E. The e.m.f. induced in the solenoid is

proportional to the rate of change of current.

32. In the above circuit, A and B are identical

light bulbs and L is a pure iron-cored inductor. Which of the following statements is/are correct?

(1) When switch S is closed, bulb A will

light up first. (2) After switch S is closed for some

time, A and B are equally bright. (3) When switch S is open, bulb A will go

out first. A. (1) only B. (3) only C. (1) and (2) only

D. (2) and (3) only E. (1), (2) and (3) 33. A capacitor C and a resistor R are

connected in series to an a.c. source of constant output voltage. Which of the following statements is/are correct?

(1) The current leads the applied voltage. (2) The impedance of the circuit

decreases when the frequency increases.

(3) The power factor of the circuit decreases when the frequency increases.

A. (1) only B. (3) only C. (1) and (2) only D. (2) and (3) only E. (1), (2) and (3) 34. The figure shows an LC oscillatory circuit

in which C is a capacitor and L is an air-cored inductor. At a certain instant the magnetic flux inside L is pointing upward as shown. Which of the following descriptions of the capacitor is/are possible at this instant?

(1) The capacitor is charging with the

upper plate positively charged. (2) The capacitor is charging with the

lower plate positively charged. (3) The capacitor is discharging with the

upper plate positively charged. (4) The capacitor is discharging with the

lower plate positively charged. A. (1) only B. (1) or (3) only C. (1) or (4) only D. (2) or (3) only E. (2) or (4) only

I

0 t

C R

C

L

B L

S

A

Page 227: AL Physics 1981-2004 MC

00 AL Physics/M.C./P.8

35. The following diagrams show the currents of sinusoidal waveform and square waveform passing in turn through a resistor. The average power dissipated is the same in both cases.

Given that the peak value of the sinusoidal

current is 1 A. What is the value of I0 of the square waveform current?

A. 2

1 A

B. 21 A

C. 1 A

D. 2 A

E. 22 A 36. The average stress in the legs of a man

standing upright is S. If the dimensions of the man are doubled while the average density of the body remains the same, the average stress in the legs would be

A. S/2. B. S. C. 2S. D. 4S. E. 8S. 37. Let r be the separation of two molecules

in a solid and U be the intermolecular potential energy of these two molecules. When r = r0, the attractive force between the two molecules is equal to their repulsive force. Let U = 0 when r = r0. Which of the following statements is INCORRECT?

A. U is non-negative for all values of r. B. When r < r0, U increases as r

decreases. C. When r > r0, U increases as r

increases. D. When r is very large, U is close to

zero.

E. When U is very large, the force between the two molecules is repulsive.

38. A fixed mass of an ideal gas goes through

the three processes from P to S indicated in the above p-T diagram: (1): P → Q, (2): Q → R, (3): R → S. During which of the above processes is work done by the gas on the surroundings?

A. (1) only B. (3) only C. (1) and (2) only D. (1) and (3) only E. (2) and (3) only 39. Which of the following statements

concerning a real gas is/are correct? (1) Collisions between molecules and

the wall of a container are not perfectly elastic.

(2) The volume of the molecules cannot be neglected.

(3) Intermolecular forces cannot be neglected.

A. (1) only B. (2) only C. (3) only D. (1) and (2) only E. (2) and (3) only 40. The energy levels of a certain atom are as

shown. Which of the following may undergo an inelastic collision with the atom?

(1) an electron with kinetic energy 3E (2) a photon with energy 2E

p/kPa

0 T/K P

Q

R S

-6E

-4E

0

Energy

time

I/A

0

1

2T T

time

I/A

0

1

2T T

Page 228: AL Physics 1981-2004 MC

00 AL Physics/M.C./P.9

(3) a photon with energy 3E A. (1) only B. (3) only C. (1) and (2) only D. (2) and (3) only E. (1), (2) and (3) 41. In a series of photoelectric emission

experiments on a certain metal surface, relationships between the following physical quantities were investigated.

f = frequency of incident light I = intensity of incident light i = photoelectric current K = maximum kinetic energy of

photoelectrons Two of these quantities, when plotted on a

graph of y against x, would give a straight line through the origin. Which of the following correctly identifies x and y? (Assume the frequencies used are greater than the threshold frequency.)

x y A. K i B. f K C. f i D. I K E. I i 42. In the operational amplifier circuit shown

the feedback is provided by a potentiometer PQ. The gain of the amplifier circuit is

(1) zero when the sliding contact is at P. (2) one when the sliding contact is at Q. (3) independent of the resistance value

of the potentiometer PQ. A. (1) only B. (3) only C. (1) and (2) only

D. (2) and (3) only E. (1), (2) and (3) 43. The diagram shows the path of an α-

particle as it approaches a massive nucleus at N. At point P the α-particle is nearest to the nucleus. Which of the following statements is correct?

A. At P the electric potential energy of

the α-particle is at a minimum. B. At P the total energy of the α-

particle is the minimum. C. At P the angular momentum of the α-

particle about N is at a minimum. D. If the initial kinetic energy of the α-

particle was greater, the distance between P and M would be larger.

E. If the atomic number of the nucleus was greater, the distance between P and M would be larger.

44. A GM counter is placed close to and in

front of a radioactive source which emits both α and γ radiation. The count rate recorded is 500 counts per minute while the background count rate is 50 counts per minute. Three different materials are placed in turn between the source and the counter. The following results are obtained.

Material Recorded count rate /

counts per minute (Nil) 500

Cardboard x 1 mm of

aluminium y

5 mm of lead z Which of the following is a suitable set of

values for x, y and z? x y z A. 350 350 150 B. 350 150 50 C. 350 150 0 D. 150 150 50 E. 150 50 50 45. The following equations represent some

typical nuclear reactions:- (I) He Li H Be 4

263

11

94 +→+

P N

y

0 x

+15 V

-15 V

Vout

Q

P

Vin

Page 229: AL Physics 1981-2004 MC

00 AL Physics/M.C./P.10

(II) n He H H 1

042

31

21 +→+

(III) n3 Br La n U 1

08535

14857

10

23592 ++→+

Which of the following descriptions of

these reactions is/are correct? (1) Reaction (I) represents a spontaneous

α-decay. (2) Reaction (II) represents a nuclear

fusion. (3) Reaction (III) represents a chain

reaction. A. (1) only B. (3) only C. (1) and (2) only D. (2) and (3) only E. (1), (2) and (3)

- End of Paper -

Page 230: AL Physics 1981-2004 MC

00 AL Physics/M.C./P.11

Question No. Key 1. D 2. B 3. C 4. A 5. C 6. A 7. C 8. A 9. B 10. B 11. C 12. B 13. A 14. B 15. E 16. B 17. E 18. A 19. D 20. D 21. E 22. A 23. A 24. E 25. D

Question No. Key 26. E 27. B 28. B 29. D 30. C 31. A 32. C 33. C 34. D 35. C 36. C 37. D 38. E 39. E 40. C 41. E 42. D 43. E 44. A 45. D

Page 231: AL Physics 1981-2004 MC

01 AL Physics/M.C./P.1

2001 Hong Kong Advanced Level Examination AL Physics

Multiple Choice Questions

1. A block rests on a rough inclined plane. Which of the following diagrams correctly shows the lines of application of all the forces acting on the block? (The dot • represents the center of mass of the block.)

A. B. C. D. E.

2. Which of the following pairs of forces is/are example(s) of action and reaction?

(1) The centripetal force keeping a

satellite in orbit round the earth and the weight of the satellite.

(2) The air resistance acting on an object falling through the air with terminal velocity and the weight of the object.

(3) The forces of attraction experienced by two parallel wires carrying currents in the same direction.

A. (1) only B. (3) only C. (1) and (2) only D. (2) and (3) only E. (1), (2) and (3) 3. A stone is projected at an angle of 45° to

the horizontal with an initial kinetic energy E. Neglecting air resistance, when the stones is halfway up, its kinetic energy is

A. 4E .

B. 2E .

C. 4

3E .

D. 2

E .

E. E . 4. Which of the following is/are vector

quantities? (1) impulse (2) moment of inertia (3) pressure A. (1) only B. (3) only C. (1) and (2) only D. (2) and (3) only E. (1), (2) and (3)

Page 232: AL Physics 1981-2004 MC

01 AL Physics/M.C./P.2

5. An object accelerates uniformly along a

straight line from X to Z. It passes X and Z with speeds u and v respectively. What is its speed when it passes Y, which is the mid-point of XZ?

A. 2

22 vu +

B. uv

C. vu

uv+

2

D. 2

vu +

E. It cannot be found as the distance between X and Z is not known.

6. For an object oscillating with simple

harmonic motion, which of the following quantities will reach the maximum value when the object is at its maximum displacement?

(1) the restoring force acting on the

object (2) the total potential energy of the

system (3) the speed of the object A. (1) only B. (3) only C. (1) and (2) only D. (2) and (3) only E. (1), (2) and (3) 7. The maximum speed of a simple

harmonic oscillator is 1 ms -1 and its amplitude is 0.5 m. What is the speed of the oscillator when its displacement from the equilibrium position is 0.3 m?

A. 0.2 ms-1

B. 0.36 ms-1

C. 0.4 ms-1

D. 0.6 ms-1

E. 0.8 ms-1

8. The figure shows a swinging simple

pendulum, starting from a point higher than P. Which of the following forces is/are acting on the pendulum bob when it is at P? (Neglect air resistance)

(1) the weight of the bob (2) a force exerted by the string pointing

towards O (3) a third force along the direction of

motion of the bob A. (1) only B. (3) only C. (1) and (2) only D. (2) and (3) only E. (1), (2) and (3) 9. A solid cylinder and a hollow cylinder,

each having the same mass and external radius, are released together from rest, side by side, at the top of a rough inclined plane. Both cylinders roll down the inclined plane without slipping. Which of the following statements is INCORRECT?

A. The solid cylinder has a smaller

moment of inertia about its axis. B. The solid cylinder reaches the

bottom of the incline first. C. The is no work done by each cylinder

against the friction due to the incline. D. The total kinetic energy of each

cylinder is the same at the bottom of the incline.

E. The solid cylinder has greater rotational kinetic energy at the bottom of the incline.

X Y Z O

P

Page 233: AL Physics 1981-2004 MC

01 AL Physics/M.C./P.3

10. A small sphere is connected to one end of a string which passes through a hole on a horizontal frictionless board. The other end of the string is pulled by a downward force F so that the sphere describes a uniform circular motion with radius r as shown.

The force F is gradually increased until

the radius reduces steadily to r/2. What is the ratio of the new kinetic energy of the sphere to its original kinetic energy? (Neglect any friction between the string and the hole.)

A. 4:1 B. 2:1 C. 1:1 D. 1:2 E. 1:4 11. The SI unit of time, second, used to be

defined as 9 192 631 770 times the period of the electromagnetic wave emitted from a caesium-133 atom. In which region of the electromagnetic spectrum is this wave to be found?

A. infra-red B. ultra-violet C. radio wave D. microwave E. visible 12. In which of the following cases can the

principle of superposition be applied to two overlapping waves of the same nature?

A. Only when the two waves have the

same frequency. B. Only when the two waves have the

same amplitude. C. Only when the two waves travel in

opposite directions. D. Only when the two waves are

coherent.

E. It can be applied in all cases. 13. A student prepares a double-slit set-up as

shown below. However, no interference fringe can be observed on the screen.

Your suggestion for improvement may be A. using a mercury lamp to provide

radiation of a shorter wavelength. B. reducing x. C. reducing D. D. increasing a. E. reducing d. 14. Which of the following statements

concerning Newton’s rings is/are correct? (1) A convex lens with long focal length

should be used. (2) Adjacent rings become farther apart

when moving outwards from the central spot.

(3) The central spot is bright when some liquid is introduced between the convex lens and the reflector.

A. (1) only B. (3) only C. (1) and (2) only D. (2) and (3) only E. (1), (2) and (3) 15. What is the intensity ratio of a note of 60

dB compared with another note of 40 dB which has the same frequency?

A. 1.5 B. 15 C. 20 D. 100 E. 200

F

r

d = 0.01 m screen

D = 1m x = 0.1 m

sodium lamp

a = 1 mm

Page 234: AL Physics 1981-2004 MC

01 AL Physics/M.C./P.4

16. The focal length of a concave mirror is f. The mirror will produce a real, inverted and diminished image when the distance of the object from the pole of the mirror is

A. greater than 2f. B. equal to 2f. C. between f and 2f. D. equal to f. E. less than f. 17. Three Polaroid sheets P, Q and R are

placed along a straight line with a lamp and a detector as shown.

Initially the directions of polarization of

P and Q are parallel but are both normal to that of R. What happens to the intensity I recorded by the detector when Q is being rotated slowly through 90° until its direction of polarization is parallel to that of R?

A. I remains unchanged. B. I increases throughout. C. I decreases throughout. D. I increases and then decreases. E. I decreases and then increases. 18. A boy sitting at the rim of a rotating

merry-go-round blows a whistle continuously with frequency f0. An observer on the ground hears the sound of the whistle. Which of the graphs below gives the possible variation of the observed frequency f of the sound in a period of revolution T?

A.

B. C. D. E. 19. Kepler discovered that the line joining the

Sun and a revolving planet sweeps out equal areas in equal times. The physical law which most directly explains this phenomenon is

A. Newton’s first law of motion. B. Newton’s third law of motion. C. Newton’s law of universal gravitation. D. conservation of angular momentum. E. conservation of energy.

detector

lamp

P Q R

f

time

f0

T

f

time

f0

T

f

time

f0

T

f

time

f0

T

f

time

f0

T

Page 235: AL Physics 1981-2004 MC

01 AL Physics/M.C./P.5

20. Two insulated uncharged metal spheres X and Y are in contact with each other. A positively charged rod is brought near X without any contact while sphere X is earthed as shown.

At steady state, which of the following

descriptions is/are correct? (1) Sphere X gains electrons. (2) Sphere Y loses electrons. (3) Sphere X acquires a negative electric

potential. (4) Sphere Y acquires a positive electric

potential. A. (1) only B. (1) and (2) only C. (1) and (3) only D. (2) and (4) only E. (1), (2), (3) and (4) 21. The above figure shows a network of

resistors. If a voltage of 100 V is applied across terminals A and B, the potential difference between C and D is 80 V. If the voltage is applied across terminals C and D instead, what is the potential difference between A and B?

A. 80 V B. 60 V C. 40 V D. 20 V E. It cannot be found as the value of R is

not known.

22. X and Y are bulbs with ratings ‘6 V, 12 W’ and ‘6 V, 3 W’ respectively. If they are connected to a 12 V supply of negligible internal resistance, which of the following connections allows the two bulbs to work at their respective rated values?

A. B. C. D. E. 23. The two plates of a parallel-plate

capacitor initially carry equal amounts of positive charge. If some charges are transferred from one plate to another, the charges on the plates are respectively + 900 µC and + 100 µC. The potential difference across the plates becomes 4 V. What is the capacitance of the capacitor?

A. 25 µF B. 100 µF C. 125 µF D. 200 µF E. 225 µF

X Y

10 Ω 60 Ω

10 Ω 60 Ω

A

B

C

D

R

Y

X

Y

X

X Y

X Y

Y

X

Page 236: AL Physics 1981-2004 MC

01 AL Physics/M.C./P.6

24. A ‘jumbo’ capacitor has capacitance 100 000 µF. Such a capacitor charged to 20 V would

(1) provide a mean current of 1 mA for

2000 s. (2) be completely discharged through a

10 Ω resistor in 1 s. (3) store 40 J of electrostatic energy. A. (1) only B. (3) only C. (1) and (2) only D. (2) and (3) only E. (1), (2) and (3) 25. If the separation between the plates of a

parallel-plate capacitor is increased by 20%, its capacitance will

A. decrease by 17%. B. increase by 17%. C. decrease by 20%. D. increase by 20%. E. decrease by 25%. 26. Magnetic flux is measured in weber.

Which of the following is/are equivalent to weber?

(1) henry-ampere (2) volt-second (3) tesla-metre A. (1) only B. (3) only C. (1) and (2) only D. (2) and (3) only E. (1), (2) and (3) 27. Which of the following properties is NOT

common to both electric and magnetic fields?

A. Both can be ‘shielded’ using suitable

materials. B. Both can exert attractive and

repulsive forces. C. Both can deflect moving charged

particles. D. Both can be represented by field

lines of closed loops. E. Both can store energy.

28. The figure shows a light and flexible

conducting loop C freely hung on a smooth horizontal rail. A bar magnet PQ approaches the loop from the right. Which of the following descriptions about this process is correct?

A. If P is an N-pole, the loop will be

repelled to the left and its area will increase slightly.

B. If P is a S-pole, the loop will be repelled to the left and its area will increase slightly.

C. If P is an N-pole, the loop will be attracted to the right and its area will decrease slightly.

D. No matter whether P is an N-pole or a S-pole, the loop will be repelled to the left and its area will increase slightly.

E. No matter whether P is an N-pole or a S-pole, the loop will be repelled to the left and its area will decrease slightly.

29. Which of the following descriptions

about a moving-coil meter is/are correct? (1) It has a massive soft iron core to

provide damping. (2) It has curved magnetic poles to assist

in producing linear scale. (3) It has weak hair springs to increase

the sensitivity. A. (1) only B. (3) only C. (1) and (2) only D. (2) and (3) only E. (1), (2) and (3)

C

P Q

Page 237: AL Physics 1981-2004 MC

01 AL Physics/M.C./P.7

30. In an a.c. circuit, two components, one of resistance 5 Ω and the other of inductive reactance 10 Ω, are connected in series to the source. The r.m.s. current is 2 A. What is the average power dissipated as heat in the circuit?

A. 10 W B. 20 W C. 40 W D. 45 W E. 60 W 31. Which of the following waveforms

CANNOT be obtained by applying sinusoidal voltages to both X- and Y-inputs of a CRO?

A. B. C. D. E.

32. Which of the following is NOT a basic assumption of the kinetic theory of an ideal gas?

A. All molecules are in random motion. B. All molecules move with the same

speed at a certain temperature. C. All molecules are point particles that

have no physical size. D. All collisions are perfectly elastic. E. All molecules do not exert force on

one another except during collisions. 33. The first law of thermodynamics can be

expressed as ∆U = Q + W. When an ideal gas is compressed while keeping its pressure constant, which of the following holds?

Q ∆U W A. positive negative negative B. positive positive positive C. zero negative negative D. negative negative negative E. negative negative positive 34. The concept that “energy change occurs in

discrete and not continuous amounts” is NOT REQUIRED to explain

A. absorption line spectrum. B. nuclear fission. C. emission of γ-rays in radioactive

decay. D. photoelectric effect. E. X-ray line spectrum emitted in X-ray

tubes. 35. The interatomic force between pairs of

similar atoms consists of two components. These are

(1) a short-range repulsive force which

varies inversely as r6, (2) a long-range attractive force which

varies inversely as r3, where r = interatomic separation. Given that a and b are positive constants,

and repulsive force is taken as positive, which of the following correctly expresses in mathematical form the interatomic potential energy of a pair of atoms?

Page 238: AL Physics 1981-2004 MC

01 AL Physics/M.C./P.8

A. 47 r

bra

B. 36 r

bra

+−

C. 36 r

bra

D. 25 r

bra

+−

E. 25 r

bra

36. In an experiment to determine the Young

modulus for a steel wire, a student obtained the following data:

length of steel wire = 1.96 ± 0.01 m diameter of steel wire = 0.61 ± 0.01 mm mass of the load = 10.00 ± 0.01 kg extension = 3.9 ± 0.1 mm acceleration of free fall = 9.8 ± 0.1

ms-2 Which of the following leads to the

greatest uncertainty in the calculated value of the Young modulus?

A. measurement of length B. measurement of diameter C. measurement of load D. measurement of extension E. assumed value of the acceleration of

free fall 37. The graph above shows the tensile stress -

tensile strain curves for three materials X, Y and Z up to their breaking points.

Which of the following statement is/are correct?

(1) X is stiffer than Y. (2) Y is stronger than Z. (3) Z can be stretched to twice its

original length without breaking. A. (1) only B. (3) only C. (1) and (2) only D. (2) and (3) only E. (1), (2) and (3) 38. The above diagram shows the steady flow

of water through a horizontal uniform pipe with a central narrow section near Y. The water levels in manometers at X, Y and Z indicate the pressure in each section. The levels in Y and Z are NOT shown. Which of the following statements is/are correct?

(1) The water speed in the narrow section

is greater than the speeds in other sections.

(2) The water speed in section Z is equal to that in section X.

(3) The water level in manometer Y is the highest.

A. (1) only B. (3) only C. (1) and (2) only D. (2) and (3) only E. (1), (2) and (3) 39. A certain photocell emits electrons when

illuminated with yellow light. This photocell will PROBABLY NOT emit electrons when illuminated with

A. blue light. B. green light. C. red light. D. ultra-violet radiation. E. X-rays.

strain

stress/MPa

600

400

200

0 0.5 1.0 1.5 2.0

X

Y

Z

X Y Z

Page 239: AL Physics 1981-2004 MC

01 AL Physics/M.C./P.9

40. Which of the following electron transitions between energy levels in an atom will emit electromagnetic radiation of the highest frequency?

A. n = 2 to n = 1 B. n = 3 to n = 2 C. n = 4 to n = 3 D. n = 4 to n = 2 E. n = 5 to n = 2 41. In the above transistor circuit, the voltage

across the base and the emitter is 0.6 V when the transistor conducts. The current amplification factor of the transistor is 100. If the output voltage is 2 V, what is the value of R?

A. 150 kΩ B. 270 kΩ C. 300 kΩ D. 540 kΩ E. 600 kΩ 42. The activity of a radioactive sample was

70 Bq at time t = 5 minutes and 49 Bq at t = 10 minutes. Deduce its activity at time t = 0.

A. 112 Bq B. 100 Bq C. 95 Bq D. 91 Bq E. 80 Bq 43. Isotopes must have the same (1) nuclear binding energy. (2) number of nucleons. (3) number of protons. A. (1) only

B. (3) only C. (1) and (2) only D. (2) and (3) only E. (1), (2) and (3) 44. Given: mass of proton = 1.0078 u mass of neutron = 1.0087 u mass of deuteron H2

1 = 2.0146 u 1 u is equivalent to 931 MeV Calculate the binding energy per nucleon,

in MeV, of a deuteron. A. 9.5 × 10-4 B. 1.9 × 10-3 C. 8.8 × 10-1 D. 9.4 × 102 E. 1.9 × 103 45. The table below gives the corrected count

rate (in counts per minute) from three samples of radioactive isotopes at three different times.

Isotope 0 min 20 min 40 min

X 480 243 119 Y 135 32 9 Z 168 118 93

The above data show that (1) X produces the most penetrating

radiation. (2) Y has the largest decay constant. (3) Z has the longest half-life. A. (1) only B. (3) only C. (1) and (2) only D. (2) and (3) only E. (1), (2) and (3)

- End of Paper -

R 2 kΩ

Vout

0 V

6 V

Page 240: AL Physics 1981-2004 MC

01 AL Physics/M.C./P.10

Question No. Key 1. E 2. B 3. C 4. A 5. A 6. C 7. E 8. C 9. E 10. A 11. D 12. E 13. E 14. A 15. D 16. A 17. D 18. D 19. D 20. A 21. C 22. D 23. B 24. A 25. A

Question No. Key 26. C 27. D 28. E 29. D 30. B 31. C 32. B 33. E 34. B 35. E 36. B 37. E 38. C 39. C 40. A 41. B 42. B 43. B 44. C 45. D

Page 241: AL Physics 1981-2004 MC

02 AL Physics/M.C./P.1

2002 Hong Kong Advanced Level Examination AL Physics

Multiple Choice Questions

1.

Two identical blocks X and Y are

connected by a light spring and the system is suspended from a fixed support by a light cord as shown. The system is in static equilibrium. If the cord is suddenly cut, what is the magnitude of the acceleration of each block at that instant?

Acceleration of X Acceleration of Y A. g g B. g 2g C. zero g D. zero 2g 2. Two small identical objects P and Q are

released from rest from the top of a building 80 m above the ground. Q is released 1 s after P. Neglecting air resistance, what is the maximum vertical separation between P and Q in the air?

A. 5 m B. 10 m C. 35 m D. 45 m 3. The SI unit of length, metre, is defined as

n times the wavelength of a certain orange-red light emitted from a krypton-86 atom. The number n is of the order

A. 104. B. 106. C. 108. D. 1010.

4.

A block of mass m falls from rest at A

onto a vertical light spring of force constant k firmly fixed to the ground. When the block reaches B, it starts to compress the spring until it reaches the lowest position C as shown. Neglecting air resistance and considering the motion of the block from A to C, which of the following statement is/are correct?

(1) The kinetic energy of the block is at a

maximum when it is at B. (2) The resultant force acting on the

block is zero when it is at C. (3) The maximum compression of the

spring is greater than k

mg.

A. (1) only B. (3) only C. (1) and (2) only D. (2) and (3) only 5.

A bob of mass m is suspended from a

fixed support by a light string. A force F is applied to the bob (F is not shown in the figure) so that the string makes an angle θ with the vertical. What is the minimum value of F required to keep the bob in static equilibrium as shown?

A

B

C

θ

X

Y

Page 242: AL Physics 1981-2004 MC

02 AL Physics/M.C./P.2

A. mg sin θ B. mg cos θ C. mg sin θ cos θ D. mg sin2θ 6.

P, Q and R are three identical spheres

moving with the same speed along a smooth horizontal track. They undergo head-on elastic collisions with spheres X, Y and Z respectively which are initially at rest. After collision, P continues to move in its original direction, Q becomes stationary while R reverses its motion. After collision, which sphere (X, Y or Z) has the greatest magnitude of

(1) momentum (2) kinetic energy? (1) (2) A. Y Z B. Y Y C. Z X D. Z Y 7. The impulse experienced by an object is

equal to its change in A. momentum. B. kinetic energy. C. velocity. D. acceleration. 8. A particle P is moving in a horizontal

circle in a clockwise direction as shown (top view). The following diagrams show the direction of the acceleration av of the particle. Which of them is/are possible?

A. (1) only

B. (3) only C. (1) and (2) only D. (2) and (3) only 9. An object undergoes a simple harmonic

motion with an amplitude A, and its total energy is E. What is the displacement of the object from the equilibrium position

when its kinetic energy is 4

3E ?

A. 2A

B. 4A

C. 4

3A

D. 23A

10.

The graph describes the motion of a

simple harmonic oscillator. Which pair of physical quantities is most likely represented by variables y and x?

y x A. Displacement Acceleration B. Force Displacement C. Kinetic energy Displacement D. Speed Displacement 11.

A thin uniform rod of mass 0.1 kg and

length 0.3 m is smoothly hinged at its lower end as shown. It is then released from rest from the vertical position.

P X Q Y R Z

0 x

y

a

P

a

P a

P

(1) (2) (3)

Page 243: AL Physics 1981-2004 MC

02 AL Physics/M.C./P.3

What is its angular speed, in radian per second, when it becomes horizontal? (Moment of inertia of the rod about one end is 3 × 10-3 kg m2.)

A. 6.3 B. 10 C. 14 D. 50 12. Due to air resistance, changes may occur

to a satellite orbiting the earth (assuming nearly circular orbit). Which of the changes below is incorrect?

A. The total mechanical energy of the

satellite will decrease. B. The angular momentum of the

satellite about the earth’s center will decrease.

C. The linear speed of the satellite will increase.

D. The time needed for the satellite to complete one revolution will increase.

13.

The waveforms in the figure show the time variation of two physical quantities P and Q. What is the phase relationship between P and Q?

A. P leads Q by

π2

.

B. P leads Q by

π4

.

C. Q leads P by

π2

.

D. Q leads P by

π4

.

14. In which of the following the

phenomenon of polarization cannot be observed?

A. Light reflected from the surface of

water. B. Microwaves emitted from a

transmitter.

C. Light scattered from a clear sky. D. Ultrasonic waves used in sonar. 15.

When a certain monochromatic light is

passed through a diffraction grating, a pattern of maxima and minima is observed as shown. Which of the following combinations would produce the largest angle θ between the first-order maxima?

Grating Colour of (lines per mm) light used A. 200 blue B. 200 red C. 400 blue D. 400 red 16.

In the figure, L1 and L2 are two thin lenses

with the same focal length 40 cm placed coaxially 20 cm apart. A beam of light originating from a distant object is incident on L1. Which of the following statements about the final image formed by the system of lenses is correct?

A. It is real and formed on the right of

L2. B. It is real and formed between L1 and

L2. C. It is virtual and formed between L1

and L2. D. It is virtual and formed on the left of

L1.

grating

θ

incident light

first-order maximum

central maximum

first-order maximum

20 cm

L1 L2 time

Q

P

Page 244: AL Physics 1981-2004 MC

02 AL Physics/M.C./P.4

17. A stretched string is 1 m long and is fixed at both ends. Stationary waves of 400 Hz and 450 Hz but no other intermediate frequency can be formed. What is the speed of the mechanical waves along the string?

A. 25 ms-1 B. 50 ms-1 C. 100 ms -1 D. 200 ms -1 18. When a source emitting sound waves of a

fixed frequency moves towards a stationary observer in air, the pitch of the sound heard by the observer is different from that when the source is at rest. This is due to a change in the

(1) number of waves emitted per second

from the source. (2) speed of sound in air. (3) wavelength of sound in air. A. (1) only B. (3) only C. (1) and (2) only D. (2) and (3) only 19. The spectra produced by element X, one

on earth and the other on a distant star, are compared.

Spectrum form element X on earth:

Spectrum from element X on the distant

star:

What is the most probable reason for the

difference in the spectra readings? A. The universe is expanding. B. The star is approaching the earth. C. The star is receding from the earth. D. The star is spinning about its axis.

20.

In the above figure the solid lines

represent part of an electric field due to a fixed point charge Q (not shown in the figure). A charge particle q, subjected only to electric force in the field, travels along the dotted curve shown. Which of the following conclusions can be drawn?

A. q is traveling from X to Y. B. The charge of q has the same sign as

that of Q. C. q has a greater speed at X than at Y. D. The electric potential at X due to Q is

higher than that at Y. 21.

The above figure shows an electron

entering a uniform field which may be electric or magnetic. Which of the following descriptions about the subsequent motion of the electron is correct?

A. Only in a magnetic field can the

electron be deflected by more than 90°.

B. Only in an electric field does the force depend on the magnitude of the charge on the electron.

C. Whether the field is electric or magnetic, the speed of the electron will increase.

D. Whether the field is electric or magnetic, the magnitude and direction of the force acting on the electron are constant.

Red Green Violet

X

Y

Page 245: AL Physics 1981-2004 MC

02 AL Physics/M.C./P.5

22.

Three identical capacitors, each of

capacitance C, form a network as shown. What is the equivalent capacitance between terminals X and Y?

A. C

B. C23

C. C32

D. 3C 23.

A parallel-plate capacitor is first charged

by connecting it to a battery. After disconnecting the battery, one of the plates of the capacitor is shifted upward slightly as shown. What would happen to each of the following quantities?

V: the potential difference between the

plates Q: the magnitude of the charge on each

plate C: the capacitance of the capacitor V Q C A. decreased unchanged increased B. unchanged decreased decreased C. increased decreased decreased D. increased unchanged decreased

24.

In the above circuit, the d.c. supply has an

e.m.f. of 9 V and its internal resistance is negligible. V1, V2 and V3 are three voltmeters with the same finite internal resistance. If V1 reads 4 V, find the reading of V3 and the potential difference across resistor R.

Voltmeter reading Potential of V3 difference across R A. 3 V 1 V B. 5 V 1 V C. 1 V 5 V D. 1 V 3 V 25. A d.c. supply of constant e.m.f. and

internal resistance is connected to a variable resistor of resistance R. Which of the following graphs best shows how the total power P delivered by the supply varies with R?

A.

B.

0 R

P

X Y

0 R

P

V1 V2

V3

R S

9 V

Page 246: AL Physics 1981-2004 MC

02 AL Physics/M.C./P.6

C.

D.

26. A capacitor is discharged through a

resistance of 1 kΩ. If the time constant of the discharging circuit is 2 ms, what is the capacitance of the capacitor?

A. 0.5 µF B. 1 µF C. 2 µF D. 5 µF 27.

A metal rod of length l is inclined at 60°

to rail PQ as shown. It is moved across a uniform magnetic field along the direction of the two horizontal rails PQ and RS. If the rod moves with a uniform velocity v and the flux density of the field is B, the e.m.f. induced in the rod is

A. 2

Blv .

B. Blv.

C. 3

2Blv .

D. 2

3Blv .

28.

A circular frame and a square frame, made

from the same type of insulated metal wires, are placed in a uniform magnetic field as shown. When the flux density of the field is increased at a steady rate, the ratio of the induced current in the circular frame to that in the square frame is

A. 1 : 1 B. 1 : π C. π : 4 D. 2 : π 29.

Three long straight parallel wires P, Q and

R carrying currents of the same magnitude are situated at the vertices of an equilateral triangle as shown. The currents in wires P and R are directed out of the paper. Which of the following indicates the direction of the force acting on wire P?

A. ?

B. ?

C. ?

D. ?

uniform magnetic

field

0 R

P

0 R

P

Q

P

R

60°

Q P

S R

l v

Page 247: AL Physics 1981-2004 MC

02 AL Physics/M.C./P.7

30. A steadily running motor draws a current

of 2 A from a battery of e.m.f. 5 V. The internal resistance of the battery is negligible. If the resistance of its armature coil is 0.5 Ω, estimate the efficiency of the motor.

A. 60% B. 70% C. 80% D. 90% 31.

In the circuit shown, the voltages across

the resistor and capacitor are respectively fed to the input terminals Y1 and Y2 of a dual trace CRO where E is the common earth terminal. Which of the following pairs of traces could be obtained? (The trace of Y1 is represented by a solid curve and the trace Y2 is represented by a dotted one.)

A.

B.

C.

D.

32. To produce an X-ray photon of energy 3.6

× 10-15 J, the bombarding electrons in the X-ray tube must be accelerated through a potential difference of at least

(Given: electronic charge = 1.6 × 10-19 C) A. 11.3 kV. B. 22.5 kV. C. 28.8 kV. D. 57.6 kV. 33.

Which of the following combinations of a

cell, an ideal diode and a resistor will give the above I-V relationship, where I and V are the applied current and voltage respectively?

A.

B.

C.

I

V

Y1 E Y2

Page 248: AL Physics 1981-2004 MC

02 AL Physics/M.C./P.8

D.

34. Which of the following is/are the

advantage(s) of using negative feedback in an operational amplifier for voltage amplification?

(1) greater stability (2) larger voltage gain (3) no distortion of the output A. (1) only B. (3) only C. (1) and (2) only D. (2) and (3) only 35. A beam of red light falls on one electrode

of a photocell and electrons are emitted. The red beam is then replaced by a blue one that has the same intensity. Which fo the following physical quantities would decrease as a result of this change?

A. The maximum kinetic energy of the

photoelectrons emitted B. The energy of each photon striking

the electrode C. The number of photons striking the

electrode per second D. The magnitude of the potential

difference across the photocell required to reduce the photo-electric current to zero

36. In which of the following calculations is

it necessary to use the Planck constant? (1) Finding the momentum of an electron

from its kinetic energy (2) Finding the energy of a photon

emitted as a result of the transition of an electron between two energy levels in an atom

(3) Finding the maximum kinetic energy of photoelectrons from the wavelength of incident radiation

A. (1) only B. (3) only C. (1) and (2) only D. (2) and (3) only

37. A hydrogen atom absorbs a photon of wavelength λ such that the electron in the ground state (energy level corresponding to n = 1) is brought to an excited state (energy level corresponding to n = 3). What is the maximum wavelength of a photon that can cause ionization of a hydrogen atom in the ground state?

A. 9

B. 8

C. 2

D. 32λ

38.

The graph shows the relation between the pressure P and the absolute temperature T of a fixed mass of an ideal gas, which changes from state A to state B along the path AB. Which of the following statements is/are correct?

(1) The graph shows that P is directly

proportional to T. (2) The volume V of the gas increases. (3) All the points on straight line AB

satisfy the relation T

PV = constant.

A. (1) only B. (3) only C. (1) and (2) only D. (2) and (3) only

P

T 0

A

B

Page 249: AL Physics 1981-2004 MC

02 AL Physics/M.C./P.9

39. What is the order of magnitude of the number of molecules in 1 cm3 of air in an atmospheric pressure of 105 Pa and at room temperature?

(Given: Universal gas constant = 8.31 JK-

1mol-1 Avogrado constant = 6.02 × 1023 mol-1) A. 1025

B. 1019 C. 1016 D. 1013 40. At 80 °C, the r.m.s. speed of the

molecules in a fixed mass of an ideal gas is c. If the temperature is increased to 160 °C, the r.m.s. speed of the gas molecules would become

A. 2c. B. 1.4c. C. 1.2c. D. 1.1c. 41.

The graph above shows how the logarithm

of the activity A of a radioactive isotope varies with time t. What is the half-life of the isotope?

A. 200 s B. 55 s C. 24 s D. 0.42 s 42. Which of the following situations cannot

be explained by using Bernoulli’s equation?

(1) An object falling with terminal

velocity in air (2) The curved path described by a

spinning tennis ball (3) The flow of water out of a tank

through a small hole near its bottom

A. (1) only B. (3) only C. (1) and (2) only D. (2) and (3) only 43.

The figure shows a typical voltage-

amplifier circuit built from a transistor with current gain 80. It operates normally on a quiescent collector current of 3 mA and with the base-emitter junction voltage VBE = +0.6 V. What should be the value of the base-bias resistor RB?

A. 128 kΩ B. 144 kΩ C. 160 kΩ D. 176 kΩ 44.

The figure shows an operational amplifier

circuit. When V1 = +2.7 V and V0 = -0.9 V, what is the potential at P?

A. 0 V B. 0.6 V C. 2.1 V D. 2.4 V 45. The diameter of the bore of a capillary

tube can be determined by introducing a small quantity of mercury into the capillary. It is possible to measure the

ln(A/Bq)

t/s 0 400

5

+ 6V

0 V

RB

B

C

E

0 V

10 kΩ

50 kΩ

+15 V

- 15 V V0 V1

V2

10 kΩ

50 kΩ

Page 250: AL Physics 1981-2004 MC

02 AL Physics/M.C./P.10

length of the mercury thread to within 2% and the mass of the mearcury used to within 4%. Assuming negligible error in the density of mercury, the percentage error in the calculated diameter of the capillary bore is at most

A. 2%. B. 3%. C. 4%. D. 6%.

- End of Paper -

Page 251: AL Physics 1981-2004 MC

02 AL Physics/M.C./P.11

Question No. Key 1. 2. 3. 4. 5. 6. 7. 8. 9. 10. 11. 12. 13. 14. 15. 16. 17. 18. 19. 20. 21. 22. 23. 24. 25.

Question No. Key 26. 27. 28. 29. 30. 31. 32. 33. 34. 35. 36. 37. 38. 39. 40. 41. B 42. B 43. B 44. C 45. D

Page 252: AL Physics 1981-2004 MC

03 AL Physics/M.C./P.1

2003 Hong Kong Advanced Level Examination AL Physics

Multiple Choice Questions

1. A uniform flexible chain of mass m is

hung between two fixed supports X and Y at the same horizontal level as shown. The chain makes an angle θ with the horizontal at each support. Find the tension in the chain at its lowest point.

A. 2

mg

B. θsin2

mg

C. θcos2

mg

D. θtan2

mg

2. Two wedge shaped blocks, each of mass

m, are placed on a smooth horizontal surface as shown. A horizontal force F is applied to one of the blocks so that the two blocks move together to the left with constant acceleration. What is the maximum value of F so that the blocks are moving without relative slipping? (Assume all contact surfaces are smooth.)

A. 2 mg

B. 3 mg

C. 2 mg D. mg 3. The figure shows part of the stroboscopic

picture of a particle initially projected horizontally into the air. The side of each square of the grid is 5 cm long.

Estimate the frequency of the strobe lamp used.

(Neglect air resistance.) A. 5.8 Hz B. 7.1 Hz C. 10.0 Hz D. 12.5 Hz 4. The two arms of a beam balance are of

slightly different lengths. An unknown mass is weighed first on the left pan and then on the right pan. The two measurements are m1 and m2 respectively. The actual mass is

A. 2

)( 21 mm +.

B. 2

21mm.

C. 2

)( 22

21 mm +

.

D. 21mm . 5. A light spring is fixed to a block Q which

rests on a horizontal surface as shown. Block P is moving with constant velocity towards Q and makes a head-on elastic collision with Q. After the collision, P

θ θ Y X

45°

F

5 cm

5 cm

P Q

Page 253: AL Physics 1981-2004 MC

03 AL Physics/M.C./P.2

reverses its direction of motion, while Q moves to the right. All contact surfaces are smooth. When will the spring be under a state of maximum compression?

A. When the two blocks move with the

same velocity. B. When block P is momentarily at rest. C. When block P starts to reverse its

direction of motion. D. When block Q starts to move to the

right. 6. A bullet is shot through a wooden block

along the horizontal direction XY. Which of the following graphs shows the variation of the horizontal speed v of the bullet with distance d along XY? (Neglect air resistance.)

A. B. C.

D. 7. A simple pendulum is pulled horizontal

and then released from rest with the string taut. Which of the following statements about the tension in the string is not correct when the pendulum reaches its vertical position?

A. The tension equals the weight of the

pendulum bob in magnitude. B. The tension attains its greatest value. C. The tension does not depend on the

length of the pendulum. D. The tension depends on the mass of

the pendulum bob. 8. An object vibrates with simple harmonic

motion. Which of the following will be doubled when the amplitude is doubled?

A. the maximum potential energy B. the maximum momentum C. the period of oscillation D. the total mechanical energy

pendulum in horizontal position

X Y

v

d 0

v

d 0

v

d 0

v

d 0

Page 254: AL Physics 1981-2004 MC

03 AL Physics/M.C./P.3

9. A light spring is mounted horizontally

with one end fixed to a wall and the other end attached to a block. The block is set to oscillate on a smooth horizontal surface as shown. When the block is at its greatest displacement from the wall, a lump of plasticine is placed lightly on it so that they move together in the subsequent motion. Which of the following physical quantities of the system would become smaller?

A. the amplitude of oscillation B. the period of oscillation C. the maximum acceleration of the

system D. the total mechanical energy of the

system 10. A certain periodic driving force is applied

to a system which has a natural frequency of oscillation. This results in the oscillation of the system. What would happen if the frequency of the periodic driving force is not close to the natural frequency of the system?

(1) The system would eventually oscillate

at the frequency of the driving force if some damping forces are present.

(2) The amplitude of oscillation would become infinite if there are no damping forces.

(3) The displacement of the system from its equilibrium position would always be in phase with the driving force.

A. (1) only B. (3) only C. (1) and (2) only D. (2) and (3) only 11. Given that the universal gravitational

constant is 6.7 × 10-11 Nm2kg-2 and the radius of the earth is 6.4 × 106 m, what is the average density of the earth?

A. 5.6 × 103 kg m-3 B. 7.3 × 103 kg m-3 C. 2.3 × 104 kg m-3

D. 6.0 × 1024 kg m-3 12. A satellite of mass m is launched from the

earth’s surface into an orbit at a height of 3R above the earth’s surface, where R is the radius of the earth. What is the gravitational potential energy gained by the satellite during this process?

A. mgR/3 B. mgR/4 C. 2mgR/3 D. 3mgR/4 13. Four students, P, Q, R and S, each made a

number of measurements of the acceleration of free fall g using the simple pendulum method so as to obtain a mean value. Their results are tabulated as follows:

Student Result g/ms-2

P 9.4 ± 0.8

Q 9.6 ± 0.2

R 9.8 ± 0.3

S 9.9 ± 1.2 Which student obtained a result that has

the largest systematic error? A. S B. R C. Q D. P 14. The figure shows the instantaneous

positions of points P, Q and R on a sinusoidal wave propagating to the right. The three points will reach their respective equilibrium positions at different times in the sequence

A. R, P, Q. B. R, Q, P. C. P, R, Q. D. P, Q, R.

P

R

Q

Page 255: AL Physics 1981-2004 MC

03 AL Physics/M.C./P.4

15. In the figure, M and N are two parallel

boundaries separating media (I), (II) and (III). A light ray undergoes total internal reflection and then refraction as shown. Arrange the speeds of light in the three media in descending order.

A. (I) > (II) > (III) B. (I) > (III) > (II) C. (II) > (III) > (I) D. (III) > (I) > (II) 16. Two different types of tinted glass, X and

Y, are used to make filters for sunglasses. A sample of each is placed in turn between a light source and a detector as shown. Both samples are identical in size and shape.

The intensity of the transmitted light

received by the detector for each sample is plotted against the wavelength of light. Which of the following deduction is/are correct?

(1) Sample X is better for absorbing red

light than sample Y. (2) Sample Y is better at protecting the

eye from ultra-violet radiation.

(3) The view is dimmer when seen through sample Y than through sample X.

A. (1) only B. (3) only C. (1) and (2) only D. (2) and (3) only 17. In the following ray diagrams, L is a

convex lens and F is one of its principal foci. Which of the ray diagrams is/are possible?

(1) (2) (3) A. (1) only B. (3) only C. (1) and (2) only D. (2) and (3) only 18. The average sound intensity level of

human speech is about 70 dB. When four people are talking at the same time, what would be the approximate sound intensity level?

A. 71 dB B. 74 dB C. 76 dB D. 130 dB

(I)

(II)

(III)

M

N

F

L

F

L

F

L

400 500 600 700 800 wavelength/nm

Level without sample

Y X

Intensity

Glass

Detector Light source

Page 256: AL Physics 1981-2004 MC

03 AL Physics/M.C./P.5

19. Light of wavelength 500 nm is incident normally on a plane diffraction grating having a ruling of 5000 lines per cm. Which of the following descriptions is/are correct?

(1) The spacing between the rulings is

2000 nm. (2) The angle through which the

second-order spectrum is diffracted is 30°.

(3) No third-order spectrum is observable. A. (1) only B. (3) only C. (1) and (2) only D. (2) and (3) only 20. Typical microscopes and telescopes are

both built from two converging lenses. In which of the following ways is a telescope similar to a microscope when both are in normal adjustment?

A. In each the focal length of the

objective lens is shorter than that of the eyepiece.

B. In each the separation of the lenses is equal to the sum of their focal lengths.

C. Each produces an intermediate image which is linearly magnified and inverted.

D. In each the final image is inverted and virtual.

21. Two cylindrical tubes are identical except

that one is open at both ends and the other is closed at one end. The air columns in the tubes are set to oscillate at their respective fundamental frequencies. Which of the following is/are the same for the sound waves in the tubes?

(1) the number of displacement nodes (2) the wave speed (3) the wavelength A. (1) only B. (3) only C. (1) and (2) only D. (2) and (3) only 22. The wavelength of infra-red radiation

ranges from about 1 mm to 1 µm. What is the approximate energy of a photon of red light?

(Given: Planck constant = 6.6 × 10-34 Js) A. 10-19 J B. 10-23 J C. 10-25 J D. 10-30 J 23. The magnitudes of the charges on two

identical small metal spheres are in the ratio 5 : 1. The Coulomb interaction force between them is F1. If they are brought into contact and then separated to their respective original positions, the Coulomb interaction force between them becomes F2. Which of the following is/are possible magnitude ratios of F1 : F2?

(1) 5 : 9 (2) 5 : 4 (3) 5 : 2 A. (1) only B. (3) only C. (1) and (2) only D. (2) and (3) only 24. In the figure, the solid curves are

concentric circles representing a set of equipotential surfaces in an electric field. The dotted curve ABC represents the path of a charged particle moving in the field. Which of the following deductions from the figure is/are correct? (Neglect the effects of gravity.)

(1) The charged particle is always

repelled from the center of the concentric circles.

(2) The speed of the charged particle at A is equal to that at C.

(3) The kinetic energy of the charged particle at B is greater than that at A.

A. (1) only B. (3) only C. (1) and (2) only

B

A

C

Page 257: AL Physics 1981-2004 MC

03 AL Physics/M.C./P.6

D. (2) and (3) only 25. In the above circuit, a battery of e.m.f. 6 V

and negligible internal resistance is connected to three resistors. What are the electric potentials at P before and after switch S is closed?

Before After A. +6 V +3 V B. +3 V +3 V C. +2 V +2 V D. +2 V +3 V 26. In the figure, A is an earthed metal plate

while B is a positively charged metal plate parallel to A. The separation between the plates is much less than the length of each plate. If A is moved closer to B, what would happen to the electric potential of B and the electric field strength between the plates? (Induced charges are not indicated.)

Electric potential of B Electric field Strength A. increase increase B. increase remain unchanged C. decrease decrease D. decrease remain unchanged

27. In the above circuit, the battery has

constant e.m.f. and negligible internal resistance. A high-resistance voltmeter connected across terminals a and b reads 4 V. If a low-resistance ammeter is connected across a and b, the ammeter should read

A. 0.6 A. B. 1.0 A. C. 1.2 A. D. 2.0 A. 28. In the circuit, the battery has constant

e.m.f. 6 V and negligible internal resistance. Initially switch K is open. Switch K is then closed and after some time a steady state is reached. The charges passing switch K during this time interval should be

A. +15 µC, from P to Q. B. +15 µC, from Q to P. C. +35 µC, from P to Q. D. +35 µC, from Q to P. 29. A capacitor is first charged by a battery

and then discharged through a resistor. The experiment is then repeated, but this time with a resistor of twice the resistance value. Which statement about the second experiment as compared to the first is correct?

A. The initial current through the resistor

is doubled. B. The energy dissipated in the resistor

is doubled.

A B

2 Ω 1 Ω

2 Ω 1 Ω

2 Ω

a

b

5 µF 10 µF

5 kΩ 1 kΩ

6 V

K

P

Q

6 V

2 Ω

4 Ω

4 Ω

S

P

Page 258: AL Physics 1981-2004 MC

03 AL Physics/M.C./P.7

C. The total charge flowing through the resistor is doubled.

D. The time for the capacitor to lose half of its charge is doubled.

30. In the above set-up, AB and CD are two

parallel infinitely long wires 20 cm apart, carrying currents I1 and I2 respectively. The magnetic flux density at the point P 10 cm from wire CD is zero. If I2 = 0.6 A, which of the following statements about I1 is correct?

A. 0.2 A flows in the same direction as I2. B. 0.2 A flows in the opposite direction to

I2. C. 1.8 A flows in the same direction as I2. D. 1.8 A flows in the opposite direction to

I2. 31. A charged particle enters a region of

uniform magnetic field whose direction is normal to the initial velocity of the particle. The subsequent path of the particle is as shown.

Which of the following is/are possible

explanations to account for the shape of the path?

(1) The flux density of the magnetic field

has decreased gradually. (2) The charged particle has lost its

charge gradually. (3) The charged particle has lost its

kinetic energy gradually. A. (1) only B. (3) only

C. (1) and (2) only D. (2) and (3) only 32. In the figure, the dotted part represents a

spherical metallic shell A of uniform thickness. The shell is neutral as a whole. A small positively charged object B is placed inside the cavity of the shell but not exactly at its center. Positive and negative charges are then induced in the shell. (Distribution of induced charges is not indicated.)

Which of the following statements is/are

correct when electrostatic equilibrium is reached?

(1) Positive induced charges are

distributed uniformly on the outer surface of the shell.

(2) The electric field at any point within the cavity is zero.

(3) The electric potential at the outer surface of the shell is higher than that at its inner surface.

A. (1) only B. (2) only C. (1) and (3) only D. (2) and (3) only 33. The efficiency of a transformer is always

less than 100%. Which of the following is not a reason for this?

A. eddy current flowing in the iron core B. heat dissipation in the primary coil C. leakage of magnetic flux D. work done against the back e.m.f.

induced in the primary coil

20 cm 10 cm

I2 I1 A C

B D

P

Magnetic field normal to paper

B

Shell A

Cavity

Page 259: AL Physics 1981-2004 MC

03 AL Physics/M.C./P.8

34. The figure shows a rectangular coil PQRS

moving from left to right with a uniform speed across an insulated metal wire in the plane of the coil. The wire carries a steady current I. Which of the following gives the correct sequence for the direction of the current induced in the coil PQRS?

A. Clockwise and then anticlockwise B. Anticlockwise and then clockwise C. Clockwise, then anticlockwise and

finally clockwise again D. Anticlockwise, then clockwise and

finally anticlockwise again 35. In the above circuit the a.c. supply has a

variable frequency but has a constant peak voltage and zero impedance. L is a coil of many turns. Which of the following ways can increase the brightness of the bulb?

(1) Increasing the frequency of the a.c.

supply (2) Inserting a soft iron core into the coil

L (3) Adding a suitable capacitor in series

with L A. (1) only B. (3) only C. (1) and (2) only D. (2) and (3) only 36. An a.c. circuit has a total impedance of 13

Ω and a total reactance of 12 Ω. What is

the average power dissipated in the circuit when the r.m.s. current passing through it is 2 A?

A. 4 W B. 20 W C. 48 W D. 52 W 37. A fixed mass of an ideal gas is contained

in a cylinder with a light piston. In which of the following processes will the internal energy of the gas increase?

(1) The gas is made to expand under

constant pressure. (2) The gas pressure is increased while

keeping the piston at a fixed position. (3) The gas is compressed suddenly so

that heat transfer to or from the gas is negligible.

A. (1) and (2) only B. (1) and (3) only C. (2) and (3) only D. (1), (2) and (3) 38. A fixed mass of an ideal gas undergoes

changes from state X to state Z via state Y as shown in the plot of pressure p against volume V below.

Which graph best shows how the absolute

temperature T of the gas varies with its volume V?

P

Q

S

R

Left Right

I

wire

piston

cylinder

L

5

2

0 1 3 5 V

P X Y

Z

Page 260: AL Physics 1981-2004 MC

03 AL Physics/M.C./P.9

A. B. C. D.

39. The graph shows how the mutual interatomic force F between two atoms varies with their separation r. F is taken to be positive when it is repulsive. A1, A2 and A3 represent the areas of the respective regions as shown.

Which of the following represents the

energy required to separate the atoms to infinity, assuming that they are originally at their equilibrium spacing?

A. A2 + A3 B. A1 + A2 – A3 C. A1 + A2 + A3 D. A2 + A3 – A1 40. Copper plates can be made by hammering

solid copper but glass fibres can only be made by drawing from molten glass. Which comparison below is the best explanation for this difference?

A. Copper has a definite melting point

while glass does not. B. Solid copper has a smaller Young

modulus than solid glass. C. Solid copper shows a plastic

deformation while solid glass does not.

D. Solid copper extends elastically more than solid glass under the same stress.

41. The second line in the Lyman series

(corresponding to the K-shell) of the hydrogen spectrum has a wavelength of 102 nm. What is the wavelength of the first line in the series?

A. 93.5 nm B. 121 nm C. 136 nm D. 153 nm 42. In an experiment on the photoelectric

effect, a student measured the potential Vs

F

r 0

A1

A2 A3

0 1 3 5 V

T

X

Y

Z

0 1 3 5 V

T

X

Y Z

0 1 3 5 V

T

X

Y Z

0 1 3 5 V

T

X

Y

Z

Page 261: AL Physics 1981-2004 MC

03 AL Physics/M.C./P.10

required to prevent photoemission when a metal was illuminated with radiations of varying wavelength λ. His observations led him to plot the graph as shown, but he omitted the axes labels. The correct labels for the axes are

x y

A. λ1

Vs

B. Vs λ C. λ Vs

D. Vs λ1

43. The sun and stars generate their energy

mainly by (1) radioactive decay. (2) nuclear fission. (3) nuclear fusion. A. (1) only B. (3) only

C. (1) and (2) only D. (2) and (3) only 44. A nuclide in a radioactive sample has a

constant chance of 6101

to decay in one

second. What is the approximate half-life of the sample?

A. 1 day B. 1 week C. 1 month D. 1 year 45. The average background count rate, in s-1,

in Hong Kong is of the order of A. 10-2. B. 100. C. 102. D. 104.

- End of Paper -

0

y

x

Page 262: AL Physics 1981-2004 MC

03 AL Physics/M.C./P.11

Question No. Key 1. Deleted 2. A (27) 3. C (30) 4. D (26) 5. A (9) 6. B (79) 7. A (51) 8. B (65) 9. C (50) 10. A (58) 11. A (67) 12. D (42) 13. D (20) 14. A (77) 15. B (50) 16. C (64) 17. B (40) 18. C (82) 19. C (68) 20. D (42) 21. C (57) 22. A (61) 23. C (35) 24. C (49) 25. D (77)

Question No. Key 26. D (43) 27. C (36) 28. A (40) 29. D (70) 30. D (72) 31. B (65) 32. A (24) 33. D (63) 34. C (31) 35. B (60) 36. B (53) 37. D (33) 38. C (37) 39. A (66) 40. C (61) 41. B (32) 42. A (47) 43. B (41) 44. B (73) 45. B (20)

Page 263: AL Physics 1981-2004 MC

04 AL Physics/M.C./P.1

2004 Hong Kong Advanced Level Examination AL Physics

Multiple Choice Questions

1. A small object at point O is kept in static equilibrium by a horizontal thread OA and a light spring OB which makes an angle θ with the vertical as shown.

If the thread is suddenly cut, what are the

direction and magnitude of the acceleration of the object at that instant?

Direction Magnitude A. I g sin θ B. I g tan θ C. II g sin θ D. II g tan θ 2. Two spheres A and B of the same size,

which have masses m and M respectively. A moves horizontally with a speed u and makes a head-on elastic collision with B, which rests on a smooth horizontal surface as shown. Referring to each of the following descriptions about B, how is m compared with M?

(1) B attains the greatest kinetic energy

after collision (2) B attains the greatest magnitude of

momentum after collision (3) B attains the greatest speed after

collision

(1) (2) (3) A. M = m M = m M << m B. M = m M << m M >> m C. M = m M >> m M << m D. M << m M >> m M << m 3. The figure shows the barrel of a gun that

aims directly at a point P 40 m from the muzzle of the gun. The barrel makes an angle θ with the vertical.

If the speed of the bullet is 50 ms-1 when it leaves the gun, calculate the separation between the bullet and point P when the bullet is vertically below P. (Neglect air resistance.)

A. 3.2 m B. 4.8 m C. 7.8 m D. It cannot be found as the value of θ is

not known. 4 . Suppose a looping the loop experiment is

performed on the moon, where the gravitational field strength is 1.6 Nkg-1 on its surface. If the radius of the circular loop is 0.5 m, find the minimum speed v to launch the ball at the lowest point of the smooth track so that it can complete looping without leaving the track.

m

A u

M

B

ball

v

0.5 m

B θ

O II

I

A

Page 264: AL Physics 1981-2004 MC

04 AL Physics/M.C./P.2

A. 0.9 ms-1 B. 1.8 ms-1 C. 2.0 ms-1 D. 4.0 ms-1 5. (1) (2) Figure (1) shows a block of mass m

suspended by a light spring. The extension of the spring is e. The spring is then cut into two identical parts and connected in parallel as shown in Figure (2). If a block of mass 2m is suspended by this set-up as shown, what is the extension of each spring?

A. 2e B. e

C. 2e

D. 4e

6. An object vibrates with simple harmonic

motion. Which of the following graphs describes correctly the variation of the resultant force F acting on the object with the displacement x?

A. B.

C. D. 7. A planet has a diameter 2 times that of the

earth and a mass 3 times that of the earth. What is the approximate gravitational field strength, in Nkg-1, on the planet’s surface?

A. 7.5 B. 10 C. 15 D. 30 8. Long ago, astronomers wanted to find out

whether the ring of Saturn is a rigid body or a group of satellites revolving around Saturn. The linear speeds v of different layers of the ring were measured and how v varies with the distance r from the centre of Saturn was determined. Which of the following relations between v and r supports the suggestion that

(1) the ring is a rigid body, (2) the ring is a group of satellites

revolving around Saturn ? (1) (2)

A. v ∝ r v ∝r1

B. v ∝ r v ∝r

1

C. v ∝ r v ∝r1

D. v ∝ r v ∝r

1

F

x 0

F

x 0

F

x 0

F

x 0 m

2m

Page 265: AL Physics 1981-2004 MC

04 AL Physics/M.C./P.3

9. Two waves P and Q of equal amplitude

and frequency arrive at the same point X. The figure shows the displacement-time graphs of waves P and Q at point X. What is the phase difference between the resultant wave and wave P at point X?

A. 30° B. 45° C. 60° D. 90° 10. (1) (2) Figure (1) shows two transverse pulses

travelling towards each other along a stretched string. They are identical and symmetric in shape but one is inverted. At the moment when the pulses overlap, the string becomes straight as shown in Figure (2). P, Q and R are points on the string situated symmetrically within the overlapping part. At the instant when the string is straight, which of the following statements about the string’s motion at P, Q and R is correct?

A. P, Q and R are moving. B. P, Q and R are at rest. C. Only Q is moving. D. Only Q is at rest.

11. A Young's double-slit experiment was performed using a monochromatic light source. Which of the following methods would result in a greater fringe separation on the screen?

(1) Using a monochromatic light source

of longer wavelength (2) Using a double slit with greater slit

separation (3) Using a double slit with larger slit

width A. (1) only B. (1) and (2) only C. (2) and (3) only D. (1), (2) and (3) 12. In the figure, P and Q are two

monochromatic light rays. One is red and one is blue. They are parallel to the principal axis of the glass lens L before being refracted by the lens.

Which of the following deductions are

correct? (1) P is the red light ray. (2) L is a concave lens. (3) Q travels faster than P in the lens. A. (1) and (3) only B. (1) and (2) only C. (2) and (3) only D. (1), (2) and (3) 13. A vibrating tuning fork is held near the

mouth of a glass tube as shown.

L

Q

P

P R Q

displacement

time 0

P

Q

Page 266: AL Physics 1981-2004 MC

04 AL Physics/M.C./P.4

Which of the following deductions is INCORRECT if resonance of the air column is observed?

A. The observed loudness is greater than

just with the fork alone. B. There is a longitudinal stationary

wave set up inside the tube. C. There is a pressure antinode at the

closed end of the tube. D. The fundamental frequency of the air

column in the tube must be equal to the frequency of the tuning fork.

14. Given: Atmospheric pressure near the earth’s

surface = 100 kPa Density of air near the earth's surface =

1.3 kg m-3 Radius of the earth = 6 400 km Which of the following is the best

estimate for the total mass of the earth’s atmosphere?

A. 5 × 1012 kg B. 5 × 1015 kg C. 5 × 1018 kg D. 5 × 1021 kg 15. Two positively charged particles A and B

of masses mA and mB respectively are suspended by two insulating threads of the same length from O. Due to the electrostatic repulsive forces between A and B, threads AO and BO make 30° and 60° respectively with the vertical as shown. Find the ratio mA: mB.

A. 3 : 1

B. 2 : 3 C. 2 : 1 D. It cannot be found as the charge on

each particle is not known.

16. In the above figure, a point charge +Q is

placed at A. The resulting electric potential at B is V. If a point charge -2Q is now placed at C, the mid-point between A and B, what is the electric potential at B produced by both point charges? (Assume that the electric potential at infinity is zero.)

A. -4V B. -3V C. -2V D. 0 17. A parallel-plate capacitor with plates A

and B is connected to a battery as shown. Plate A is earthed. P is a point between the plates. If plate B is shifted upwards slightly to position B’ while the capacitor remains connected to the battery, what would happen to each of the following physical quantities?

E : the electric field strength at P V : the electric potential at P W : the energy stored in the capacitor E V W A. decrease increase decrease B. decrease decrease decrease C. remain unchanged decrease decrease D. remain unchanged increase increase

60°

30°

O

A

B

+Q

A C B

A

B B’

P

Page 267: AL Physics 1981-2004 MC

04 AL Physics/M.C./P.5

18. A positively charged particle is projected with a certain initial velocity into the electric field due to a charge fixed at O. Part of its trajectory (curve AB) is sketched as shown. Which of the following deductions is correct?

A. The trajectory must be a part of an

ellipse. B. The acceleration of the particle at B is

greater than that at A. C. The kinetic energy of the particle at B

is greater than that at A. D. The electrical potential energy of the

particle at B is greater than that at A. 19. The resistance of a reverse-biased diode is

to be determined by the ammeter-voltmeter method using moving-coil meters. Which of the following circuits is correctly connected?

A. B. C.

D. 20. In the above network of resistors, the

resistance of S is infinitely large and the two resistors R are identical. If the equivalent resistance across CD is 2.5 kΩ, what is the equivalent resistance across AC?

A. 2.5 kΩ B. 3.5 kΩ C. 5.0 kΩ D. infinitely large 21.

The graph shows the I-V characteristic of two light bulbs X and Y, which are marked respectively as ‘200 V, 100 W’ and ‘200 V, 60 W’. If X and Y are connected in series to a 200 V mains supply, what is the approximate power dissipated in each bulb?

1 kΩ

A

B

D

C

S

R

R

O

A

B

A

V

A

V

A

V

A

V

Page 268: AL Physics 1981-2004 MC

04 AL Physics/M.C./P.6

X Y A. 12 W 36 W B. 15 W 25 W C. 40 W 20 W D. 50 W 30 W 22. In the above circuit the battery has an

e.m.f. of 6 V and internal resistance 2 Ω. S is a standard load resistor of 6 Ω and R is a rheostat. What will the setting of R be if the power delivered by the battery to the load resistor S is at a maximum?

A. 0 Ω B. 2 Ω C. 4 Ω D. 6 Ω 23. A basic meter of 100 µA full-scale

deflection and 1 kΩ internal resistance is converted into a d.c. voltmeter of 5 V full-scale deflection. What is the internal resistance of this voltmeter?

A. 100 kΩ B. 50 kΩ C. 25 kΩ D. 5kΩ 24. Two isolated metal spheres, A and B, each

has a net charge +Q distributed uniformly over its surface. The radius of sphere A is smaller than that of sphere B. Which of the following statements is INCORRECT?

A. Sphere A is at a higher potential. B. Sphere B has a larger capacitance. C. Sphere B stores more electrical

energy. D. If spheres A and B are connected by a

wire, the charge on sphere A will decrease.

25. Two circuits, (1) and (2), each connects

two identical capacitors with a d.c. supply

of e.m.f. E and two identical resistors as shown.

(1) (2) Find the ratio of the total electrical energy

stored in the capacitors in circuit (1) to that in circuit (2) when a steady state is reached.

A. 2 : 1 B. 1 : 2 C. 4 : 1 D. 1 : 4 26. The Hall effect (1) provides evidence that the charge

carriers in metals are negatively charged.

(2) can be used to determine the density of free electrons in a metal.

(3) can be used to determine the flux density of the magnetic field due to an a.c. only.

Which of the above statements are

correct? A. (1) and (3) only B. (1) and (2) only C. (2) and (3) only D. (1), (2) and (3)

S

6 Ω

R 6 V, 2 Ω R R

C C

E

R R

C C

E

Page 269: AL Physics 1981-2004 MC

04 AL Physics/M.C./P.7

27. For the relation

ltageprimary vo

voltagesecondary

= primary in the turnsof no.

secondary in the turnsof no.

to hold for a transformer, which of the following conditions are necessary?

(1) The flux leakage of the iron core is

negligible. (2) The energy loss in the primary coil is

negligible. (3) The secondary coil is on open circuit. A. (1) and (3) only B. (1) and (2) only C. (2) and (3) only D. (1), (2) and (3) 28. The figure shows a secondary coil placed

at one end of a solenoid, which is connected to an a.c. source. The changing magnetic flux in the solenoid induces an alternating voltage in the secondary coil, which is connected to a CRO.

Which of the following changes will NOT

affect the amplitude of the induced voltage in the secondary coil?

A. Increasing the frequency of the a.c. in

the solenoid B. Increasing the number of turns on the

secondary coil C. Placing the secondary coil in the

middle of solenoid, without changing its orientation

D. Replacing the solenoid with one of greater cross-sectional area while keeping the same current

29. Three long, parallel, straight current-carrying wires P, Q and R are placed in the same plane in air as shown.

If F is the force per unit length between

two long, parallel, straight wires, placed a distance d apart and each carrying a current of 1 A, what is the net force per unit length acting on the wire R shown?

A. 0 B. F C. 2F D. 3F 30. In the above LCR series circuit, L is a pure

inductor. Which of the following statements is INCORRECT?

A. Energy is dissipated in resistor R but

not in C or L. B. Electrical energy is transferred

between C and L. C. The impedance of the circuit depends

on the frequency of the a.c. supply. D. The current through C is 180° out of

phase with that through L. 31.

The figure shows how a half-wave

rectified sinusoidal a.c. flowing through a resistor varies with time. What is the value of a steady d.c. that gives the same heating effect as the rectified a.c.?

d

d

1 A P

2 A Q

2 A R

0

2

0.01 0.02 0.03 0.04 0.05 t/s

I/A

L C R

Page 270: AL Physics 1981-2004 MC

04 AL Physics/M.C./P.8

A. 21

A

B. 2

1A

C. 1A

D. 2 A 32. A beam of blue light fal1s on the cathode

of a photocel1 so that electrons are emitted. The blue beam is then replaced by a yel1ow one with the same intensity and electrons are also emitted. What would happen to each of the fol1owing physical quantities when the blue beam is replaced by the yel1ow beam?

K : the maximum kinetic energy of the

electrons emitted I : the magnitude of the photoelectric

current K I A. increase remain unchanged B. decrease remain unchanged C. decrease increase D. decrease decrease 33. The ionization energy for a hydrogen atom

in ground state is 13.6 eV. If the atom is in the first excited state, the energy for ionizing it should be

A. 3.4 eV. B. 4.5 eV. C. 6.8 eV. D. 10.2 eV. 34. Which of the following descriptions about

spectra is/are correct? (1) The emission spectrum from a very

hot liquid is continuous. (2) The dark lines in the solar spectrum

illustrate that the elements corresponding to these dark lines are absent from the sun.

(3) The dark lines in the solar spectrum are due to the solar radiation being absorbed by the elements in the earth's atmosphere corresponding to these dark lines.

A. (1) only B. (3) only C. (1) and (2) only D. (2) and (3) only

35. The annual world fuel consumption C

increases every year. It is asserted that C has increased exponentially in the past twenty years (i.e. C = C0e

kt where k is a positive constant and t is the time). Which of the following mathematical procedures are suitable for testing this assertion?

(1) Plot a graph of log C against time and

see whether it is a straight line. (2) Plot a graph of C against time and see

whether it takes the same period of time for the value of C at any time to double.

(3) Calculate a series of values of

yearprevioustheforCyearcertainaforC

and see

whether they are the same. A. (1) and (3) only B. (1) and (2) only C. (2) and (3) only D. (1), (2) and (3) 36. In which of the following graphs does the

shaded area represent an amount of energy?

(1) Q = charge on a capacitor V = p.d. between plates (2) e = extension of a wire F = force causing the extension

0 Q

V

0 e

F

Page 271: AL Physics 1981-2004 MC

04 AL Physics/M.C./P.9

(3) V = volume occupied by a gas p = pressure exerted by the gas A. (1) only B. (3) only C. (1) and (2) only D. (2) and (3) only 37. An ideal gas of volume V and pressure p

undergoes a change from state X to state Z via state Y along the path as shown in the p-V plot.

Which of the following descriptions about

the gas at X, Y and Z is correct? A. The gas is at its coolest at X and is at

its hottest at Y. B. The gas is at its coolest at X and is at

its hottest at Z. C. The gas is at its coolest at Y and is at

its hottest at X. D. The gas has the same degree of

hotness at X, Y and Z. 38. The advantages of having negative

feedback in a voltage amplifier are that it (1) produces a voltage gain which is

independent of the characteristics of the amplifier.

(2) increases the range of frequency over which the gain is constant.

(3) ensures that the waveform of the output voltage matches the shape of the waveform of the input voltage.

A. (1) and (3) only

B. (1) and (2) only C. (2) and (3) only D. (1), (2) and (3) 39. The figure shows a transistor in a simple

voltage-amplifier circuit operating satisfactorily such that the quiescent collector-emitter voltage is half the voltage of the battery connected. The base-emitter junction voltage VBE is +0.6 V and the transistor d.c. current gain is 50. Find the resistance value of the base-bias resistor RB if the load is 1 kΩ.

A. 45 kΩ B. 50 kΩ C. 90 kΩ D. 100 kΩ 40. The above circuit is used to generate

square waves from sinusoidal waves. The amplitude of the a.c. input voltage is 3 V. Which values for resistors R1 and R2 will produce the best square wave output?

R1/kΩ R2/kΩ A. 1 2 B. 1 10 C. 2 1 D. 10 1 41. Which of the following relations about

units is/are correct?

V

p

0

X

Y

Z

0 V1

p

6 V

1 kΩ RB

B E

C

_ +

V0

R1

R2

+ 9 V

- 9 V

Page 272: AL Physics 1981-2004 MC

04 AL Physics/M.C./P.10

(1) 1 F = 1 sΩ-1 (2) 1 H = 1 Ωs (3) 1 Wb = 1 Vs-1 A. (1) only B. (1) and (2) only C. (2) and (3) only D. (1), (2) and (3) 42. The activity of a freshly prepared sample

of 60Co is 1.0 × 106 Bq. The half-life of 60Co is 5.3 years. Estimate the number of 60Co nuclei in the sample that decay in the first day.

A. 5.2 × 102 B. 3.2 × 108 C. 8.6 × 1010 D. It cannot be estimated as the initial

number of nuclei in the sample is not given.

43. A β-source is placed in front of a GM tube

connected to a counter. The corrected count rate recorded is NOT equal to the activity of the source because

(1) most β-particles cannot pass through

the mica end-window of the GM tube. (2) not all the β-particles are emitted in

the direction of the GM tube. (3) β-particles arriving within the dead

time of the GM tube cannot be resolved.

A. (1) and (3) only B. (1) and (2) only C. (2) and (3) only D. (1), (2) and (3) 44. The following nuclear reaction represents

the two deuterons, H21 , which combine to

form a helium isotope, He32 , with the

release of energy.

energyXHeH2 32

21 ++→

Which of the following statements are

correct? (1) This is an example of nuclear fusion. (2) The total mass of He3

2 and X is

greater than that of the two H21 .

(3) X is a neutron. A. (1) and (3) only B. (1) and (2) only C. (2) and (3) only D. (1), (2) and (3) 45. In which of the following would the

application of the measuring instrument affect the physical quantity being measured?

(1) Using a moving-coil voltmeter to

measure the voltage across a resistor in a circuit

(2) Using a mercury-in-glass thermometer to measure the temperature of a beaker of hot water

(3) Using a spring balance to measure the weight of an object

A. (1) only B. (3) only C. (1) and (2) only D. (2) and (3) only

- End of Paper -

Page 273: AL Physics 1981-2004 MC

04 AL Physics/M.C./P.11

Question No. Key 1. D (34) 2. C (40) 3. A (24) 4. C (42) 5. C (21) 6. B (80) 7. A (82) 8. B (46) 9. B (68) 10. D (31) 11. A (78) 12. B (40) 13. D (39) 14. C (40) 15. A (62) 16. B (76) 17. B (49) 18. D (33) 19. D (34) 20. B (57) 21. A (9) 22. A (57) 23. B (80) 24. C (57) 25. D (61)

Question No. Key 26. B (60) 27. D (21) 28. D (35) 29. D (53) 30. D (44) 31. C (28) 32. C (35) 33. A (48) 34. A (31) 35. D (22) 36. C (71) 37. A (43) 38. B (41) 39. C (56) 40. B (55) 41. B (38) 42. C (25) 43. C (65) 44. A (74) 45. C (36)


Recommended